Com 170 Bat Dang Thuc Can

You might also like

Download as pdf or txt
Download as pdf or txt
You are on page 1of 168

V Quc B Cn Nguyn Vn Thch Nguyn Phi Hng

Phan Hng Sn V Thnh Vn


Collected problems
About inequality
Ngy 19 thng 5 nm 2007
www.VNMATH.com
ii
www.VNMATH.com
Mc lc
1 Problems 1
2 Solution 17
2.1 Li gii cc bi ton . . . . . . . . . . . . . . . . . . . . . . . . . . . . . . . . . . . . . . 17
2.2 Tc gi cc bi ton . . . . . . . . . . . . . . . . . . . . . . . . . . . . . . . . . . . . . . 164
iii
www.VNMATH.com
iv MC LC
www.VNMATH.com
Chng 1
Problems
1. Cho x, y, z l cc s dng tha xy + yz + zx = 1, chng minh
1
_
1 + (2x y)
2
+
1
_
1 + (2y z)
2
+
1
_
1 + (2z x)
2

3
2
2. Cho cc s dng a, b, c tha abc = 1, chng minh rng
a

b + c
b + c + 1
+
b

c + a
c + a + 1
+
c

a + b
a + b + 1

2
3. Vi mi s khng m a, b, c, ta c
_
a
4a + 4b + c
+
_
b
4b + 4c + a
+
_
c
4c + 4a + b
1
4. Cho cc s dng a, b, c, chng minh
1
a
2
+ bc
+
1
b
2
+ ca
+
1
c
2
+ ab

a + b + c
ab + bc + ca
_
1
a + b
+
1
b + c
+
1
c + a
_
5. Chng minh rng vi mi s dng a, b, c ta lun c
a
3
2a
2
ab + 2b
2
+
b
3
2b
2
bc + 2c
2
+
c
3
2c
2
ca + 2a
2

a + b + c
3
6. Cho cc s khng m a, b, c tha a + b + c = 1. Chng minh bt ng thc
_
a +
(b c)
2
4
+
_
b +
(c a)
2
4
+
_
c +
(a b)
2
4

3 +
_
1

3
2
_
(|a b| +|b c| +|c a|)
7. Cho cc s dng a, b, c tha a + b + c = 3, chng minh bt ng thc
a
3/2
b + b
3/2
c + c
3/2
a 3
8. Chng minh rng vi mi s thc a, b, c, ta c
ab
4a
2
+ b
2
+ 4c
2
+
bc
4b
2
+ c
2
+ 4a
2
+
ca
4c
2
+ a
2
+ 4b
2

1
3
1
www.VNMATH.com
2 CHNG 1. PROBLEMS
9. Cho cc s khng m a, b, c tha a + b + c = 3, chng minh

a
2
+ b
2
(a + 1)(b + 1)
+

b
2
+ c
2
(b + 1)(c + 1)
+

c
2
+ a
2
(c + 1)(a + 1)

3

2
10. Vi mi a b c 0, t
P =
a
b + c
+
b
c + a
+
c
a + b
Q =
2(b + c) a
4a + b + c
+
2(c + a) b
4b + c + a
+
2(a + b) c
4c + a + b
Chng minh rng
(a) Nu a + c 2b th P Q.
(b) Nu a + c 2b th P Q.
11. Cho cc s khng m a, b, c tha a + b + c = 1, t x = a
2
+ b
2
+ c
2
, chng minh bt ng thc
_
1 + 2a
2
x +
_
1 + 2b
2
x +
_
1 + 2c
2
x

11 9x
12. Chng minh rng vi mi a, b, c > 0, ta c
1
a(a + b)
+
1
b(b + c)
+
1
c(c + a)

3
2(abc)
2/3
13. Chng minh rng nu a, b, c > 0 th
1
a

a + b
+
1
b

b + c
+
1
c

c + a

3

2abc
14. Cho cc s dng x, y, z tha x
2
+ y
2
+ z
2
3, chng minh rng
x
5
x
2
x
5
+ y
2
+ z
2
+
y
5
y
2
y
5
+ z
2
+ x
2
+
z
5
z
2
z
5
+ x
2
+ y
2
0
15. Cho n 3 v a
1
, a
2
, . . . , a
n
l cc s khng m tha a
2
1
+ a
2
2
+ + a
2
n
= 1, chng minh bt ng
thc
1

3
(a
1
+ a
2
+ + a
n
) a
1
a
2
+ a
2
a
3
+ + a
n
a
1
16. Cho cc s dng a, b, c, chng minh bt ng thc
_
a
b
+
b
c
+
c
a
+
_
ab + bc + ca
a
2
+ b
2
+ c
2

3 + 1
17. Chng minh rng vi mi a, b, c > 0, ta c
a
2
b
2
+
b
2
c
2
+
c
2
a
2
+
8(ab + bc + ca)
a
2
+ b
2
+ c
2
11
18. Chng minh rng vi mi s dng a
1
, a
2
, . . . , a
n
, b
1
, b
2
, . . . , b
n
, ta c
_
n

i=1
a
2
i
__
n

i=1
b
2
i
_

_
n

i=1
b
i
(a
i
+ b
i
)
__
n

i=1
a
2
i
b
i
a
i
+ b
i
_
www.VNMATH.com
3
19. Chng minh rng vi cc s thc a, b, c i mt khc nhau, ta c
(a
2
+ b
2
+ c
2
ab bc ca)
_
1
(a b)
2
+
1
(b c)
2
+
1
(c a)
2
_

27
4
20. Cho cc s khng m a, b, c, d tha a
2
+ b
2
+ c
2
+ d
2
= 4, chng minh bt ng thc
1
3 abc
+
1
3 bcd
+
1
3 cda
+
1
3 dab
2
21. Cho cc s dng a, b, c, chng minh bt ng thc
a
b
+
b
c
+
c
a
3
_
a
2
+ b
2
+ c
2
ab + bc + ca
22. Cho cc s khng m a, b, c, chng minh bt ng thc
7
_
3(a
2
+ b
2
+ c
2
)
a + b + c
+
a
2
b + b
2
c + c
2
a
a
3
+ b
3
+ c
3
8
23. Chng minh rng vi mi s dng a, b, c ta c
a
3
a
3
+ abc + b
3
+
b
3
b
3
+ abc + c
3
+
c
3
c
3
+ abc + a
3
1
24. Cho cc s dng a, b, c, d, chng minh rng
abc
(d + a)(d + b)(d + c)
+
abd
(c + a)(c + b)(c + d)
+
acd
(b + a)(b + c)(b + d)
+
bcd
(a + b)(a + c)(a + d)

1
2
25. Chng minh rng vi mi a, b, c > 0, ta c
a
b+c
+ b
c+a
+ c
a+b
1
26. Cho n 3, n N v x
1
, x
2
, . . . , x
n
l cc s khng m c tng bng 1. Tm gi tr ln nht ca
biu thc
P(x
1
, x
2
, . . . , x
n
) = x
3
1
x
2
2
+ x
3
2
x
2
3
+ + x
3
n
x
2
1
+ n
2(n1)
x
3
1
x
3
2
x
3
n
27. Cho cc s thc a
1
, a
2
, . . . , a
n
tha a
1
a
2
a
n
= 1, tm cc hng s tt nht m, M sao cho
_
a
2
1
+ n
2
1 +
_
a
2
2
+ n
2
1 + +
_
a
2
n
+ n
2
1 m(a
1
+ a
2
+ + a
n
) + M
28. Chng minh rng vi mi s dng a, b, c, d, ta c
a
3a
2
+ 2b
2
+ c
2
+
b
3b
2
+ 2c
2
+ d
2
+
c
3c
2
+ 2d
2
+ a
2
+
d
3d
2
+ 2a
2
+ b
2

1
6
_
1
a
+
1
b
+
1
c
+
1
d
_
29. Cho cc s dng x, y, z, chng minh bt ng thc
x(y + z)
x
2
+ yz
+
y(z + x)
y
2
+ zx
+
z(x + y)
z
2
+ xy

x + y + z
3

xyz

x
2
+ yz
x(y + z)
+
y
2
+ zx
y(z + x)
+
z
2
+ xy
z(x + y)
30. Vi mi s dng a, b, c tha a + b + c = 3, ta c
a
b
2
+ c
+
b
c
2
+ a
+
c
a
2
+ b

3
2
www.VNMATH.com
4 CHNG 1. PROBLEMS
31. Vi mi s khng m a, b, c tha a + b + c = 3, ta c
a
_
b
3
+ 1 + b
_
c
3
+ 1 + c
_
a
3
+ 1 5
32. Tm hng s k tt nht sao cho bt ng thc sau ng vi mi a, b, c > 0
(a + b + c)
_
1
a
+
1
b
+
1
c
_
9 +
k max{(a b)
2
, (b c)
2
, (c a)
2
}
(a + b + c)
2
33. Cho cc s dng x, y, z c tch bng 1, chng minh rng vi mi k 0, ta c
3
_
x
y + k
+
3
_
y
z + k
+
3
_
z
x + k

3
3

k + 1
34. Cho cc s dng a, b, c, chng minh bt ng thc
b
2
+ c
2
a(b + c)
+
c
2
+ a
2
b(c + a)
+
a
2
+ b
2
c(a + b)
(a
2
+ b
2
+ c
2
)

3
abc(a + b + c)
35. Cho cc s dng a, b, c, chng minh bt ng thc
2
_
a
2
b
+
b
2
c
+
c
2
a
_
+ 3(a + b + c)
15(a
2
+ b
2
+ c
2
)
a + b + c
36. Chng minh rng vi mi s thc dng x, y, z c tch bng 1 v vi mi k 0, ta c
4
_
x
y + k
+
4
_
y
z + k
+
4
_
z
x + k

3
4

k + 1
37. Chng minh rng vi mi s khng m a, b, c v vi mi k 3, ta c
a(b
k
+ c
k
)
a
2
+ bc
+
b(c
k
+ a
k
)
b
2
+ ca
+
c(a
k
+ b
k
)
c
2
+ ab
a
k1
+ b
k1
+ c
k1
38. Cho cc s khng m a, b, c, chng minh bt ng thc
a
4
a
3
+ abc + b
3
+
b
4
b
3
+ abc + c
3
+
c
4
c
3
+ abc + a
3

a
3
+ b
3
+ c
3
a
2
+ b
2
+ c
2
39. Cho cc s dng x, y, z, t tha
1
x + 1
+
1
y + 1
+
1
z + 1
+
1
t + 1
= 1
Chng minh rng
min
_
1
x
+
1
y
+
1
z
,
1
y
+
1
z
+
1
t
,
1
z
+
1
t
+
1
x
,
1
t
+
1
x
+
1
y
_
1
max
_
1
x
+
1
y
+
1
z
,
1
y
+
1
z
+
1
t
,
1
z
+
1
t
+
1
x
,
1
t
+
1
x
+
1
y
_
40. Cho cc s khng m a, b, c, chng minh bt ng thc
a
2

4a
2
+ ab + 4b
2
+
b
2

4b
2
+ bc + 4c
2
+
c
2

4c
2
+ ca + 4a
2

a + b + c
3
www.VNMATH.com
5
41. Cho cc s dng a, b, c, chng minh bt ng thc
a(b + c)
a
2
+ bc
+
b(c + a)
b
2
+ ca
+
c(a + b)
c
2
+ ab

1
2

(a + b + c)
_
1
a
+
1
b
+
1
c
_
+ 27
42. Cho cc s khng m a, b, c tha a + b + c = 1, chng minh bt ng thc
a

a + 2b
+
b

b + 2c
+
c

c + 2a

_
3
2
43. Cho cc s khng m a, b, c, tm hng s k tt nht bt ng thc sau ng
a
b + c
+
b
c + a
+
c
a + b

3
2
+
k max{(a b)
2
, (b c)
2
, (c a)
2
}
ab + bc + ca
44. Cho cc s khng m a, b, c, chng minh bt ng thc
_
a
a + b
_
3
+
_
b
b + c
_
3
+
_
c
c + a
_
3

3
8

_
a
2
+ b
2
+ c
2
ab + bc + ca
_
2
45. Cho a, b, c, d l cc s dng tha mn a, b, c 1 v abcd = 1, chng minh rng
1
(a
2
a + 1)
2
+
1
(b
2
b + 1)
2
+
1
(c
2
c + 1)
2
+
1
(d
2
d + 1)
2
4
46. Vi mi s khng m a, b, c, chng minh rng
_
a
2
+ 4bc
b
2
+ c
2
+
_
b
2
+ 4ca
c
2
+ a
2
+
_
c
2
+ 4ab
a
2
+ b
2
2 +

2
47. Cho cc s khng m a, b, c, chng minh bt ng thc
(a b)(13a + 5b)
a
2
+ b
2
+
(b c)(13b + 5c)
b
2
+ c
2
+
(c a)(13c + 5a)
c
2
+ a
2
0
48. Chng minh rng vi mi s dng a, b, c, n, ta c
_
a
2
+ bc
b + c
_
n
+
_
b
2
+ ca
c + a
_
n
+
_
c
2
+ ab
a + b
_
n
a
n
+ b
n
+ c
n
49. Cho cc s khng m a, b, c tha a +b +c = 1. Ty theo gi tr ca n N, hy tm gi tr ln nht
v gi tr nh nht ca biu thc
P(a, b, c) = a(b c)
n
+ b(c a)
n
+ c(a b)
n
50. Cho cc s dng a, b, c tha a + b + c = 3, tm hng s k ln nht sao cho
a
5
+ b
5
+ c
5
3
a
3
+ b
3
+ c
3
3
k
51. Cho cc s khng m a, b, c tha a
2
+ b
2
+ c
2
= 8, chng minh bt ng thc
4(a + b + c 4) abc
www.VNMATH.com
6 CHNG 1. PROBLEMS
52. Cho m, n (3n
2
> m
2
) l cc s thc cho trc v a, b, c l cc s thc tha mn a + b + c =
m, a
2
+ b
2
+ c
2
= n
2
. Tm gi tr ln nht v gi tr nh nht ca biu thc sau
P = a
2
b + b
2
c + c
2
a
53. Tm hng s k nh nht sao cho vi mi a, b, c 0 th

a
3
ka
2
+ (b + c)
2
+

b
3
kb
2
+ (c + a)
2
+

c
3
kc
2
+ (a + b)
2

_
3(a + b + c)
k + 4
54. Chng minh rng nu a, b, c > 0 v a + b + c = 3 th
(ab + bc + ca)
_
a
b
2
+ 9
+
b
c
2
+ 9
+
c
a
2
+ 9
_

9
10
55. Cho cc s dng a, b, c tha a + b + c = 3, chng minh bt ng thc
ab

c
2
+ 3
+
bc

a
2
+ 3
+
ca

b
2
+ 3

3
2
56. Chng minh rng vi mi a, b, c dng th
_
b + c
a
+
_
c + a
b
+
_
a + b
c

16(a + b + c)
3
3(a + b)(b + c)(c + a)
57. Tm hng s k ln nht sao cho bt ng thc sau ng
1
a(1 + bc)
2
+
1
b(1 + ca)
2
+
1
c(1 + ab)
2

k
(1 + ab)(1 + bc)(1 + ca)
+
3
4

k
8
trong a, b, c l cc s dng tha abc = 1.
58. Cho cc s khng m a, b, c, chng minh bt ng thc sau vi k =
ln 3
ln 3ln 2
_
a
2
b
2
+ bc + c
2
_
1/k
+
_
b
2
c
2
+ ca + a
2
_
1/k
+
_
c
2
a
2
+ ab + b
2
_
1/k
2
59. Cho cc s khng m a, b, c chng minh bt ng thc
_
a
2
+ bc
b
2
+ bc + c
2
+
_
b
2
+ ca
c
2
+ ca + a
2
+
_
c
2
+ ab
a
2
+ ab + b
2

6
60. Chng minh rng vi mi x, y [0, 1], ta c
1
x
2
x + 1
+
1
y
2
y + 1
1 +
1
x
2
y
2
xy + 1
61. Cho cc s dng a, b, c, chng minh bt ng thc
_
a
a + b
+
_
b
b + c
+
_
c
c + a

3

2

_
ab + bc + ca
a
2
+ b
2
+ c
2
62. Chng minh rng vi mi a, b, c 0, ta c bt ng thc
a
2
(b + c)
(b
2
+ c
2
)(2a + b + c)
+
b
2
(c + a)
(c
2
+ a
2
)(2b + c + a)
+
c
2
(a + b)
(a
2
+ b
2
)(2c + a + b)

2
3
www.VNMATH.com
7
63. Cho cc s dng a, b, c, chng minh rng vi mi k 2, ta c bt ng thc
a + b + c
3

abc

k
_
a + c
b + c
+
k
_
c + b
a + b
+
k
_
b + a
c + a
64. Cho cc s khng m a, b, c, chng minh bt ng thc
3
_
a
b + c
+
3
_
b
c + a
+
3
_
c
a + b
2

abc
(a + b)(b + c)(c + a)
+ 1
65. Cho cc s thc a, b, c, d tha a
2
+ b
2
+ c
2
+ d
2
= 4, chng minh bt ng thc
9(a + b + c + d) 4abcd + 32
66. Cho cc s khng m a, b, c, chng minh bt ng thc
_
a
2
+ 256bc
b
2
+ c
2
+
_
b
2
+ 256ca
c
2
+ a
2
+
_
c
2
+ 256ab
a
2
+ b
2
12
67. Cho cc s dng x, y, z c tch bng 1, chng minh rng
x
y
4
+ 2
+
y
z
4
+ 2
+
z
x
4
+ 2
1
68. Chng minh rng vi mi s dng a, b, c, d ta c bt ng thc
_
1
a
+
1
b
+
1
c
+
1
d
__
1
a + b
+
1
b + c
+
1
c + d
+
1
d + a
_

16
abcd + 1
69. Cho cc s dng a, b, c, d tha a
2
+ b
2
+ c
2
+ d
2
= 4, chng minh bt ng thc
a + b + c + d
2

3

(abcd + 1)
_
1
a
+
1
b
+
1
c
+
1
d
_
70. Cho cc s dng a
1
, a
2
, . . . , a
n
tha a
1
a
2
a
n
= 1. Khi , vi mi k R, ta c
1
(1 + a
1
)
k
+
1
(1 + a
2
)
k
+ +
1
(1 + a
n
)
k
min
_
1,
n
2
k
_
71. Cho a, b, c l cc s dng, chng minh rng
(a)
a
9
bc
+
b
9
ca
+
c
9
ab
+
2
abc
a
5
+ b
5
+ c
5
+ 2
(b)
a
9
bc
+
b
9
ca
+
c
9
ab
+
3
abc
a
4
+ b
4
+ c
4
+ 3
72. Cho x, y, z, t l cc s dng tha xyzt = 1, chng minh rng
1
xy + yz + zx + 1
+
1
yz + zt + ty + 1
+
1
zt + tx + xz + 1
+
1
tx + xy + yt + 1
1
73. Chng minh rng vi mi x, y, z, t > 0 th
(x + y)(x + z)(x + t)(y + z)(y + t)(z + t) 4xyzt(x + y + z + t)
2
www.VNMATH.com
8 CHNG 1. PROBLEMS
74. Chng minh rng vi mi s dng a
1
, a
2
, . . . , a
n
tha a
1
a
2
a
n
= 1 ta c bt ng thc
_
a
2
1
+ 1 +
_
a
2
2
+ 1 + +
_
a
2
n
+ 1

2(a
1
+ a
2
+ + a
n
)
75. Chng minh rng vi mi s dng a, b, c ta c bt ng thc
a +

ab +
3

abc
3

3
_
a
a + b
2

a + b + c
3
76. Cho cc s khng m a, b, c, chng minh bt ng thc
a
3

b
2
bc + c
2
+
b
3

c
2
ca + a
2
+
c
3

a
2
ab + b
2
a
2
+ b
2
+ c
2
77. Chng minh rng vi mi a, b, c khng m
_
a
2
a
2
+ 6ab + 2b
2
+
_
b
2
b
2
+ 6bc + 2c
2
+
_
c
2
c
2
+ 6ca + 2a
2
1
78. Cho cc s khng m a, b, c, chng minh bt ng thc
_
a
b + c
+
_
b
c + a
+
_
c
a + b
+ 3
_
3(ab + bc + ca)
a
2
+ b
2
+ c
2

7

2
2
79. Cho cc s khng m a, b, c, chng minh bt ng thc
a
b + c
+
b
c + a
+
c
a + b
+
16(ab + bc + ca)
a
2
+ b
2
+ c
2
8
80. Cho cc s khng m a, b, c, chng minh bt ng thc
3(a
3
+ b
3
+ c
3
) + 2abc 11
_
a
2
+ b
2
+ c
2
3
_
3/2
81. Cho cc s khng m a, b, c, d tha a
2
+ b
2
+ c
2
+ d
2
= 1, chng minh bt ng thc
a
3
1 bcd
+
b
3
1 cda
+
c
3
1 dab
+
d
3
1 abc

4
7
82. Cho cc s khng m a, b, c, d tha a
3
+ b
3
+ c
3
+ d
3
= 1, chng minh bt ng thc
1
a
3
1 bcd
+
b
3
1 cda
+
c
3
1 dab
+
d
3
1 abc

4
3
83. Cho cc s dng a, b, c, d tha a + b + c + d = 4, chng minh rng
1
ab
+
1
bc
+
1
cd
+
1
da
a
2
+ b
2
+ c
2
+ d
2
84. Cho cc s dng x, y, z, tm hng s k ln nht sao cho
x
y
+
y
z
+
z
x
+ 3k (k + 1)
x + y + z
3

xyz
www.VNMATH.com
9
85. Cho cc s khng m a, b, c, d, chng minh bt ng thc
_
a
a + b + c
+
_
b
b + c + d
+
_
c
c + d + a
+
_
d
d + a + b

4

3
86. Chng minh rng vi mi a, b, c, d [1, 2], ta c
a + b
c + d
+
c + d
a + b

a + c
b + d

3
2
87. Chng minh rng vi mi a, b, c > 0, ta lun c
a
2
b
c(b + c)
+
b
2
c
a(c + a)
+
c
2
a
b(a + b)

3
2

a
2
+ b
2
+ c
2
a + b + c
88. Cho cc s khng m a, b, c, tha a
2
+ b
2
+ c
2
= 3, chng minh rng
1 + 4abc 5 min{a, b, c}
89. Vi mi a, b, c 0 v ab + bc + ca = 1, ta c
1

2a
2
+ 3bc
+
1

2b
2
+ 3ca
+
1

2c
2
+ 3ab

6
3
90. Cho a, b, c l cc s thc khc 0 tha a
2
+ b
2
+ c
2
= (a b)
2
+ (b c)
2
+ (c a)
2
, chng minh bt
ng thc
1.
a
b
+
b
c
+
c
a
5 2.
1
12

a
2
b + b
2
c + c
2
a
(a + b + c)
3

5
36
91. Tm hng s k > 0 nh nht sao cho bt ng thc
_
a + k(b c)
2
+
_
b + k(c a)
2
+
_
c + k(a b)
2

3
ng vi mi a, b, c 0 v a + b + c = 1.
92. Chng minh rng vi mi a, b, c 0 th

a
3
+ abc
(b + c)
3
+

b
3
+ abc
(c + a)
3
+

c
3
+ abc
(a + b)
3

a
b + c
+
b
c + a
+
c
a + b
93. Cho cc s dng a, b, c, chng minh rng
ab
2
c
2
+
bc
2
a
2
+
ca
2
b
2
+ a + b + c
6(a
2
+ b
2
+ c
2
)
a + b + c
94. Tm gi tr ln nht ca biu thc
P = (a b)(b c)(c a)(a + b + c)
vi a, b, c 0 tha a
2
+ b
2
+ c
2
= 1.
95. Vi mi s dng a, b, c, d,
b(a + c)
c(a + b)
+
c(b + d)
d(b + c)
+
d(c + a)
a(c + d)
+
a(d + b)
b(d + a)
4
www.VNMATH.com
10 CHNG 1. PROBLEMS
96. Chng mnh rng vi mi s thc a, b, c th
a
2
bc
a
2
+ 2b
2
+ 3c
2
+
b
2
ca
b
2
+ 2c
2
+ 3a
2
+
c
2
ca
c
2
+ 2a
2
+ 3b
2
0
97. Cho cc s khng m x, y, z, chng minh bt ng thc
x
4
x
4
+ x
2
yz + y
2
z
2
+
y
4
y
4
+ y
2
zx + z
2
x
2
+
z
4
z
4
+ z
2
xy + x
2
y
2
1
98. Cho cc s dng a, b, c tha abc = 1, chng minh rng
1
a
2
a + 1
+
1
b
2
b + 1
+
1
c
2
c + 1
3
99. Chng minh rng vi mi s dng a, b, c,
3a
2
2ab b
2
3a
2
+ 2ab + 3b
2
+
3b
2
2bc c
2
3b
2
+ 2bc + 3c
2
+
3c
2
2ca a
2
3c
2
+ 2ca + 3a
2
0
100. Cho cc s dng a, b, c tha a
4
+ b
4
+ c
4
= 3, chng minh bt ng thc
a
2
b
3
+ 1
+
b
2
c
3
+ 1
+
c
2
a
3
+ 1

3
2
101. Cho cc s dng a, b, c, chng minh bt ng thc
9
2

(a
2
+ b
2
+ c
2
)
3
(a + b + c)
4

a
3
a + b
+
b
3
b + c
+
c
3
c + a
102. Cho cc s dng a, b, c, d tha a + b + c + d = 4, tm hng s k tt nht sao cho
1
a
+
1
b
+
1
c
+
1
d
4 k(a
2
+ b
2
+ c
2
+ d
2
4)
103. Cho cc s dng x, y, z tha xy + yz + zx = 1, chng minh bt ng thc
x(y + z)
2
(1 + yz)
2
+
y(z + x)
2
(1 + zx)
2
+
z(x + y)
2
(1 + xy)
2

3

3
4
104. Cho cc s khng m a, b, c tha a + b + c = 3, chng minh bt ng thc
_
a +
_
b
2
+ c
2
+
_
b +
_
c
2
+ a
2
+
_
c +
_
a
2
+ b
2
3
_

2 + 1
105. Cho a, b, c l di ba cnh ca mt tam gic, chng minh rng
a
3a + b c
+
b
3b + c a
+
c
3c + a b
1
106. Cho cc s dng a, b, c tha a
2
+ b
2
+ c
2
= 3, chng minh bt ng thc
a
ab + 3
+
b
bc + 3
+
c
ca + 3

3
4
www.VNMATH.com
11
107. Cho cc s khng m a, b, c, chng minh bt ng thc

a
2
b
2
+ (c + a)
2
+

b
2
c
2
+ (a + b)
2
+

c
2
a
2
+ (b + c)
2

3

5
108. Cho a, b, c l di ba cnh ca mt tam gic, chng minh bt ng thc
a(a b)
a
2
+ 2bc
+
b(b c)
b
2
+ 2ca
+
c(c a)
c
2
+ 2ab
0
109. Cho cc s dng a, b, c, chng minh
_
a
2
a
2
+ 7ab + b
2
+
_
b
2
b
2
+ 7bc + c
2
+
_
c
2
c
2
+ 7ca + a
2
1
110. Cho cc s khng m a, b, c, chng minh bt ng thc
1

a
2
+ bc
+
1

b
2
+ ca
+
1

c
2
+ ab

2
_
1
a + b
+
1
b + c
+
1
c + a
_
111. Cho a, b, c l di 3 cnh ca 1 tam gic, chng minh rng
3
_
a
b
+
b
c
+
c
a
3
_
2
_
b
a
+
c
b
+
a
c
3
_
112. Chng minh rng nu a, b, c l di 3 cnh ca 1 tam gic th
a
2
b
c
+
b
2
c
a
+
c
2
a
b
a
2
+ b
2
+ c
2
113. Cho cc s khng m a, b, c chng minh bt ng thc
a
2
b
2
+
b
2
c
2
+
c
2
a
2
+
9(ab + bc + ca)
a
2
+ b
2
+ c
2
12
114. Cho cc s khng m a, b, c, chng minh bt ng thc
a
b
+
b
c
+
c
a
3
_
a
2
+ b
2
+ c
2
ab + bc + ca
_
2/3
115. Cho cc s khng m a, b, c, chng minh bt ng thc
a
b
+
b
c
+
c
a
2
3

9(a
3
+ b
3
+ c
3
)
(a + b)(b + c)(c + a)
116. Cho cc s khng m x, y, z tha x + y
2
+ z
2
= 1, chng minh bt ng thc
x
3
x
2
+ xy + y
2
+
y
3
y
2
+ yz + z
2
+
z
3
z
2
+ zx + x
2

1
2
117. Cho a, b, c l di 3 cnh ca mt tam gic, chng minh bt ng thc
a
2
+ b
2
a
2
+ c
2
+
b
2
+ c
2
b
2
+ a
2
+
c
2
+ a
2
c
2
+ b
2

a + b
a + c
+
b + c
b + a
+
c + a
c + b
www.VNMATH.com
12 CHNG 1. PROBLEMS
118. Cho a, b, c l di 3 cnh ca mt tam gic, chng minh rng
3(a
3
b + b
3
c + c
3
a) (a
2
+ b
2
+ c
2
)(ab + bc + ca)
119. Cho cc s thc a, b, c, chng minh bt ng thc
15a
2
b
2
c
2
+ 12(a
4
+ b
4
+ c
4
)(a
2
+ b
2
+ c
2
) 11(a
6
+ b
6
+ c
6
) + 30abc(a
3
+ b
3
+ c
3
)
120. Cho cc s khng m a, b, c, d tha a + b + c + d = 3, chng minh bt ng thc
ab(b + c) + bc(c + d) + cd(d + a) + da(a + b) 4
121. Cho a, b, c l cc s khn m tha a
2
+ b
2
+ c
2
= 1, chng minh rng
_
1
_
a + b
2
_
2
__
1
_
b + c
2
_
2
__
1
_
c + a
2
_
2
_

8
27
122. Cho cc s khng m a, b, c, d, chng minh bt ng thc
ab
a + b
+
bc
b + c
+
cd
c + d
+
da
d + a

_
(a + c)(b + d)
123. Chng minh rng vi mi s dng a, b, c ta c bt ng thc
a
b
+
b
c
+
c
a

_
a
2
+ c
2
b
2
+ c
2
+
_
c
2
+ b
2
a
2
+ b
2
+
_
b
2
+ a
2
c
2
+ a
2
124. Cho cc s khng m a, b, c tha a + b + c = 5, chng minh bt ng thc
16(a
3
b + b
3
c + c
3
a) + 640 11(ab
3
+ bc
3
+ ca
3
)
125. Cho cc s dng a, b, c, chng minh bt ng thc
1
a + b + c

_
1
a + b
+
1
b + c
+
1
c + a
_

1
ab + bc + ca
+
1
2(a
2
+ b
2
+ c
2
)
126. Chng minh rng vi mi s khng m a, b, c, d ta c
1
a
3
+ b
3
+
1
a
3
+ c
3
+
1
a
3
+ d
3
+
1
b
3
+ c
3
+
1
b
3
+ d
3
+
1
c
3
+ d
3

243
2(a + b + c + d)
3
127. Chng minh rng vi mi s khng m a, b, c, d ta c
1
a
2
+ b
2
+ c
2
+
1
b
2
+ c
2
+ d
2
+
1
c
2
+ d
2
+ a
2
+
1
d
2
+ a
2
+ b
2

12
(a + b + c + d)
2
128. Cho cc s dng a, b, c, chng minh bt ng thc
_
a(b + c)
a
2
+ bc
+
_
b(c + a)
b
2
+ ca
+
_
c(a + b)
c
2
+ ab

a +

b +

c
_
_
1

a
+
1

b
+
1

c
_
129. Chng minh rng vi mi s dng a, b, c th
a
2
bc

a
2
+ 2b
2
+ 3c
2
+
b
2
ca

b
2
+ 2c
2
+ 3a
2
+
c
2
ab

c
2
+ 2a
2
+ 3b
2
0
www.VNMATH.com
13
130. Cho cc s dng a, b, c tha a + b + c = 1, chng minh bt ng thc
_
1
a
2
_
2
+
_
1
b
2
_
2
+
_
1
c
2
_
2

8(a
2
+ b
2
+ c
2
)
2
(1 a)(1 b)(1 c)
131. Cho cc s khng m a, b, c, d tha a + b + c + d = 1, chng minh bt ng thc

a
4
b
4
+ c
4
d
4
2a
2
c
2
+ 2b
2
d
2
+ 4ab
2
c + 4cd
2
a 4bc
2
d 4da
2
b

1
132. Cho cc s dng a, b, c, chng minh bt ng thc
ab(a
2
+ bc)
b + c
+
bc(b
2
+ ca)
c + a
+
ca(c
2
+ ab)
a + b

_
3abc(ab
2
+ bc
2
+ ca
2
)
133. Tm hng s a nh nht sao cho bt ng thc sau
_
x + y + z
3
_
a
_
xy + yz + zx
3
_
3a
2

(x + y)(y + z)(z + x)
8
ng vi mi s thc dng x, y, z.
134. Cho cc s khng m a, b, c tha a
2
+ b
2
+ c
2
= 1, chng minh bt ng thc
1
a

1 + bc
+
b

1 + ca
+
c

1 + ab

3
2
135. Cho a, b, c l cc s khng m, chng minh bt ng thc
_
a(b + c)
b
2
+ c
2
+
_
b(c + a)
c
2
+ a
2
+
_
c(a + b)
a
2
+ b
2

_
2 + 2

_
1 + 4

abc(a + b)(b + c)(c + a)


(a
2
+ b
2
)(b
2
+ c
2
)(c
2
+ a
2
)
136. Cho a, b, c l cc s thc dng, chng minh rng
a
2
ab + b
2
a + b
+
b
2
bc + c
2
b + c
+
c
2
ca + a
2
c + a

3
2

a
3
+ b
3
+ c
3
a
2
+ b
2
+ c
2
137. Chng minh rng vi mi s dng a, b, c > 0 tha abc = 1, ta c bt ng thc
1
(1 + a)
2
+
1
(1 + b)
2
+
1
(1 + c)
2
+
1
a + b + c + 1
1
138. Cho cc s dng x, y, x tha x + y + z = 1. Chng minh rng
_
x
2
+ xyz +
_
y
2
+ xyz +
_
z
2
+ xyz
_
x
2
+ y
2
+ z
2
+ xy + yz + zx + 2
_
3xyz
139. Chng minh rng nu x, y, z l cc s khng m tha x
2
+ y
2
+ z
2
= 1 th
9
3

18

1
3
_
1
_
x+y
2
_
2
+
1
3
_
1
_
y+z
2
_
2
+
1
3
_
1
_
z+x
2
_
2
1 +
4
3

6
140. Chng minh rng vi mi s khng m a, b, c tha a + b + c = 1,
a

4a + 5b
2
+
b

4b + 5c
2
+
c

4c + 5a
2

17
www.VNMATH.com
14 CHNG 1. PROBLEMS
141. Tm hng s k = k(n) ln nht sao cho bt ng thc sau ng vi mi s thc a
1
, a
2
, . . . , a
n
a
2
1
+ a
2
2
+ + a
2
n
k(n)(a
1
a
2
+ a
2
a
3
+ + a
n1
a
n
)
142. Vi mi s dng a, b, c, ta c
3
_
a
2
+ bc
b + c
+
3
_
b
2
+ ca
c + a
+
3
_
c
2
+ ab
a + b

3
_
9(a + b + c)
143. Cho cc s khng m a, b, c, chng minh bt ng thc
_
a +
b
2
c
_
2
+
_
b +
c
2
a
_
2
+
_
c +
a
2
b
_
2

12(a
3
+ b
3
+ c
3
)
a + b + c
144. Cho cc s khng m a, b, c tha ab + bc + ca = 1, chng minh bt ng thc
1

a + bc
+
1

b + ca
+
1

c + ab
2

2
145. Cho cc s dng a, b, c tha a + b + c =
1
a
+
1
b
+
1
c
, chng minh
_
a + b
b + 1
+
_
b + c
c + 1
+
_
c + a
a + 1
3
146. Cho a
1
, a
2
, . . . , a
5
l cc s dng tha
a
1
a
2
a
5
= a
1
(1 + a
2
) + a
2
(1 + a
3
) + + a
5
(1 + a
1
) + 2
Tm gi tr nh nht ca biu thc
P =
1
a
1
+
1
a
2
+ +
1
a
5
.
147. Vi mi s dng a, b, c, ta c
a(a + c)
b(b + c)
+
b(b + a)
c(c + a)
+
c(c + b)
a(a + b)

3(a
2
+ b
2
+ c
2
)
ab + bc + ca
148. Chng minh rng vi mi a, b, c dng,
a(b + c)

a
2
+ bc
+
b(c + a)

b
2
+ ca
+
c(a + b)

c
2
+ ab

_
6(a
2
+ b
2
+ c
2
)
149. Cho a, b, c l cc s dng, chng minh rng
3 +
a
b
+
b
c
+
c
a
2

(a + b + c)
_
1
a
+
1
b
+
1
c
_
150. Cho a, b, c l cc s khng m tha mn ab + bc + ca = 1, chng minh
a
2
b
+
b
2
c
+
c
2
a
2(a
2
+ b
2
+ c
2
)

3 2
151. Tm hng s k ln nht sao cho bt ng thc sau ng
a + b + c + kabc k + 3
vi mi s khng m a, b, c tha mn ab + bc + ca + 6abc = 9.
www.VNMATH.com
15
152. Cho cc s khng m a, b, c tha a
2
+ b
2
+ c
2
= 1. Chng minh rng
a
3
b
2
bc + c
2
+
b
3
c
2
ca + a
2
+
c
3
a
2
ab + b
2

2
153. Cho cc s khng m x, y, z tha 6 x + y + z 3, chng minh rng

1 + x +
_
1 + y +

1 + z
_
xy + yz + zx + 15
154. Cho cc s dng x, y, z tha xyz = 1, chng minh bt ng thc
y + z
x
3
+ yz
+
z + x
y
3
+ zx
+
x + y
z
3
+ xy

1
x
2
+
1
y
2
+
1
z
2
155. Cho cc s dng a, b, c, chng minh bt ng thc
3
9

9a(a + b)
2(a + b + c)
2
+
3

6bc
(a + b)(a + b + c)
4
156. Cho cc s khng m a, b, c, chng minh bt ng thc
1
(a + 2b)
2
+
1
(b + 2c)
2
+
1
(c + 2a)
2

1
ab + bc + ca
157. Cho cc s khng m a, b, c, chng minh bt ng thc
a
2
a
2
+ ab + b
2
+
b
2
b
2
+ bc + c
2
+
c
2
c
2
+ ca + a
2
+
ab + bc + ca
a
2
+ b
2
+ c
2
2
158. Cho cc s khng m x, y, z tha x + y + z = 3, chng minh bt ng thc
x
2
y + y
2
z +
3
2
xyz 4
159. Cho cc s khng m a, b, c, chng minh bt ng thc
1
a
2
+ bc
+
1
b
2
+ ca
+
1
c
2
+ ab

3(a + b + c)
2
2(a
2
+ b
2
+ c
2
)(ab + bc + ca)
160. Cho cc s khng m a, b, c, chng minh bt ng thc
4
3
(ab
2
+ bc
2
+ ca
2
) + a
2
+ b
2
+ c
2
+ 2 3(ab + bc + ca)
161. Cho cc s khng m a, b, c, chng minh bt ng thc
1

4a
2
+ bc
+
1

4b
2
+ ca
+
1

4c
2
+ ab

4
a + b + c
162. Cho cc s thc a, b, c, chng minh bt ng thc
1 + a
2
b
2
(a b)
2
+
1 + b
2
c
2
(b c)
2
+
1 + c
2
a
2
(c a)
2

3
2
163. Cho cc s khng m a, b, c, chng minh rng
a
2
b
+
b
2
c
+
c
2
a
3
_
a
4
+ b
4
+ c
4
a
2
+ b
2
+ c
2
www.VNMATH.com
16 CHNG 1. PROBLEMS
164. Cho cc s dng a, b, c, chng minh rng
_
a
b
+
b
c
+
c
a
2 +
8abc
(a + b)(b + c)(c + a)
2
165. Cho cc s thc a, b, c, chng minh bt ng thc
_
a(b + c)
(a + b)(a + c)
_
2
+
_
b(c + a)
(b + c)(b + a)
_
2
+
_
c(a + b)
(c + a)(c + b)
_
2

1
2
166. Cho cc s khng m x, y, z tha x + y + z = 1. Chng minh bt ng thc
_
x + y
2
+
_
y + z
2
+
_
z + x
2

11
5
167. Cho cc s khng m a, b, c, d tha a + b + c + d = 4, tm hng s k >
64
27
nh nht bt ng
thc sau ng
1
k abc
+
1
k bcd
+
1
k cda
+
1
k dab

4
k 1
168. Cho cc s khng m a, b, c, chng minh bt ng thc
3(a + b + c) 2
_
_
a
2
+ bc +
_
b
2
+ ca +
_
c
2
+ ab
_
169. Cho dy dng {x
n
} tha
k

i=1
x
i

k vi mi k = 1, 2, . . . , n, chng minh bt ng thc


x
2
1
+ x
2
2
+ + x
2
n

1
4
_
1 +
1
2
+
1
3
+ +
1
n
_
170. Cho cc s khng m a, b, c tha 6 a + b + c 3, chng minh bt ng thc

a + 1 +

b + 1 +

c + 1

15 + ab + bc + ca
www.VNMATH.com
Chng 2
Solution
2.1. Li gii cc bi ton
1 Cho x, y, z l cc s dng tha xy + yz + zx = 1, chng minh
1
_
1 + (2x y)
2
+
1
_
1 + (2y z)
2
+
1
_
1 + (2z x)
2

3
2
Li gii. t a = 2x y, b = 2y z, c = 2z x, do a +b +c = x +y +z > 0 v t xy +yz +zx = 1,
ta c
14(a
2
+ b
2
+ c
2
) + 35(ab + bc + ca) = 49
Li c 3(14(a
2
+ b
2
+ c
2
) + 35(ab + bc + ca)) 49(a + b + c)
2
, nn
a + b + c

3
Ta s chng minh vi mi s thc a, b, c tha mn a + b + c

3, th
P(a, b, c) =
1

a
2
+ 1
+
1

b
2
+ 1
+
1

c
2
+ 1

3

3
2
Nu c 0, thay c bi c

= c, th ta cng c a+b+c

3, v gi tr ca biu thc P vn khng i,


do , khng mt tnh tng qut, ta c th gi s a, b, c > 0, khi , t a = ka
1
, b = kb
1
, c = kc
1
vi k 1, a
1
, b
1
, c
1
> 0 sao cho a
1
+ b
1
+ c
1
=

3, th
P(a, b, c) =

cyc
1
_
k
2
a
2
1
+ 1

cyc
1
_
a
2
1
+ 1
= P(a
1
, b
1
, c
1
)
Nh vy, ta c th gi s a, b, c > 0 v a + b + c =

3. Xt hm s f(x) =
1

x
2
+1
, ta c
f

(x) =
2x
2
1
(x
2
+ 1)
5/2
T y, ta c th d dng kim tra c f lm trn
_
0,
1

2
_
v li trn
_
1

2
,

3
_
.
Khng mt tnh tng qut, gi s a b c > 0, t y suy ra c
1

3
, Xt 2 trng hp
Trng hp 1. b
1

2
, s dng bt ng thc Jensen
f(b) + f(c) 2f
_
b + c
2
_
=
2
_
_
b+c
2
_
2
+ 1
=
4
_
_
3 a
_
2
+ 4
Ta cn chng minh
4
_
_
3 a
_
2
+ 4
+
1

a
2
+ 1

3

3
2
(2.1)
17
www.VNMATH.com
18 CHNG 2. SOLUTION
Tht vy, t a =
t

3
th 3 t 1 v ta cn chng minh
4

t
2
6t + 21
+
1

t
2
+ 3

3
2
Hay
16
t
2
6t + 21
+
1
t
2
+ 3
+
8
_
(t
2
+ 3)(t
2
6t + 21)
(t
2
+ 3)(t
2
6t + 21)

9
4
S dng bt ng thc AMGM, ta c
_
t
2
+ 3
t
2
+ 7
4
,
_
t
2
6t + 21
t
2
6t + 37
8
Nh vy, ta ch cn chng minh
16
t
2
6t + 21
+
1
t
2
+ 3
+
(t
2
+ 7)(t
2
6t + 37)
4(t
2
+ 3)(t
2
6t + 21)

9
4
Hay
(t 1)
2
(t 2)
2
0
Bt ng thc ny hin nhin ng.
Trng hp 2. b
1

2
, ta c
f(a) + f(b) f
_
1

2
_
+ f
_
a + b
1

2
_
S dng bt ng thc Jensen,
f
_
1

2
_
+ f(c) 2f
_
c +
1

2
2
_
= 2f
_
_

3
_
a + b
1

2
_
2
_
_
Nh vy, ta cn chng minh
2f
_
_

3
_
a + b
1

2
_
2
_
_
+ f
_
a + b
1

2
_

3
2
Bt ng thc ny ng theo (2.1). Bt ng thc c chng minh xong. ng thc xy ra khi
v ch khi x = y = z =
1

3
.
Nhn xt. Bt ng thc trn vn ng vi mi x, y, z R tha mn xy + yz + zx = 1.

2 Cho cc s dng a, b, c tha abc = 1, chng minh rng


a

b + c
b + c + 1
+
b

c + a
c + a + 1
+
c

a + b
a + b + 1

2
Li gii. S dng bt ng thc Holder, ta c
_

cyc
a

b + c
b + c + 1
_
2
_

cyc
a(b + c + 1)
2
b + c
_
(a + b + c)
3
www.VNMATH.com
19
Do , ta cn chng minh
(a + b + c)
3
2

cyc
a(b + c + 1)
2
b + c
hay

cyc
a
3
+ 3

cyc
a
b
+ 3

cyc
b
a
+ 6 4

cyc
ab + 4

cyc
a + 2

cyc
a
b + c
S dng bt ng thc AMGM, ta li c

cyc
a
b

cyc
ab,

cyc
b
a

cyc
ab, 2

cyc
a
b + c

1
2

cyc
a
b
+
1
2

cyc
b
a
Do ,
V T V P

cyc
a
3
+
5
2

cyc
a
b
+
5
2

cyc
b
a
4

cyc
ab 4

cyc
a + 6

cyc
a
3
+

cyc
ab 4

cyc
a + 6 =

cyc
_
a
3
4a +
1
a
+ 2
_
Xt hm s f(x) = x
3
4x +
1
x
+ 2 + 2 ln x vi x > 0, ta c
f

(x) = (x 1)
_
3x + 3 +
1
x
2

1
x
_
Nu x 1 th
1
x
2

1
x
, nu x 1 th 1
1
x
, do
f

(x) = 0 x = 1
T y, ta d dng kim tra c
f(x) f(1) = 0 x > 0
Hay
x
3
4x +
1
x
+ 2 2 ln x x > 0
Vy

cyc
_
a
3
4a +
1
a
+ 2
_
2

cyc
ln a = 0
Bt ng thc c chng minh xong. ng thc xy ra khi v ch khi a = b = c = 1.

3 Vi mi s khng m a, b, c, ta c
_
a
4a + 4b + c
+
_
b
4b + 4c + a
+
_
c
4c + 4a + b
1
Li gii. Cch 1. S dng bt ng thc Cauchy Schwarz, ta c

cyc
_
a
4a + 4b + c

cyc
a
4a + 4b + c
Khng mt tnh tng qut, gi s a +b +c = 3 v b l s hng nm gia a v c, ta cn chng minh

cyc
a
a + b + 1
1
www.VNMATH.com
20 CHNG 2. SOLUTION
hay
a
2
b + b
2
c + c
2
a + abc 4
V b l s hng nm gia a v c nn
c(b a)(b c) 0
Suy ra
b
2
c + c
2
a abc + bc
2
Do
a
2
b + b
2
c + c
2
a + abc b(a + c)
2

1
2
_
2b + (a + c) + (a + c)
3
_
3
= 4
Bt ng thc c chng minh xong. ng thc xy ra khi v ch khi a = b = c.
Cch 2. S dng bt ng thc Cauchy Schwarz, ta c
_

cyc
_
a
4a + 4b + c
_
2
=
_

cyc
_
a
(4a + 4b + c)(4a + b + 4c)

4a + b + 4c
_
2

cyc
a
(4a + 4b + c)(4a + b + 4c)
__

cyc
(4a + b + 4c)
_
=
9(a + b + c)(a
2
+ b
2
+ c
2
+ 8(ab + bc + ca))
(4a + 4b + c)(4b + 4c + a)(4c + 4a + b)
Ta cn chng minh
9(a + b + c)(a
2
+ b
2
+ c
2
+ 8(ab + bc + ca)) (4a + 4b + c)(4b + 4c + a)(4c + 4a + b)
Hay
7

cyc
a
3
+ 3

cyc
ab(a + b) 39abc
Theo bt ng thc AMGM th

cyc
a
3
3abc,

cyc
ab(a + b) 6abc
Do ta c pcm.

4 Cho cc s dng a, b, c, chng minh


1
a
2
+ bc
+
1
b
2
+ ca
+
1
c
2
+ ab

a + b + c
ab + bc + ca
_
1
a + b
+
1
b + c
+
1
c + a
_
Li gii. Ta c bt ng thc cn chng minh tng ng vi

cyc
ab + bc + ca
a
2
+ bc

cyc
a + b + c
b + c
Hay

cyc
a(a
2
b
2
c
2
+ ab + ac bc)
(b + c)(a
2
+ bc)
0

cyc
a(a + 2b + c)(a b) + a(a + b + 2c)(a c)
(b + c)(a
2
+ bc)
0
www.VNMATH.com
21

cyc
(a b)
_
a(a + 2b + c)
(b + c)(a
2
+ bc)

b(2a + b + c)
(a + c)(b
2
+ ca)
_
0

cyc
z(a
2
b
2
)(a b) 0
Vi
x = (a(b + c)(b
2
+ c
2
) + 2a
2
(b
2
+ c
2
) + 3a
2
bc + a
3
(b + c) b
2
c
2
)(a
2
+ bc)
y = (b(c + a)(c
2
+ a
2
) + 2b
2
(c
2
+ a
2
) + 2b
2
ca + b
3
(c + a) c
2
a
2
)(b
2
+ ca)
z = (c(a + b)(a
2
+ b
2
) + 2c
2
(a
2
+ b
2
) + 2c
2
ab + c
3
(a + b) a
2
b
2
)(c
2
+ ab)
Khng mt tnh tng qut, gi s a b c > 0, khi d thy x, y 0. Li c
y + z b(c + a)(c
2
+ a
2
)(b
2
+ ca) a
2
b
2
(c
2
+ ab)
a
3
b(b
2
+ ca) a
2
b
2
(c
2
+ ab) = a
2
bc(a
2
bc) 0
Ch rng a b c > 0 nn (c
2
a
2
)(c a) (a
2
b
2
)(a b). T y, ta c pcm. ng thc
xy ra khi v ch khi a = b = c hoc a = t > 0, b = c 0 v cc hon v.
Cch 2. Ta c
2

cyc
1
(b + c)
2

cyc
a + b + c
ab + bc + ca

1
b + c
=

cyc
1
b + c
_
2
b + c

a + b + c
ab + bc + ca
_
=

cyc
b(a b) + c(a c)
(b + c)
2
(ab + bc + ca)
=

cyc
a b
ab + bc + ca
_
b
(b + c)
2

a
(c + a)
2
_
=
1
ab + bc + ca

cyc
(ab c
2
)(a b)
2
(a + c)
2
(b + c)
2
Ch rng
2

cyc
1
(a + c)
2

cyc
1
a
2
+ bc
=

cyc
_
1
(a + c)
2
+
1
(b + c)
2

1
c
2
+ ab
_
=

cyc
ab(a b)
2
+ (c
2
ab)
2
(a + c)
2
(b + c)
2
(c
2
+ ab)
Do bt ng thc tng ng
0

cyc
(a b)
2
(a + c)
2
(b + c)
2
_
ab
c
2
+ ab

ab c
2
ab + bc + ca
_
+

cyc
(c
2
ab)
2
(a + c)
2
(b + c)
2
(c
2
+ ab)
=

cyc
c(c
3
+ a
2
b + b
2
a)(a b)
2
(ab + bc + ca)(a + c)
2
(b + c)
2
(c
2
+ ab)
+

cyc
(c
2
ab)
2
(a + c)
2
(b + c)
2
(c
2
+ ab)
Bt ng thc ny hin nhin ng. Vy ta c pcm.
Nhn xt. T bt ng thc ny, ta c
1
a
2
+ bc
+
1
b
2
+ ca
+
1
c
2
+ ab

3
2

(a + b + c)
2
(ab + bc + ca)
2

5 Chng minh rng vi mi s dng a, b, c ta lun c


a
3
2a
2
ab + 2b
2
+
b
3
2b
2
bc + 2c
2
+
c
3
2c
2
ca + 2a
2

a + b + c
3
www.VNMATH.com
22 CHNG 2. SOLUTION
Li gii. Ta c bt ng thc cn chng minh tng ng vi

cyc
S
c
(a b)
2
0
trong
S
a
=
2c b
2b
2
bc + 2c
2
, S
b
=
2a c
2c
2
ca + 2a
2
, S
c
=
2b a
2a
2
ab + 2b
2
Xt 2 trng hp
Trng hp 1. a b c > 0, khi , d thy S
b
0. Ta s chng minh
S
b
+ 2S
c
0 (1)
a
2
S
b
+ 2b
2
S
a
0 (2)
Tht vy, ta c
(1) 6a
2
b + 4ab
2
3abc + 8bc
2
4ac
2
2b
2
c 0 (ng do a b c)
(2) f(a) =
a
2
(2a c)
2c
2
ca + 2a
2
+
2b
2
(2c b)
2b
2
bc + 2c
2
0
Li c
f

(a) =
a(4a
3
4a
2
c + 13ac
2
4c
3
)
(2c
2
ca + 2a
2
)
2
0
Do , f(a) l hm ng bin. Suy ra,
f(a) f(b) =
3b
2
c
2b
2
bc + 2c
2
0
Cc bt ng thc (1) v (2) c chng minh.
T cc bt ng thc ny v vi ch rng a b c > 0 nn (ac)
2
max
_
a
2
b
2
(b c)
2
, (a b)
2
_
,
ta c
2

cyc
S
c
(a b)
2
=
_
S
b
(c a)
2
+ 2S
a
(b c)
2
_
+
_
S
b
(c a)
2
+ 2S
c
(a b)
2
_

(b c)
2
b
2
(a
2
S
b
+ 2b
2
S
a
) + (a b)
2
(S
b
+ 2S
c
) 0
Trng hp 2. c b a > 0, d thy S
c
, S
a
0. Nu 2a c th bt ng thc cn chng minh
hin nhin ng. Xt trng hp ngc li c 2a, tc l S
b
0. Xt 2 trng hp nh
Trng hp 2.1. 2b c + a, ta c
S
b
(c a)
2
+ S
c
(a b)
2
0 (3)
m(b) =
(a b)
2
(2b a)
2a
2
ab + 2b
2
+
(c a)
2
(2a c)
2c
2
ca + 2a
2
0
m

(b) =
(b a)(4b
3
+ 9a
2
b 7a
3
)
(2a
2
ab + 2b
2
)
2
0
Do , m(b) l hm ng bin. Suy ra,
m(b) m
_
a + c
2
_
=
a(a c)
2
(16a
2
2ac + c
2
)
2(4a
2
+ ac + c
2
)(2a
2
ac + 2c
2
)
0
www.VNMATH.com
23
Vy (3) ng. Do ,

cyc
S
c
(a b)
2
S
b
(c a)
2
+ S
c
(a b)
2
0
Trng hp 2.2. c + a 2b.
Trng hp 2.2.1. 2b a 4a, ta s chng minh
S
c
+ 3S
b
0 (4)
S
a
+
3
2
S
b
0 (5)
Tht vy
(4) g(c) =
2b a
2a
2
ab + 2b
2
+
3(2a c)
2c
2
ca + 2a
2
0
Ta c
g

(c) =
6c(c 4a)
(2c
2
ca + 2a
2
)
2
0
Do , g(c) l hm ng bin. Suy ra,
g(c) g(2b a) =
4b
3
4ab
2
a
2
b + 13a
3
(2a
2
ab + 2b
2
)(5a
2
10ab + 8b
2
)
0
(5) h(a) =
4c 2b
2b
2
bc + 2c
2
+
6a 3c
2c
2
ca + 2a
2
0
h

(a) =
3(3c
2
+ 4ca 4a
2
)
(2c
2
ca + 2a
2
)
2
0
Do , h(a) l hm ng bin. Suy ra,
++, Nu c 2b th
h(a) h(0) =
(c 2b)(2c + 3b)
2c(2c
2
bc + 2b
2
)
0
++, Nu 2b c th
h(a) h(2b c) =
(2b c)(4b
2
+ 13bc 2c
2
)
(2b
2
bc + 2c
2
)(8b
2
10bc + 5c
2
)
0
Tm li, ta lun c h(a) 0.
T (4) v (5) vi ch rng (c a)
2
3(b a)
2
+
3
2
(b c)
2
, ta c

cyc
S
c
(a b)
2
(S
c
+ 3S
b
)(a b)
2
+
_
S
a
+
3
2
S
b
_
(b c)
2
0
Trng hp 2.2.2. 2b a 4a a
2
5
b, ta c
S
a
+ S
b
+ S
c
0 (6)
S
a
S
b
+ S
b
S
c
+ S
c
S
a
0 (7)
(6) hin nhin ng v theo (5), ta c
S
a
+ S
b
+ S
c
= S
a
+
_
3
2
S
b
+ S
c
_

1
2
S
b
0
www.VNMATH.com
24 CHNG 2. SOLUTION
By gi ta s chng minh (7), ta c
(7) k(c) = 4(ab
3
+ bc
3
+ ca
3
) + 7abc(a + b + c) 2(a
3
b + b
3
c + c
3
a)
6(a
2
b
2
+ b
2
c
2
+ c
2
a
2
) 0
k

(c) = 12bc
2
+ 4a
3
+ 14abc + 7ab(a + b) 2b
3
6ac
2
12c(a
2
+ b
2
)
k

(c) = 24bc 12ac + 14ab 12a


2
12b
2
24b
2
12ab + 14ab 12a
2
12b
2
= 12b
2
+ 2ab 12a
2
0
Do , k

(c) l hm ng bin. Suy ra,


k

(c) k

(b) = 4a
3
5a
2
b + 15ab
2
2b
3
0 (do a
2
5
b)
Suy ra, k(c) l hm ng bin. Do ,
k(c) k(b) = b(2a
3
5a
2
b + 16ab
2
4b
3
) 0 (do a
2
5
b)
T y, ta c pcm. ng thc xy ra khi v ch khi a = b = c.

6 Cho cc s khng m a, b, c tha a + b + c = 1. Chng minh bt ng thc


_
a +
(b c)
2
4
+
_
b +
(c a)
2
4
+
_
c +
(a b)
2
4

3 +
_
1

3
2
_
(|a b| +|b c| +|c a|)
Li gii. Khng mt tnh tng qut, gi s a b c 0. t a + b = 2t, a b = 2m, k =
1
4
th do gi
thit, ta c t m t c 0. Khi , bt ng thc cn chng minh tr thnh
f(m) =
_
t + m + k(m + c t)
2
+
_
t m + k(m + t c)
2
+
_
c + 4km
2

_
2

3
_
(t + mc)

3
Ta c
f

(m) =
4k(2t c) 1
4 (t + m + k(m + c t)
2
)
3/2
+
4k(2t c) 1
4 (t m + k(m + t c)
2
)
3/2
+
4kc
(c + 4km
2
)
3/2
=
c
2
_
a +
1
4
(b c)
2
_
3/2

c
2
_
b +
1
4
(c a)
2
_
3/2
+
c
_
c +
1
4
(a b)
2
_
3/2
Ch rng a b c 0 nn
a +
1
4
(b c)
2
c
1
4
(a b)
2
=
3
4
(a c)(b + 1) 0
b +
1
4
(c a)
2
c
1
4
(a b)
2
=
3
4
(b c)(a + 1) 0
Suy ra
_
a +
1
4
(b c)
2
_
3/2

_
c +
1
4
(a b)
2
_
3/2
0
_
b +
1
4
(c a)
2
_
3/2

_
c +
1
4
(a b)
2
_
3/2
0
www.VNMATH.com
25
Do ,
f

(m)
c
2
_
c +
1
4
(a b)
2
_
3/2

c
2
_
c +
1
4
(a b)
2
_
3/2
+
c
_
c +
1
4
(a b)
2
_
3/2
= 0
Suy ra, f(m) l hm li. Do ,
f(m) max {f(0), f(t c)}
Nh vy, ta ch cn chng minh
max {f(0), f(t c)}

3
iu ny c ngha l ta ch cn chng minh bt ng thc cho trong trng hp 3 s a, b, c c
2 s bng nhau, khng mt tnh tng qut, gi s b = c. Ta cn chng minh

a + 2
_
b +
(a b)
2
4

3 +
_
2

3
_
|a b|
Hay

_
3a 1
2
_
2
+ 2(1 a)

3 +
_
1

3
2
_
|3a 1|

a
t t =

3a th ta c t

3, ta cn chng minh
_
3t
4
14t
2
+ 27 6 2t +
_
2

3 2
_
|t
2
1|
Xt 2 trng hp
Trng hp 1. t 1, ta c bt ng thc tng ng
_
3t
4
14t
2
+ 27 6 2t +
_
2

3 2
_
(t
2
1)
Hay
2(t 1)
_
t

3
___
6

3 9
_
t
2
+
_
3 +

3
_
t + 18 11

3
_
0
Bt ng thc ny hin nhin ng do

3 t 1.
Trng hp 2. t 1, bt ng thc tr thnh
_
3t
4
14t
2
+ 27 6 2t
_
2

3 2
_
(t
2
1)
Hay
2(t 1)
__
6

3 9
_
t
3
+
_
2

3 3
_
t
2
+
_
2

3 9
_
t + 6

3 3
_
0
Bt ng thc ny cng ng do 1 t 0. Bi ton c gii quyt hon ton. ng thc xy ra
khi v ch khi a = b = c =
1
3
hoc a = 1, b = c = 0 v cc hon v.
Nhn xt. Ta c 1 kt qu "yu" hn nhng kh "p" l
_
a +
(b c)
2
4
+
_
b +
(c a)
2
4
+
_
c +
(a b)
2
4
2
vi mi a, b, c 0, a + b + c = 1.

7 Cho cc s dng a, b, c tha a + b + c = 3, chng minh bt ng thc


a
3/2
b + b
3/2
c + c
3/2
a 3
www.VNMATH.com
26 CHNG 2. SOLUTION
Li gii. S dng bt ng thc Cauchy Schwarz, ta c
a
3/2
b + b
3/2
c + c
3/2
a
_
(ab + bc + ca)(a
2
b + b
2
c + c
2
a)
Nh vy, ta ch cn chng minh
(ab + bc + ca)(a
2
b + b
2
c + c
2
a) 9
Hay
(ab + bc + ca)(a + b + c)(a
2
b + b
2
c + c
2
a) 27
Hay
(ab + bc + ca)
_

cyc
a
3
b +

cyc
a
2
b
2
+ 3abc
_
27
Ch rng
1
2

cyc
(a
2
c
2
2ab + bc + ca)
2
0 nn

cyc
a
3
b
1
3
_

cyc
a
2
_
2
Ta cn chng minh
(ab + bc + ca)
_
_
_

cyc
a
2
_
2
+ 3

cyc
a
2
b
2
+ 9abc
_
_
81
t x = ab + bc + ca th theo bt ng thc AMGM v Schur, ta c x 3, 3abc 4x 9, bt
ng thc tr thnh
x((9 2x)
2
+ 3x
2
9abc) 81
Nh vy, ta ch cn chng minh
x((9 2x)
2
+ 3x
2
3(4x 9)) 81
Hay
(x 3)(7x
2
27x + 27) 0
Bt ng thc ny hin nhin ng do 3 x 0. Bt ng thc c chng minh xong. ng
thc xy ra khi v ch khi a = b = c = 1.

8 Chng minh rng vi mi s thc a, b, c, ta c


ab
4a
2
+ b
2
+ 4c
2
+
bc
4b
2
+ c
2
+ 4a
2
+
ca
4c
2
+ a
2
+ 4b
2

1
3
Li gii. D thy trong 3 s a, b, c lun tn ti t nht 2 s cng du, gi s bc 0, nu ab 0, ac 0
th

cyc
ab
4a
2
+ b
2
+ 4c
2

bc
4b
2
+ c
2
+ 4a
2

1
4
<
1
3
Nh vy, ta ch cn xt trng hp cc s a, b, c cng du, v do , ta ch cn xt a, b, c 0 l .
Khng mt tnh tng qut, ta c th gi s a
2
+ b
2
+ c
2
= 3 v b l s hng nm gia a v c, bt
ng thc tr thnh
4

cyc
ab + 4

cyc
ab
3
+ abc

cyc
ab
2
+ a
2
b
2
c
2
16 + 4

cyc
a
2
b
2
www.VNMATH.com
27
V b l s hng nm gia a v c nn
a(b a)(b c) 0
Suy ra
ab
2
+ a
2
c abc + a
2
b
Do

cyc
ab
2
b(a
2
+ c
2
) + abc = 2 (b 1)
2
(b + 2) + abc 2 + abc
S dng kt qu bi ton trc, ta c

cyc
ab
3

1
3
_

cyc
a
2
_
2
= 3
Nh vy, ta ch cn chng minh
4

cyc
ab + abc(2 + abc) + a
2
b
2
c
2
4 + 4

cyc
a
2
b
2
Mt khc, s dng bt ng thc AMGM, ta c
2abc a
2
b
2
c
2
+ 1
Do , ta phi chng minh
P(a, b, c) = 4

cyc
a
2
b
2
4

cyc
ab 3a
2
b
2
c
2
+ 3 0
V y l mt bt ng thc i xng vi a, b, c nn khng mt tnh tng qut, gi s a = max{a, b, c}
suy ra a 1, b
2
+ c
2
2, ta c
P(a, b, c) P
_
a,
_
b
2
+ c
2
2
,
_
b
2
+ c
2
2
_
= (b c)
2
_
2 +
4a
_
2(b
2
+ c
2
) + b + c
+
(3a
2
4)(b + c)
2
4
_
Ta c
2 +
(3a
2
4)(b + c)
2
4
2
(b + c)
2
4
2
b
2
+ c
2
2
> 0
Do
P(a, b, c) P
_
a,
_
b
2
+ c
2
2
,
_
b
2
+ c
2
2
_
Nh vy, ta ch cn chng minh
P(a, t, t) 0
vi a t 0, a
2
+ 2t
2
= 3.
Hay
(a
2
+ 2t
2
)
3
+ 12t
2
(a
2
+ 2t
2
)(2a
2
+ t
2
) 27a
2
t
4
+ 4t(2a + t)(a
2
+ 2t
2
)
2
Hay
(a t)
2
(a
2
(a 3t)
2
+ 4a
2
t
2
+ 16t
4
) 0
Bt ng thc ny hin nhin ng. Vy ta c pcm. ng thc xy ra khi v ch khi a = b = c.

www.VNMATH.com
28 CHNG 2. SOLUTION
9 Cho cc s khng m a, b, c tha a + b + c = 3, chng minh

a
2
+ b
2
(a + 1)(b + 1)
+

b
2
+ c
2
(b + 1)(c + 1)
+

c
2
+ a
2
(c + 1)(a + 1)

3

2
Li gii. Bnh phng 2 v ri nhn 2 v vi (a + 1)(b + 1)(c + 1), ta c bt ng thc tng ng

cyc
(a
2
+ b
2
)(1 + c) + 2

cyc
_
(a
2
+ c
2
)(b
2
+ c
2
)(a + 1)(b + 1)
9
2
(a + 1)(b + 1)(c + 1)
Theo bt ng thc Cauchy Schwarz th

cyc
_
(a
2
+ c
2
)(b
2
+ c
2
)(a + 1)(b + 1)

cyc
(c
2
+ ab)
_
1 +

ab
_
Ta cn chng minh

cyc
(a
2
+ b
2
)(1 + c) + 2

cyc
(c
2
+ ab)
_
1 +

ab
_

9
2
(a + 1)(b + 1)(c + 1)
Hay
8

cyc
a
2
+

cyc
ab + 4

cyc

ab(c
2
+ ab) 36 + 15abc
S dng bt ng thc AMGM v Schur, ta c
4

cyc

ab(c
2
+ ab) 15abc 9abc 12

cyc
ab 27
Nh vy, ta ch cn chng minh
8

cyc
a
2
+

cyc
ab + 12

cyc
ab 27 36
Hay
ab + bc + ca 3
Bt ng thc ny ng theo bt ng thc AMGM. Vy ta c pcm. ng thc xy ra khi v
ch khi a = b = c = 1.

10 Vi mi a b c 0, t
P =
a
b + c
+
b
c + a
+
c
a + b
Q =
2(b + c) a
4a + b + c
+
2(c + a) b
4b + c + a
+
2(a + b) c
4c + a + b
Chng minh rng
1. Nu a + c 2b th P Q.
2. Nu a + c 2b th P Q.
Li gii. Khng mt tnh tng qut, gi s a + b + c = 1.
(1) Bt ng thc cn chng minh tng ng vi

cyc
3a 1
(3a + 1)(1 a)
0
www.VNMATH.com
29
Hay

cyc
z(a b)
2
0
vi x = (1 9a
2
)(1 a), y = (1 9b
2
)(1 b), z = (1 9c
2
)(1 c).
Ch rng a b c, a + c 2b nn
b
1
3
, y, z 0, a c 2(b c) 0, a b b c 0
Do , ta ch cn chng minh
x + 4y + z 0
Hay
F(a, b, c) = 9(a
3
+ c
3
) 9(a
2
+ c
2
) + 36b
3
36b
2
3b + 5 0
Ta c
F(a, b, c) =
(1 3b)(11 + 30b 45b
2
+ 9(a c)
2
)
4
0
(2) Bng bin i tng t, ta c bt ng thc tng ng

cyc
S
c
(a b)
2
0
vi S
a
= (9a
2
1)(1 a), S
b
= (9b
2
1)(1 b), S
c
= (9c
2
1)(1 c).
Do a b c, 2b a + c nn
1
2
b
1
3
, S
a
, S
b
0, a c 2(a b) 0, b c a b 0
Nh vy, ta ch cn chng minh
S
a
+ 4S
b
+ S
c
0
Hay
G(a, b, c) = 9(a
3
+ c
3
) + 9(a
2
+ c
2
) 36b
3
+ 36b
2
+ 3b 5 0
Ta c
G(a, b, c) =
(3b 1)(11 + 30b 45b
2
+ 9(a c)
2
)
4
0
Bi ton c gii quyt hon ton.
ng thc c 2 bt ng thc xy ra khi v ch khi 2b = a + c.

11 Cho cc s khng m a, b, c tha a + b + c = 1, t x = a


2
+ b
2
+ c
2
, chng minh bt ng thc
_
1 + 2a
2
x +
_
1 + 2b
2
x +
_
1 + 2c
2
x

11 9x
Li gii. Bnh phng 2 v ri thu gn, ta c th vit li bt ng thc nh sau

cyc
_
(1 + a
2
b
2
c
2
)(1 + b
2
c
2
a
2
) 8

cyc
ab
S dng bt ng thc GM-HM, ta c
_
(1 + a
2
b
2
c
2
)(1 + b
2
c
2
a
2
)
(1 + a
2
b
2
c
2
)(1 + b
2
c
2
a
2
)
1 c
2
www.VNMATH.com
30 CHNG 2. SOLUTION
Ta cn chng minh

cyc
(1 + a
2
b
2
c
2
)(1 + b
2
c
2
a
2
)
1 c
2
8

cyc
ab
Hay
2

cyc
c(a b)
2
1 + c
0
Bt ng thc ny hin nhin ng, vy ta c pcm. ng thc xy ra khi v ch khi a = b = c =
1
3
hoc a = 1, b = c = 0 v cc hon v.

12 Chng minh rng vi mi a, b, c > 0, ta c


1
a(a + b)
+
1
b(b + c)
+
1
c(c + a)

3
2(abc)
2/3
Li gii. Khng mt tnh tng qut, ta c th gi s abc = 1. Khi , tn ti cc s dng x, y, z sao
cho a =
x
y
, b =
z
x
, c =
y
z
, bt ng thc tr thnh

cyc
y
2
x
2
+ yz

3
2
S dng bt ng thc Cauchy Schwarz, ta c

cyc
y
2
x
2
+ yz

(x
2
+ y
2
+ z
2
)
2
x
2
y
2
+ y
2
z
2
+ z
2
x
2
+ x
3
y + y
3
z + z
3
x
Mt khc, ta li c
(x
2
+ y
2
+ z
2
)
2
3(x
3
y + y
3
z + z
3
x) =
1
2

cyc
(x
2
z
2
2xy + yz + zx)
2
0
(x
2
+ y
2
+ z
2
)
2
3(x
2
y
2
+ y
2
z
2
+ z
2
x
2
) =
1
2

cyc
(x
2
y
2
)
2
0
Nn t y, ta d dng suy ra pcm. ng thc xy ra khi v ch khi a = b = c.

13 Chng minh rng nu a, b, c > 0 th


1
a

a + b
+
1
b

b + c
+
1
c

c + a

3

2abc
Li gii. Tng t bi trn, ta cng a bi ton v chng minh rng vi mi x, y, z > 0 th

cyc
y

y
_
x(x
2
+ yz)

3

2
S dng bt ng thc Cauchy Schwarz, ta c
V T
(x + y + z)
2
_
xy(x
2
+ yz) +
_
yz(y
2
+ zx) +
_
zx(z
2
+ xy)

(x + y + z)
2
_
(xy + yz + zx)(x
2
+ y
2
+ z
2
+ xy + yz + zx)
www.VNMATH.com
31
Mt khc, theo bt ng thc AMGM th
8(xy + yz + zx)(x
2
+ y
2
+ z
2
+ xy + yz + zx)
(x
2
+ y
2
+ z
2
+ 3(xy + yz + zx))
2

16
9
(x + y + z)
2
T y, ta c pcm. ng thc xy ra khi v ch khi a = b = c.

14 Cho cc s dng x, y, z tha x


2
+ y
2
+ z
2
3, chng minh rng
x
5
x
2
x
5
+ y
2
+ z
2
+
y
5
y
2
y
5
+ z
2
+ x
2
+
z
5
z
2
z
5
+ x
2
+ y
2
0
Li gii. Bt ng thc cho c vit li nh sau

cyc
1
x
5
+ y
2
+ z
2

3
x
2
+ y
2
+ z
2
T y, ta suy ra c ch cn xt trng hp x
2
+y
2
+z
2
= 3 l , khi , bt ng thc tng
ng

cyc
1
x
5
x
2
+ 3
1
S dng bt ng thc AMGM, ta c
x
5
=
x
6
x

2x
6
x
2
+ 1
t a = x
2
, b = y
2
, c = z
2
th ta c a + b + c = 3 v ta cn chng minh

cyc
1
2a
3
a+1
a + 3
1
Hay

cyc
a + 1
2a
3
a
2
+ 2a + 3
1
Hay

cyc
(a 1)
2
(2a
2
+ 3a + 3)
2a
3
a
2
+ 2a + 3
0
Khng mt tnh tng qut, gi s a b c, suy ra a 1 c. Xt 2 trng hp
Trng hp 1. b + c 1, suy ra a 2, khi , ta c
2a
2
+ 3a + 3 > 0, 2b
2
+ 3b + 3 > 0, 2c
2
+ 3c + 3 > 0
Nn kt qu bi ton l hin nhin.
Trng hp 2. b + c 1, suy ra a 2, ta c
(2a
3
a
2
+ 2a + 3) 5(a + 1) = 2a
3
a
2
3a 2 = a
3
_
2
1
a

3
a
2

2
a
3
_
a
3
_
2
1
2

3
2
2

2
2
3
_
=
1
2
a
3
> 0
www.VNMATH.com
32 CHNG 2. SOLUTION
Suy ra
a+1
2a
3
a
2
+2a+3

1
5
. Nh vy, ta ch cn chng minh
b + 1
2b
3
b
2
+ 2b + 3
+
c + 1
2c
3
c
2
+ 2c + 3

4
5
iu ny lun ng v vi mi 1 x 0, ta c
x + 1
2x
3
x
2
+ 2x + 3

2
5
Tht vy, bt ng thc tng ng
4x
3
(x + 1)(2x 1)
Nu x
1
2
th ta c ngay pcm, nu x
1
2
th
4x
3
(x + 1)(2x 1) 4x
3
2(2x 1) = 2(2x
3
2x + 1)
2(x
2
2x + 1) = 2(x 1)
2
0
Bt ng thc c chng minh xong. ng thc xy ra khi v ch khi x = y = z = 1.

15 Cho n 3 v a
1
, a
2
, . . . , a
n
l cc s khng m tha a
2
1
+ a
2
2
+ + a
2
n
= 1, chng minh bt ng
thc
1

3
(a
1
+ a
2
+ + a
n
) a
1
a
2
+ a
2
a
3
+ + a
n
a
1
Li gii. t f
n
(a
1
, a
2
, . . . , a
n
) =
1

3
(a
1
+a
2
+ +a
n
) (a
1
a
2
+a
2
a
3
+ +a
n
a
1
). Khng mt tnh
tng qut, gi s a
1
= max{a
1
, a
2
, . . . , a
n
}. Nu a
n

1

3
th
f
n
(a
1
, a
2
, . . . , a
n
) f
n1
_
a
1
, a
2
, . . . , a
n2
,
_
a
2
n1
+ a
2
n
_
=
1

3
_
a
n1
+ a
n

_
a
2
n1
+ a
2
n
_
+ (a
n2
+ a
1
)
_
a
2
n1
+ a
2
n
a
n1
(a
n2
+ a
n
) a
n
a
1

_
1

3
a
n
__
a
n1
+ a
n

_
a
2
n1
+ a
2
n
_
0
Suy ra
f
n
(a
1
, a
2
, . . . , a
n
) f
n1
_
a
1
, a
2
, . . . , a
n2
,
_
a
2
n1
+ a
2
n
_
Nu a
n

1

3
th ta c a
1

1

3
, suy ra a
n1

1

3
, do
f
n
(a
1
, a
2
, . . . , a
n
) f
n1
_
a
1
, a
2
, . . . , a
n3
,
_
a
2
n2
+ a
2
n1
, a
n
_
=
1

3
_
a
n2
+ a
n1

_
a
2
n2
+ a
2
n1
_
+ a
n3
_
_
a
2
n2
+ a
2
n1
a
n2
_
+ a
n
_
_
a
2
n2
+ a
2
n1
a
n1
_
a
n2
a
n1
a
n2
_
1

3
a
n1
_
0
Suy ra
f
n
(a
1
, a
2
, . . . , a
n
) f
n1
_
a
1
, a
2
, . . . , a
n3
,
_
a
2
n2
+ a
2
n1
, a
n
_
T y, ta suy ra c ta ch cn chng minh bt ng thc trong trng hp n = 3 l nhng
trong trng hp ny, bt ng thc l hin nhin nn ta c pcm. ng thc xy ra khi v ch
khi n = 3 v a
1
= a
2
= a
3
=
1

3
.

www.VNMATH.com
33
16 Cho cc s dng a, b, c, chng minh bt ng thc
_
a
b
+
b
c
+
c
a
+
_
ab + bc + ca
a
2
+ b
2
+ c
2

3 + 1
Li gii. Trc ht, ta chng minh kt qu sau vi mi a, b, c > 0
(a + b + c)
2
_
a
b
+
b
c
+
c
a
_
9(a
2
+ b
2
+ c
2
)
Tht vy, bt ng thc tng ng

cyc
S
c
(a b)
2
0
trong
S
a
=
b
c
+
a
b
+
2a
c

5
2
, S
b
=
c
a
+
b
c
+
2b
a

5
2
, S
c
=
a
b
+
c
a
+
2c
b

5
2
Khng mt tnh tng qut, gi s a = max{a, b, c}. Nu c b th ta c
a
b
+
b
c
+
c
a

a
c
+
c
b
+
b
a
nn
khng mt tnh tng qut, ta ch cn xt a b c > 0 l , khi , d thy S
a
0. Ta s chng
minh
S
a
+ 2S
b
0, S
c
+ 2S
b
0, S
b
+ S
c
0
Tht vy, ta c
S
a
+ 2S
b
= 2
_
a
c
+
c
a
_
+
_
a
b
+
4b
a
_
+
3b
c

15
2
4 + 4 + 3
15
2
> 0
S
c
+ 2S
b
= 2
_
b
c
+
c
b
_
+
_
a
b
+
4b
a
_
+
3c
a

15
2
4 + 4
15
2
> 0
S
b
+ S
c
=
_
a
b
+
2b
a
_
+
_
b
2c
+
2c
b
_
+
_
b
2c
+
2c
a
_
5
a
b
+ 2
_
b
a
+
2b
a
+ 2 5
=
_
a
2b
+
2b
a
_
+
_
a
2b
+
_
b
a
+
_
b
a
_
3 2 +
3
3

2
3 > 0
T y, ta c
+, Nu S
b
0 th

cyc
S
c
(a b)
2
(S
a
+ 2S
b
)(b c)
2
+ (S
c
+ 2S
b
)(a b)
2
0
+, Nu S
b
0 th

cyc
S
c
(a b)
2
(S
c
+ S
b
)(a b)
2
0
Bt ng thc trn c chng minh, s dng bt ng thc ny, ta suy ra c, ta ch cn chng
minh
3

a
2
+ b
2
+ c
2
(a + b + c)
2
+
_
ab + bc + ca
a
2
+ b
2
+ c
2

3 + 1
t x =
_
ab+bc+ca
a
2
+b
2
+c
2
1, ta cn chng minh
3

2x
2
+ 1
+ x

3 + 1
D dng kim tra c bt ng thc ny ng vi mi 1 x 0, vy ta c pcm. ng thc xy
ra khi v ch khi a = b = c.

www.VNMATH.com
34 CHNG 2. SOLUTION
17 Chng minh rng vi mi a, b, c > 0, ta c
a
2
b
2
+
b
2
c
2
+
c
2
a
2
+
8(ab + bc + ca)
a
2
+ b
2
+ c
2
11
Li gii. Trc ht, ta s chng minh kt qu sau vi mi x, y, z > 0 tha xyz = 1
x
2
+ y
2
+ z
2
+ 6
3
2
_
x + y + z +
1
x
+
1
y
+
1
z
_
Tht vy, khng mt tnh tng qut, gi s x = min{x, y, z}. t t =

yz v
P(x, y, z) = x
2
+ y
2
+ z
2
+ 6
3
2
_
x + y + z +
1
x
+
1
y
+
1
z
_
Ta c
P(x, y, z) P(x, t, t) =
1
2
_

z
_
2
_
2
_

y +

z
_
2
3
3
bc
_

1
2
_

z
_
2
(8 3 3) 0
Li c
P(x, t, t) = P
_
1
t
2
, t, t
_
=
(t 1)
2
((t
2
2t 1)
2
+ t
2
+ 1)
2t
4
0
Bt ng thc c chng minh. Tr li bi ton ca ta, s dng bt ng thc trn vi x =
a
b
, y =
b
c
, z =
c
a
, ta suy ra c ta ch cn chng minh
3
2

cyc
a
2
+ b
2
ab
+
8(ab + bc + ca)
a
2
+ b
2
+ c
2
17
Hay

cyc
S
c
(a b)
2
0
trong
S
a
=
3
bc

8
a
2
+ b
2
+ c
2
, S
b
=
3
ca

8
a
2
+ b
2
+ c
2
, S
c
=
3
ab

8
a
2
+ b
2
+ c
2
Khng mt tnh tng qut, gi s a b c, khi , d thy S
a
S
b
S
c
, li c
S
b
+ S
c
=
3(b + c)
abc

16
a
2
+ b
2
+ c
2

6
a

bc

16
a
2
+ 2bc

6
a

bc

16
2a

2bc
> 0
T y, ta d dng suy ra pcm. ng thc xy ra khi v ch khi a = b = c.
Nhn xt. Ch rng
_
ab+bc+ca
a
2
+b
2
+c
2
_
2
+ 1
2(ab+bc+ca)
a
2
+b
2
+c
2
, ta suy ra
a
2
b
2
+
b
2
c
2
+
c
2
a
2
+ 4
_
ab + bc + ca
a
2
+ b
2
+ c
2
_
2
7
Kt qu ny c tm ra bi bn Nguyn Anh Cng v c a ln http://mathnfriend.org/

18 Chng minh rng vi mi s dng a


1
, a
2
, . . . , a
n
, b
1
, b
2
, . . . , b
n
, ta c
_
n

i=1
a
2
i
__
n

i=1
b
2
i
_

_
n

i=1
b
i
(a
i
+ b
i
)
__
n

i=1
a
2
i
b
i
a
i
+ b
i
_
www.VNMATH.com
35
Li gii. t f
n
(a
1
, a
2
, . . . , a
n
) = V T V P. Ta s chng minh bt ng thc cho bng quy np.
Vi n := 1 th bt ng thc l hin nhin, gi s bt ng thc ng vi n := n, khi , s dng
gi thit quy np, ta c
f
n+1
(a
1
, a
2
, . . . , a
n+1
) f
n+1
(a
1
, a
2
, . . . , a
n+1
) f
n
(a
1
, a
2
, . . . , a
n
)
=
1
a
n+1
+ b
n+1
n

i=1
(a
n+1
b
i
b
n+1
a
i
)
2
(a
n+1
a
i
+ b
n+1
a
i
+ a
n+1
b
i
)
a
i
+ b
i
0
Vy bt ng thc cho cng ng khi n := n + 1 nn theo nguyn l quy np, n ng vi mi
n.

19 Chng minh rng vi cc s thc a, b, c i mt khc nhau, ta c


(a
2
+ b
2
+ c
2
ab bc ca)
_
1
(a b)
2
+
1
(b c)
2
+
1
(c a)
2
_

27
4
Li gii. Khng mt tnh tng qut, gi s a = min{a, b, c}, t b = a + x, c = a + y th ta c
x, y > 0, x = y (do a, b, c phn bit nhau) v bt ng thc tr thnh
(x
2
xy + y
2
)
_
1
x
2
+
1
y
2
+
1
(x y)
2
_

27
4
Li t t =
x
y
+
y
x
1, d thy t > 1, bt ng thc c vit li nh sau
4t
3
t 1
27
Hay
(2t 3)
2
(t + 3) 0
Bt ng thc ny hin nhin ng, vy ta c pcm.

20 Cho cc s khng m a, b, c, d tha a


2
+ b
2
+ c
2
+ d
2
= 4, chng minh bt ng thc
1
3 abc
+
1
3 bcd
+
1
3 cda
+
1
3 dab
2
Li gii. D dng chng minh c vi mi
8
3

3
x 0, ta c
2
3 x

5x
2
3x + 12
14
S dng bt ng thc ny vi ch l max{abc, bcd, cda, dab}
8
3

3
, ta suy ra c ta ch cn
chng minh
5(a
2
b
2
c
2
+ b
2
c
2
d
2
+ c
2
d
2
a
2
+ d
2
a
2
b
2
) 3(abc + bcd + cda + dab) 8
C nhiu cch chng minh cho bt ng thc ny, xin c gii thiu vi cc bn cch chng
minh sau da vo k thut hm li. t t
2
=
a
2
+b
2
2
, k
2
=
c
2
+d
2
2
v x = ab, y = cd th ta c
t
2
x 0, k
2
y 0, bt ng thc c vit li nh sau
f(x) = 10x
2
k
2
+ 10y
2
t
2
3x
_
2y + 2k
2
3y
_
2x + 2t
2
8 0
www.VNMATH.com
36 CHNG 2. SOLUTION
Ta c
f

(x) = 20k
2
+
3y
(2x + 2t
2
)
3/2
0
Suy ra f(x) l hm li, do
f(x) max{f(t
2
), f(0)}
Ta c
f(0) =
_
yt

2 + 1
__
5yt

2 8
_
(do yt

2
8
3

3
<
8
5
)
f(t
2
) = 10y
2
t
2
6yt + 10k
2
t
2
3t
_
2y + 2k
2
8 = g(y)
Tng t nh trn, ta cng c g(y) l hm li nn
g(y) max{g(k
2
), g(0)}
Ta cng c
g(0) =
_
kt

2 + 1
__
5kt

2 8
_
(do kt

2
8
3

3
<
8
5
)
g(t
2
) = 4(kt 1)(5kt + 1) 0 (do kt
k
2
+ t
2
2
= 1)
Bt ng thc c chng minh xong. ng thc xy ra khi v ch khi a = b = c = d = 1.

21 Cho cc s dng a, b, c, chng minh bt ng thc


a
b
+
b
c
+
c
a
3
_
a
2
+ b
2
+ c
2
ab + bc + ca
Li gii. Nhn xt rng ta ch cn chng minh bt ng thc cho trong trng hp a b c l ,
khi , ta c
b
a
+
a
c
+
c
b

a
b
+
b
c
+
c
a
. Mt khc, s dng kt qu bi ton 17, ta c

cyc
a
2
b
2
+ 6
3
2

cyc
a
2
+ b
2
ab
Suy ra
_

cyc
a
b
_
2
=

cyc
a
2
b
2
+

cyc
b
a
+

cyc
b
a

3
2

cyc
a
2
+ b
2
ab
6 +

cyc
a
b
+

cyc
b
a
=
5
2

cyc
a
2
+ b
2
ab
6
Ta cn chng minh
5
2

cyc
a
2
+ b
2
ab
6
9(a
2
+ b
2
+ c
2
)
ab + bc + ca
Hay

cyc
(a b)
2
_
5
ab

9
ab + bc + ca
_
0
Hay

cyc
(a b)
2
_
5c
a
+
5c
b
4
_
0
Khng mt tnh tng qut, gi s a b c > 0, khi , ta c
5a
b
+
5a
c
4
5b
c
+
5b
a
4
5c
a
+
5c
b
4
www.VNMATH.com
37
Li c
_
5b
c
+
5b
a
4
_
+
_
5c
a
+
5c
b
4
_

5(b
2
+ c
2
)
bc
8 2 > 0
Bt ng thc c chng minh xong. ng thc xy ra khi v ch khi a = b = c.

22 Cho cc s khng m a, b, c, chng minh bt ng thc


7
_
3(a
2
+ b
2
+ c
2
)
a + b + c
+
a
2
b + b
2
c + c
2
a
a
3
+ b
3
+ c
3
8
Li gii. S dng kt qu bi ton 16, ta c
b
a
+
a
c
+
c
b

9(a
2
+ b
2
+ c
2
)
(a + b + c)
2
Suy ra
a
2
b + b
2
c + c
2
a
9abc(a
2
+ b
2
+ c
2
)
(a + b + c)
2
Ta cn chng minh
7
_
3(a
2
+ b
2
+ c
2
)
a + b + c
+
9abc(a
2
+ b
2
+ c
2
)
(a + b + c)
2
(a
3
+ b
3
+ c
3
)
8
Khng mt tnh tng qut, gi s a +b +c = 1, t x = ab +bc +ca th ta c
1
3
x 0. Hn na,
theo bt ng thc Schur, ta suy abc
4x1
9
, do
9abc
a
3
+ b
3
+ c
3
=
9abc
3abc + 1 3x

3(4x 1)
2 5x
Nh th, ta phi chng minh
7
_
3(1 2x) +
3(4x 1)(1 2x)
2 5x
8
Ta c
147(1 2x)
_
8
3(4x 1)(1 2x)
2 5x
_
2
=
(3x 1)
2
(227 550x 64x
2
)
(2 5x)
2

(3x 1)
2
_
227 550
1
3
64
1
9
_
(2 5x)
2
=
329(3x 1)
2
9(2 5x)
2
0
Bt ng thc c chng minh xong. ng thc xy ra khi v ch khi a = b = c.

23 Chng minh rng vi mi s dng a, b, c ta c


a
3
a
3
+ abc + b
3
+
b
3
b
3
+ abc + c
3
+
c
3
c
3
+ abc + a
3
1
Li gii. t x =
b
a
, y =
a
c
, z =
c
b
th ta c x, y, z > 0, xyz = 1 v bt ng thc tr thnh

cyc
1
x
3
+
x
y
+ 1
1
www.VNMATH.com
38 CHNG 2. SOLUTION
Hay

cyc
1
x
3
+ x
2
z + 1
1
Hay

cyc
yz
x
2
+ yz + zx
1
S dng bt ng thc Cauchy Schwarz, ta c

cyc
yz
x
2
+ yz + zx

(yz + zx + xy)
2
yz(x
2
+ yz + zx) + zx(y
2
+ zx + xy) + xy(z
2
+ xy + yz)
= 1
Bt ng thc c chng minh xong. ng thc xy ra khi v ch khi a = b = c hoc
a
b
0,
b
c
0
v cc hon v.

24 Cho cc s dng a, b, c, d, chng minh rng


abc
(d + a)(d + b)(d + c)
+
abd
(c + a)(c + b)(c + d)
+
acd
(b + a)(b + c)(b + d)
+
bcd
(a + b)(a + c)(a + d)

1
2
Li gii. t x =
1
a
, y =
1
b
, z =
1
c
, t =
1
d
th ta c x, y, z, t > 0 v bt ng thc tr thnh

cyc
x
3
(x + y)(x + z)(x + t)

1
2
S dng bt ng thc Cauchy Schwarz, ta suy ra c ta ch cn chng minh
2
_

cyc
x
2
_
2

cyc
x(x + y)(x + z)(x + t)
Hay
2
_

cyc
x
2
_
2

cyc
x
4
+

cyc
(x
3
y + y
3
z + z
3
x) + (x + y + z + t)(xyz + yzt + zxt + txy)
S dng kt qu bi ton trc, ta c 3(x
3
y + y
3
z + z
3
x) (x
2
+ y
2
+ z
2
)
2
, ta cn chng minh
2
_

cyc
x
2
_
2

cyc
x
4
+
1
3

cyc
(x
2
+ y
2
+ z
2
)
2
+ (x + y + z + t)(xyz + yzt + zxt + txy)
Hay
4
3

cyc
(x
2
y
2
+ y
2
z
2
+ z
2
x
2
) (x + y + z + t)(xyz + yzt + zxt + txy)
S dng bt ng thc AMGM, ta c
V T

cyc
xyz(x + y + z) +
1
3

cyc
(x
2
y
2
+ y
2
z
2
+ z
2
x
2
)

cyc
xyz(x + y + z) + 4xyzt
= (x + y + z + t)(xyz + yzt + zxt + txy) = V P
Bt ng thc c chng minh xong. ng thc xy ra khi v ch khi a = b = c = d.

www.VNMATH.com
39
25 Chng minh rng vi mi a, b, c > 0, ta c
a
b+c
+ b
c+a
+ c
a+b
1
Li gii. Nu 1 trong cc s a, b, c khng nh hn 1 th bt ng thc l hin nhin. Xt trng hp
a, b, c 1, khi c 2 kh nng
Kh nng 1. Nu a+b+c 1, suy ra max{a+b, b+c, c+a} 1, s dng bt ng thc Bernoulli,
ta c
1
a
b+c
=
_
1 +
1
a
1
_
b+c
1 +
_
1
a
1
_
(b + c) 1 +
b + c
a
=
a + b + c
a
Suy ra
a
b+c

a
a + b + c
S dng tng t vi b, c ri cng li, ta c pcm.
Kh nng 2. Nu a + b + c 1, li s dng bt ng thc Bernoulli, ta c
1
a
c

a + c(1 a)
a
,
1
a
b

a + b(1 a)
a
Suy ra
a
b+c

a
2
(a + b(1 a))(a + c(1 a))
S dng tng t vi b, c, ta cn chng minh

cyc
a
2
(a + b(1 a))(a + c(1 a))
1
S dng bt ng thc Cauchy Schwarz, ta c

cyc
a
2
(a + b(1 a))(a + c(1 a))

(a + b + c)
2

cyc
(a + b(1 a))(a + c(1 a))
Ta li c
(a +b +c)
2

cyc
(a +b(1 a))(a +c(1 a)) = (ab +bc +ca)(a +b +c 1) +abc(3 a b c) 0
Bt ng thc c chng minh xong.

26 Cho n 3, n N v x
1
, x
2
, . . . , x
n
l cc s khng m c tng bng 1. Tm gi tr ln nht ca biu
thc
P(x
1
, x
2
, . . . , x
n
) = x
3
1
x
2
2
+ x
3
2
x
2
3
+ + x
3
n
x
2
1
+ n
2(n1)
x
3
1
x
3
2
x
3
n
Li gii. Khng mt tnh tng qut, ta c th gi s x
1
= max{x
1
, x
2
, . . . , x
n
}, ta s chng minh
P(x
1
, x
2
, . . . , x
n
) P(x
1
, x
2
+ + x
n
, 0, . . . , 0)
Tht vy, ta c
P(x
1
, x
2
+ + x
n
, 0, . . . , 0) = x
3
1
(x
2
+ + x
n
)
2
2(x
3
1
x
2
x
3
+ x
3
1
x
3
x
4
+ + x
3
1
x
n1
x
n
) + x
3
1
x
2
2
+ x
3
1
x
2
n
(x
3
1
x
2
x
3
+ x
3
1
x
3
x
4
+ + x
3
1
x
n1
x
n
) + (x
3
2
x
2
3
+ + x
3
n1
x
2
n
) + x
3
1
x
2
2
+ x
3
n
x
2
1
x
3
1
x
2
x
3
+ x
3
1
x
2
2
+ x
3
2
x
2
3
+ + x
3
n
x
2
1
www.VNMATH.com
40 CHNG 2. SOLUTION
Ta cn chng minh
x
3
1
x
2
x
3
n
2(n1)
x
3
1
x
3
2
x
3
n
Nu n = 3, bt ng thc tr thnh x
2
x
3

1
9
. S dng bt ng thc AMGM, ta c
x
2
x
3

_
x
2
+ x
3
2
_
2

_
x
1
+ x
2
+ x
3
3
_
2
=
1
9
Nu n > 3, bt ng thc tr thnh
x
2
2
x
2
3
x
3
4
. . . x
3
n

1
n
2(n1)
S dng bt ng thc AMGM, ta c
x
2
2
x
2
3
x
3
4
x
3
n
(x
2
x
3
x
n
)
2

_
x
2
+ x
3
+ + x
n
n 1
_
2(n1)

_
x
1
+ x
2
+ + x
n
n
_
2(n1)
=
1
n
2(n1)
Bt ng thc c chng minh xong. Li c
x
3
1
(x
2
+ + x
n
)
2
= 108
_
x
1
3
_
3
_
x
2
+ . . . + x
n
2
_
2
108
_
x
1
+ x
2
+ + x
n
5
_
5
=
108
3125
Suy ra P(x
1
, x
2
, . . . , x
n
)
108
3125
. Mt khc, cho x
1
=
3
5
, x
2
=
2
5
, x
3
= = x
n
= 0, ta c
P(x
1
, x
2
, . . . , x
n
) =
108
3125
. Vy
max P(x
1
, x
2
, . . . , x
n
) =
108
3125
.

27 Cho cc s thc a
1
, a
2
, . . . , a
n
tha a
1
a
2
a
n
= 1, tm cc hng s tt nht m, M sao cho
_
a
2
1
+ n
2
1 +
_
a
2
2
+ n
2
1 + +
_
a
2
n
+ n
2
1 m(a
1
+ a
2
+ + a
n
) + M
Li gii. Cho a
1
= a
2
= = a
n
= 1, ta suy ra c mn + M n
2
. Li cho a
1
= a
2
= = a
n1
=
1
x
> 0, a
n
= x
n1
, ta c
_
x
2n2
+ n
2
1 + (n 1)
_
1
x
2
+ n
2
1 m
_
x
n1
+
n 1
x
_
+ M
_
1 +
n
2
1
x
2n2
+ (n 1)
_
1
x
2n
+
n
2
1
x
2n2
m
_
1 +
n 1
x
n
_
+
M
x
n1
Cho x , ta suy ra c m 1, do
m
n

i=1
a
i
+ M m
n

i=1
a
i
+ n
2
mn = m
_
n

i=1
a
i
n
_
+ n
2

i=1
a
i
+ n(n 1)
T y, ta s chng minh m = 1, M = n(n 1) l cc hng s cn tm, tc l
n

i=1
_
a
2
i
+ n
2
1
n

i=1
a
i
+ n(n 1)
Ta s chng minh vi mi x > 0 th
_
x
2
+ n
2
1 x +
n(n 1)
x + n 1
www.VNMATH.com
41
n + 1
x +

x
2
+ n
2
1

n
x + n 1
S dng bt ng thc Cauchy Schwarz, ta c
x +
_
x
2
+ n
2
1 x +
x + n
2
1
n
=
(n + 1)(x + n 1)
n
S dng bt ng thc ny ln lt cho a
1
, a
2
, . . . , a
n
ri cng li, ta cn chng minh
n

i=1
1
a
i
+ n 1
1
t a
i
= x
n
i
> 0, ta c theo bt ng thc AMGM
n 1
a
i
+ n 1
= 1
a
i
a
i
+ n 1
= 1
x
n1
i
x
n1
i
+ (n 1)x
1
x
i1
x
i+1
x
n
1
x
n1
i
x
n1
1
+ x
n1
2
+ + x
n1
n
Cho i = 1, 2, . . . , n, ri cng li ta c pcm. Vy m = 1, M = n(n 1) l cc hng s tt nht ca
bt ng thc cho.

28 Chng minh rng vi mi s dng a, b, c, d, ta c


a
3a
2
+ 2b
2
+ c
2
+
b
3b
2
+ 2c
2
+ d
2
+
c
3c
2
+ 2d
2
+ a
2
+
d
3d
2
+ 2a
2
+ b
2

1
6
_
1
a
+
1
b
+
1
c
+
1
d
_
Li gii. S dng bt ng thc AMGM v bt ng thc Cauchy Schwarz, ta c
18a
3a
2
+ 2b
2
+ c
2
=
18a
2(a
2
+ b
2
) + a
2
+ c
2

9
2b + c

2
b
+
1
c
Tng t, ta c
18b
3b
2
+ 2c
2
+ d
2

2
c
+
1
d
,
18c
3c
2
+ 2d
2
+ a
2

2
d
+
1
a
,
18d
3d
2
+ 2a
2
+ b
2

2
a
+
1
b
Cng tng ng cc bt ng thc trn v theo v, ta c pcm. ng thc xy ra khi v ch khi
a = b = c = d.

29 Cho cc s dng x, y, z, chng minh bt ng thc


x(y + z)
x
2
+ yz
+
y(z + x)
y
2
+ zx
+
z(x + y)
z
2
+ xy

x + y + z
3

xyz

x
2
+ yz
x(y + z)
+
y
2
+ zx
y(z + x)
+
z
2
+ xy
z(x + y)
Li gii. 1. Trc ht, ta s chng minh

cyc
x(y + z)
x
2
+ yz

x + y + z
3

xyz
t x = a
3
, b = y
3
, z = c
3
(a, b, c > 0), ta c bt ng thc tng ng

cyc
a
3
(b
3
+ c
3
)
a
6
+ b
3
c
3

a
3
+ b
3
+ c
3
abc
S dng bt ng thc x
3
+ y
3
xy(x + y) x, y > 0, ta c

cyc
a
3
(b
3
+ c
3
)
a
6
+ b
3
c
3

1
abc

cyc
a
2
(b
3
+ c
3
)
a
2
+ bc
www.VNMATH.com
42 CHNG 2. SOLUTION
Ta cn chng minh

cyc
a
2
(b
3
+ c
3
)
a
2
+ bc

cyc
a
3
Hay

cyc
a
2
(a
3
+ abc b
3
c
3
)
a
2
+ bc
0

cyc
a
3
(a b)(a c)
a
2
+ bc
+

cyc
ab(a b)
2
(a + b)(ac + bc ab)
(a
2
+ bc)(b
2
+ ca)
0
Khng mt tnh tng qut, gi s c = min{a, b, c}, ta c
V T
a
3
(a b)(a c)
a
2
+ bc
+
b
3
(b a)(b c)
b
2
+ ca

a
2
b
2
(a b)
2
(a + b)
(a
2
+ bc)(b
2
+ ca)
=
c(a b)
2
(a + b)(a
3
+ b
3
a
2
c b
2
c)
(a
2
+ bc)(b
2
+ ca)
0
2. Ta cn phi chng minh

cyc
x
2
+ yz
x(y + z)

x + y + z
3

xyz
Nu
xy+yz+zx
x+y+z

3

xyz, s dng bt ng thc Cauchy Schwarz v bt ng thc AMGM,


ta c
V T =

cyc
x
2
x(y + z)
+

cyc
yz
x(y + z)

(x + y + z)
2
2(xy + yz + zx)
+
(xy + yz + zx)
2
2xyz(x + y + z)

(x + y + z)(xy + yz + zx)
xyz

x + y + z
3

xyz
Nu
3

xyz
xy+yz+zx
x+y+z
, s dng bt ng thc ny, ta cn chng minh
(xy + yz + zx)

cyc
x
2
+ yz
x(y + z)
(x + y + z)
2
Hay

cyc
y
2
z
2
+ x
2
yz
x(y + z)
xy + yz + zx
Hay

cyc
(yz + xy)(yz + zx)
xy + zx
(xy + yz) + (yz + zx) + (zx + xy)
Bt ng thc ny ng theo bt ng thc AMGM, vy ta c pcm.

30 Vi mi s dng a, b, c tha a + b + c = 3, ta c
a
b
2
+ c
+
b
c
2
+ a
+
c
a
2
+ b

3
2
www.VNMATH.com
43
Li gii. Xt 2 trng hp
Trng hp 1. a b c, s dng bt ng thc Cauchy Schwarz, ta c

cyc
a
b
2
+ c

(a + b + c)
2

cyc
ab
2
+

cyc
ab
=
9

cyc
ab
2
+

cyc
ab
Ta cn chng minh

cyc
ab
2
+

cyc
ab 6
Hay
2

cyc
a
3
+ 3

cyc
a
2
b + 3abc 6

cyc
ab
2
Bt ng thc ny ng do

cyc
a
3

cyc
a
2
b

cyc
ab
2

cyc
a
3
+ 3abc

cyc
a
2
b +

cyc
ab
2
2

cyc
ab
2
Trng hp 2. c b a, bt ng thc c vit li nh sau
2

cyc
a
4
+ 2

cyc
a
2
b
3
+ 2

cyc
a
2
b
2
+ 3abc 3a
2
b
2
c
2
+ 3

cyc
ab
3
+ 3

cyc
a
3
b
2
S dng kt qu bi ton trc v bt ng thc AMGM, ta c

cyc
a
4
+ 2

cyc
a
2
b
2
3

cyc
ab
3
, 1 abc
Ta cn phi chng minh

cyc
a
4
+ 2

cyc
a
2
b
3
3

cyc
a
3
b
2
Hay

cyc
a
5
+

cyc
ab(a
3
+ b
3
) + 6

cyc
a
2
b
3
9

cyc
a
3
b
2
Bt ng thc ny ng do

cyc
a
5

cyc
a
2
b
3

cyc
a
3
b
2

cyc
ab(a
3
+ b
3
)

cyc
a
2
b
2
(a + b) =

cyc
a
3
b
2
+

cyc
a
2
b
3
2

cyc
a
3
b
2
Bt ng thc c chng minh xong. ng thc xy ra khi v ch khi a = b = c = 1.

31 Vi mi s khng m a, b, c tha a + b + c = 3, ta c
a
_
b
3
+ 1 + b
_
c
3
+ 1 + c
_
a
3
+ 1 5
Li gii. S dng bt ng thc AMGM, ta c

cyc
a
_
b
3
+ 1 =

cyc
a
_
(b + 1)(b
2
b + 1)
1
2

cyc
a(b
2
+ 2) =
1
2

cyc
ab
2
+ 3
www.VNMATH.com
44 CHNG 2. SOLUTION
Ta cn chng minh

cyc
ab
2
4
Khn mt tnh tng qut, gi s c b a 0, ta c
a(b a)(b c) 0
Suy ra
ab
2
+ a
2
c a
2
b + abc a
2
b + 2abc
Do

cyc
ab
2
bc
2
+ a
2
b + 2abc = b(a + c)
2

1
2
_
2b + (a + c) + (a + c)
3
_
3
= 4
Bt ng thc c chng minh xong. ng thc xy ra khi v ch khi (a, b, c) = (0, 1, 2) v cc
hon v tng ng.

32 Tm hng s k tt nht sao cho bt ng thc sau ng vi mi a, b, c > 0


(a + b + c)
_
1
a
+
1
b
+
1
c
_
9 +
k max{(a b)
2
, (b c)
2
, (c a)
2
}
(a + b + c)
2
Li gii. Khng mt tnh tng qut, gi s a b c, ta cn tm k sao cho
(a + b + c)
_
1
a
+
1
b
+
1
c
_
9 +
k(a c)
2
(a + b + c)
2
Cho b =
a+c
2
, bt ng thc tr thnh
3(a c)
2
2ac

4k(a c)
2
9(a + c)
2
Hay
k
27(a + c)
2
8ac
Cho a = c, ta suy ra c k
27
2
, ta s chng minh y l gi tr cn tm, tc l
(a + b + c)
_
1
a
+
1
b
+
1
c
_
9 +
27(a c)
2
2(a + b + c)
2
t a = b + x, c = b y th ta c x 0, b y 0, bt ng thc tng ng vi
9(xy)
2
b
3
+3(y x)(x
2
+16xy +y
2
)b
2
+(4x
4
+11x
3
y +78x
2
y
2
+11xy
3
+4y
4
)b +2xy(y x)
3
0
Nu y x th ta c ngay pcm, xt x y, khi , ta c
2x
4
b + 2xy(y x)
3
2x
4
y + 2xy(y x)
3
2x
4
y 2x
4
y = 0
Ta cn chng minh
f(b) = 9(x y)
2
b
2
+ 3(y x)(x
2
+ 16xy + y
2
)b + 2x
4
+ 11x
3
y + 78x
2
y
2
+ 11xy
3
+ 4y
4
0
Nhng bt ng thc ny ng v

f
= 9(7x
4
+ 12x
3
y + 54x
2
y
2
+ 12xy
3
+ 15y
4
)(x y)
2
0
Vy ta c pcm, t ta i n kt lun
k
max
=
27
2
.

www.VNMATH.com
45
33 Cho cc s dng x, y, z c tch bng 1, chng minh rng vi mi k 0, ta c
3
_
x
y + k
+
3
_
y
z + k
+
3
_
z
x + k

3
3

k + 1
Li gii. Do x, y, z > 0, xyz = 1 nn tn ti a, b, c > 0 sao cho x =
a
4
b
4
, y =
c
4
a
4
, z =
b
4
c
4
, bt ng thc
tr thnh

cyc
a
8/3
b
4/3
3

c
4
+ ka
4

3
3

k + 1
S dng bt ng thc Holder, ta c
_

cyc
a
8/3
b
4/3
3

c
4
+ ka
4
_
3
_

cyc
(c
4
+ ka
4
)
_

cyc
a
2
b
_
4
Ta cn chng minh
_

cyc
a
2
b
_
4

27
k + 1

cyc
(c
4
+ ka
4
)
Hay

cyc
a
2
b

4
_
27(a
4
+ b
4
+ c
4
)
Khng mt tnh tng qut, gi s a
4
+ b
4
+ c
4
= 3, suy ra a
4

3 < 2, do

cyc
4(a
3
a
2
) =

cyc
4(a
3
a
2
)

cyc
(a
4
1) =

cyc
(a 1)
2
(1 + 2a a
2
) 0
Suy ra
a
3
+ b
3
+ c
3
a
2
+ b
2
+ c
2
1
Do , ta ch cn chng minh

cyc
a
2
b

3(a
3
+ b
3
+ c
3
)
a
2
+ b
2
+ c
2
Hay

cyc
S
a
(b c)
2
trong
S
a
=
a
2
+ b
2
c
b, S
b
=
b
2
+ c
2
a
c, S
c
=
c
2
+ a
2
b
a
C 2 trng hp xy ra
Trng hp 1. a b c, khi d thy S
a
, S
c
0, ta li c
S
a
+ 2S
b
=
a
2
c
+
b
2
a
2c +
b(b c)
c
+
b
2
+ 2c
2
a

a
2
c
+
b
2
a
2c
a
2
b
+
b
2
a
2c a + b 2c 0
Ta s chng minh S
c
+ 2S
b
0. Tht vy, nu a
2
2b
2
, ta c
S
c
+ 2S
b
=
(a b)(a
2
2b
2
)
ab
+
c
2
b
+
2c
2
a
+ 2(b c) 0
Nu 2b
2
a
2
v a 2c, ta c
S
c
+ 2S
b
=
a(a b)
b
+
c
2
b
+
2c
2
a
+
2b
2
a
2c
2b
2
a
2c a 2c 0
www.VNMATH.com
46 CHNG 2. SOLUTION
Nu 2b
2
a
2
v 2c a, ta c
S
c
+ 2S
b
=
a(a b)
b
+
c
2
b
+
2c
2
a
+
2b
2
a
2c
a(a b)
b
+
a
2
4b
+
a
2
+
2b
2
a
2c
=
(a b)(5a 4b)
4ab
+
3a
4
+
b
2
a
+ b 2c
3a
4
+
a
2
c 0
Trng hp 2. c b a, khi d thy S
b
, S
c
0, ta c
S
b
+ S
a
=
b(b a) + c(c a)
a
+
a
2
+ b
2
c
0
Bt ng thc c chng minh xong. ng thc xy ra khi v ch khi x = y = z = 1.
Nhn xt. Bt ng thc vi 4 s vn cn ng
3
_
x
y + k
+
3
_
y
z + k
+
3
_
z
t + k
+
3
_
t
x + k

4
3

k + 1
x, y, z, t, k > 0, xyzt = 1

34 Cho cc s dng a, b, c, chng minh bt ng thc


b
2
+ c
2
a(b + c)
+
c
2
+ a
2
b(c + a)
+
a
2
+ b
2
c(a + b)
(a
2
+ b
2
+ c
2
)

3
abc(a + b + c)
Li gii. Bt ng thc tng ng

cyc
b
2
+ c
2
a(b + c)

(a
2
+ b
2
+ c
2
)
_
3abc(a + b + c)
abc(a + b + c)
S dng bt ng thc AMGM, ta c ab + bc + ca
_
3abc(a + b + c), ta cn chng minh
abc(a + b + c)

cyc
b
2
+ c
2
a(b + c)
(ab + bc + ca)(a
2
+ b
2
+ c
2
)
Hay
1
2
abc

cyc
(b c)
2
b + c
0
Bt ng thc c chng minh. ng thc xy ra khi v ch khi a = b = c.

35 Cho cc s dng a, b, c, chng minh bt ng thc


2
_
a
2
b
+
b
2
c
+
c
2
a
_
+ 3(a + b + c)
15(a
2
+ b
2
+ c
2
)
a + b + c
Li gii. Khng mt tnh tng qut, gi s a = min{a, b, c}, t b = a + x, c = a + y (x, y 0), bt
ng thc c th vit li nh sau
(x
2
xy + y
2
)a
3
+ 3xy(2y x)a
2
+ (x
4
5x
3
y + 6x
2
y
2
+ xy
3
+ y
4
)a + xy
3
(x + y) 0
Ta s chng minh
g(a) = (x
2
xy + y
2
)a
2
+ 3xy(2y x)a + x
4
5x
3
y + 6x
2
y
2
+ xy
3
+ y
4
0
www.VNMATH.com
47
Tht vy, ta c

g
= (4x
6
24x
5
y + 39x
4
y
2
4x
3
y
3
12x
2
y
4
+ 4y
6
) = f(x)
Nu x 3y, ta c f

(x) = 12x(x2y)(2x
2
(x3y) +xy
2
+y
3
) 0 nn f(x) l hm ng bin, suy
ra f(x) f(3y) = 31y
6
0. Nu x 3y, ta c
f(x) = (2x
3
6x
2
y + xy
2
+ y
3
)
2
+ x
3
y
2
(3y x) + y
3
(x
3
+ 4y
3
y(x + y)
2
)
y
3
(x
3
+ 4y
3
y(x + y)
2
) y
3
_
1
4
(x + y)
3
+ 3y
3
y(x + y)
2
_
= y
3
_
1
8
(x + y)
3
+
1
8
(x + y)
3
+ 3y
3
y(x + y)
2
_

_
3
3

3
4
1
_
y
4
(x + y)
2
0
Nh th, ta lun c f(x) 0, suy ra g(a) 0. Vy ta c pcm. ng thc xy ra khi v ch khi
a = b = c.

36 Chng minh rng vi mi s thc dng x, y, z c tch bng 1 v vi mi k 0, ta c


4
_
x
y + k
+
4
_
y
z + k
+
4
_
z
x + k

3
4

k + 1
Li gii. Do x, y, z > 0, xyz = 1 nn tn ti cc s dng a, b, c sao cho x =
a
5
b
5
, y =
c
5
a
5
, z =
b
5
c
5
, khi
bt ng thc tr thnh

cyc
a
5/2
b
5/4
4

c
5
+ ka
5

3
4

k + 1
S dng bt ng thc Holder, ta c
_

cyc
a
5/2
b
5/4
4

c
5
+ ka
5
_
4
_

cyc
(c
5
+ ka
5
)
_

cyc
a
2
b
_
5
Ta cn chng minh
_

cyc
a
2
b
_
5

81
k + 1

cyc
(c
5
+ ka
5
)
Hay

cyc
a
2
b

5
_
81(a
5
+ b
5
+ c
5
)
S dng kt qu bi trn, ta cn chng minh
15(a
2
+ b
2
+ c
2
)
a + b + c
3(a + b + c) 2
5
_
81(a
5
+ b
5
+ c
5
)
Khng mt tnh tng qut, gi s a + b + c = 1, t ab + bc + ca =
1q
2
3
, r = abc (1 q 0), th
th ta c r
(1q)
2
(1+2q)
27
, khi , ta c
a
5
+ b
5
+ c
5
=
1
9
(15(q
2
+ 2)r + 35q
4
25q
2
1)
Do , bt ng thc tng ng
5q
2
+ 1
5
_
9(15(q
2
+ 2)r + 35q
4
25q
2
1)
www.VNMATH.com
48 CHNG 2. SOLUTION
Do r
(1q)
2
(1+2q)
27
nn ta ch cn chng minh
5q
2
+ 1
5
_
5(q
2
+ 2)(1 q)
2
(1 + 2q) + 315q
4
225q
2
9
Hay
5q
2
+ 1
5
_
10q
5
+ 300q
4
+ 20q
3
250q
2
+ 1
Ta c
V T
5
V P = 5q
2
(625q
8
+ 625q
6
+ 250q
4
2q
3
10q
2
4q + 55) 0 (do 1 q 0)
Bt ng thc c chng minh xong. ng thc xy ra khi v ch khi x = y = z = 1.
Nhn xt. T kt qu bi ny, ta c th suy ra c kt qu bi 33. Mt cu hi t nhin t ra l: Vi
nhng gi tr no ca n th bt ng thc sau ng
n
_
x
y + k
+
n
_
y
z + k
+
n
_
z
x + k

3
n

k + 1

37 Chng minh rng vi mi s khng m a, b, c v vi mi k 3, ta c


a(b
k
+ c
k
)
a
2
+ bc
+
b(c
k
+ a
k
)
b
2
+ ca
+
c(a
k
+ b
k
)
c
2
+ ab
a
k1
+ b
k1
+ c
k1
Li gii. Ta s ch ra rng ta ch cn xt bt ng thc trong trng hp k = 3 l , tht vy xt hm
s
f(k) =
1
a
k1
+ b
k1
+ c
k1

cyc
a(b
k
+ c
k
)
a
2
+ bc
Ta c
(a
k1
+ b
k1
+ c
k1
)
2
f

(k) =

cyc
a
k1
b
k1
(a b)(ln a ln b)
_
c
c
2
+ ab
+
ab(a + b c)
(a
2
+ bc)(b
2
+ ca)
_

cyc
ca
k1
b
k1
(a b)(ln a ln b)
_
1
c
2
+ ab

ab
(a
2
+ bc)(b
2
+ ca)
_
=

cyc
c
2
a
k1
b
k1
(a b)(ln a ln b)(a
3
+ b
3
)
(a
2
+ bc)(b
2
+ ca)(c
2
+ ab)
0
Do f(k) l hm ng bin, v nh th nu bt ng thc trong trng hp k = 3 th cng ng
cho mi k 3. Ta cn phi chng minh vi k = 3 th bt ng thc ng, hay

cyc
a(b
3
+ c
3
)
a
2
+ bc

cyc
a
2
Hay

cyc
a(b
3
+ c
3
)
a
2
+ bc

cyc
(b
2
bc + c
2
)

cyc
ab

cyc
a
2

cyc
M
a
(a b)(a c) 0
trong
M
a
=
(a + b c)(a + c b)
a
2
+ bc
, M
b
=
(b + c a)(a + b c)
b
2
+ ca
, M
c
=
(c + a b)(b + c a)
c
2
+ ab
www.VNMATH.com
49
Khng mt tnh tng qut, gi s a b c, xt 2 trng hp
Trng hp 1. b + c a, khi , ta c M
a
, M
b
, M
c
0, li c
aM
a
bM
b
=
(a + b c)(a b)(ab(a + b c) + c(a
2
+ b
2
) + c
2
(a + b))
(a
2
+ bc)(b
2
+ ca)
0
Trng hp 2. a b + c, khi , ta c M
a
0 M
b
, M
c
, vit li bt ng thc nh sau
(a c)(M
a
(a b) + M
c
(b c)) M
b
(a b)(b c) 0
Ta cn chng minh
M
a
(a b) + M
c
(b c) 0
Ta c
M
a
(a b) + M
c
(b c) = (a + c b)
_
(a b)(a + b c)
a
2
+ bc

(b c)(a b c)
c
2
+ ab
_
(a + c b)(a b c)
_
a
a
2
+ bc

b c
c
2
+ ab
_
=
c(a + c b)(a b c)(a
2
b
2
+ ac + bc)
(a
2
+ bc)(c
2
+ ab)
0
Bt ng thc c chng minh xong. ng thc xy ra khi v ch khi a = b = c hoc (a, b, c)
(1, 1, 0).

38 Cho cc s khng m a, b, c, chng minh bt ng thc


a
4
a
3
+ abc + b
3
+
b
4
b
3
+ abc + c
3
+
c
4
c
3
+ abc + a
3

a
3
+ b
3
+ c
3
a
2
+ b
2
+ c
2
Li gii. Xt 2 trng hp
Trng hp 1. a
3
b
2
+ b
3
c
2
+ c
3
a
2
abc(a
2
+ b
2
+ c
2
), khi s dng bt ng thc Cauchy
Schwarz, ta c

cyc
a
4
a
3
+ abc + b
3

(a
3
+ b
3
+ c
3
)
2
a
5
+ b
5
+ c
5
+ abc(a
2
+ b
2
+ c
2
) + a
2
b
3
+ b
2
c
3
+ c
2
a
3
Ta cn chng minh
(a
3
+ b
3
+ c
3
)(a
2
+ b
2
+ c
2
) a
5
+ b
5
+ c
5
+ abc(a
2
+ b
2
+ c
2
) + a
2
b
3
+ b
2
c
3
+ c
2
a
3
Hay
a
3
b
2
+ b
3
c
2
+ c
3
a
2
abc(a
2
+ b
2
+ c
2
) (ng)
Trng hp 2. a
3
b
2
+ b
3
c
2
+ c
3
a
2
abc(a
2
+ b
2
+ c
2
), khi , ta chng minh c
3(a
4
b
3
+ b
4
c
3
+ c
4
a
3
)
1
abc
(a
3
b
2
+ b
3
c
2
+ c
3
a
2
)
2
(a
3
b
2
+ b
3
c
2
+ c
3
a
2
)(a
2
+ b
2
+ c
2
)
abc(a
2
+ b
2
+ c
2
)
2
Do , s dng bt ng thc Cauchy Schwarz, ta c

cyc
a
4
a
3
+ abc + b
3

(a
4
+ b
4
+ c
4
)
2
a
7
+ b
7
+ c
7
+ abc(a
4
+ b
4
+ c
4
) + a
4
b
3
+ b
4
c
3
+ c
4
a
3

3(a
4
+ b
4
+ c
4
)
2
3(a
7
+ b
7
+ c
7
) + 3abc(a
4
+ b
4
+ c
4
) + abc(a
2
+ b
2
+ c
2
)
2
www.VNMATH.com
50 CHNG 2. SOLUTION
Ta cn chng minh
3(a
4
+ b
4
+ c
4
)
2
3(a
7
+ b
7
+ c
7
) + 3abc(a
4
+ b
4
+ c
4
) + abc(a
2
+ b
2
+ c
2
)
2

a
3
+ b
3
+ c
3
a
2
+ b
2
+ c
2
Khng mt tnh tng qut, gi s a + b + c = 1, t ab + bc + ca =
1q
2
3
(1 q 0) v r = abc,
th th ta c 0 r
(1q)
2
(1+2q)
27
v bt ng thc tr thnh f(r) 0, vi
f(r) = 99r
3
+
29 178q
2
31q
4
3
r
2

(q
2
+ 5)(20q
4
6q
2
+ 1)
9
r +
(q
2
1)
2
(8q
6
+ 21q
4
3q
2
+ 1)
81
Theo bt ng thc Schur, ta c 27r 1 4q
2
, do
f

(r) = 594r +
2(29 178q
2
31q
4
)
3
22(1 4q
2
) +
2(29 178q
2
31q
4
)
3
=
2(31q
4
+ 46q
2
+ 4)
3
< 0
Suy ra f(r) l hm lm, do
f(r) min
_
f(0)f,
_
(1 q)
2
(1 + 2q)
27
__
Li c
f(0) =
(q
2
1)
2
(8q
6
+ 21q
4
3q
2
+ 1)
81
0
f
_
(1 q)
2
(1 + 2q)
27
_
=
2q
2
(q 1)
2
(46q
6
+ 54q
5
+ 102q
4
+ 13q
3
+ 36q
2
12q + 4)
2187
0
hon thnh chng minh ca bi ton, xin c nu mt li gii cho bt ng thc
3abc(a
4
b
3
+ b
4
c
3
+ c
4
a
3
) (a
3
b
2
+ b
3
c
2
+ c
3
a
2
)
2
t x =
1
a
, y =
1
b
, z =
1
c
, th th bt ng thc tr thnh
_

cyc
x
2
y
_
2
3

cyc
x
3
y
Hay

cyc
S
x
(y z)
2
0
trong
S
x
=
y
2
z
2

y
z
+
2x
y

3
2
, S
y
=
z
2
x
2

z
x
+
2y
z

3
2
, S
z
=
x
2
y
2

x
y
+
2z
x

3
2
C 2 trng hp xy ra
+, Nu x y z, khi ta c S
x
0, li c
S
y
+ S
z
=
z
2
x
2
+
z
x
+
2y
z
+
x
2
y
2

x
y
3
z
x
+
2y
z
+
x
2
y
2

x
y
3

y
x
+
x
2
y
2

x
y
1 =
(x + y)(x y)
2
xy
2
0
S
z
+ 2S
y
=
x
2
y
2

x
y
+
2z
2
x
2
+
4y
z

9
2

x
2
y
2

x
y
+
2y
2
x
2

1
2
0
www.VNMATH.com
51
S
x
+ 2S
y
=
y
2
z
2
+
3y
z
+
2x
y
+
2z
2
x
2

2z
x

9
2

2x
y
+
2y
2
x
2

2y
x

1
2
0
+, Nu z y x, khi ta c
S
y
=
z
2
x
2

z
x
+
2y
z

3
2

z
2
x
2

z
x
+
2x
z

3
2
0
S
y
+ S
z
=
z
2
x
2
+
z
x
+
2y
z
+
x
2
y
2

x
y
3
z
2
x
2
+
2y
z
3
z
2
y
2
+
2y
z
3 0
S
x
+ S
y
=
z
2
x
2
+
y
2
z
2
+
y
z
+
2x
y

z
x
3
z
x
+
y
2
z
2
+
y
z
+
2x
y
4
=
yz
3
+ 2(x
2
2xy)z
2
+ xy
2
z + xy
3
xyz
2

y
4
+ 2(x
2
2xy)y
2
+ xy
3
+ xy
3
xyz
2
=
y(x
2
+ (x y)
2
)
xz
2
0
Bt ng thc c chng minh xong. ng thc xy ra khi v ch khi (a, b, c) (1, 1, 1) hoc
(a, b, c) (1, 0, 0).

39 Cho cc s dng x, y, z, t tha


1
x + 1
+
1
y + 1
+
1
z + 1
+
1
t + 1
= 1
Chng minh rng
min
_
1
x
+
1
y
+
1
z
,
1
y
+
1
z
+
1
t
,
1
z
+
1
t
+
1
x
,
1
t
+
1
x
+
1
y
_
1
max
_
1
x
+
1
y
+
1
z
,
1
y
+
1
z
+
1
t
,
1
z
+
1
t
+
1
x
,
1
t
+
1
x
+
1
y
_
Li gii. t a =
1
x+1
, b =
1
y+1
, c =
1
z+1
, d =
1
t+1
, th th ta c 1 a, b, c, d 0 v a + b + c + d = 1.
Khng mt tnh tng qut, gi s a b c d, bt ng thc tng ng vi
b
1 b
+
c
1 c
+
d
1 d
1
a
1 a
+
b
1 b
+
c
1 c
Hay
b
a + c + d
+
c
a + b + d
+
d
a + b + c
1
a
b + c + d
+
b
c + d + a
+
c
d + a + b
Do a b c d nn
b
a + c + d

b
b + c + d
,
c
a + b + d

c
b + c + d
,
d
a + b + c

d
b + c + d
Suy ra
b
a + c + d
+
c
a + b + d
+
d
a + b + c
1
Tng t, ta c
a
b + c + d

a
a + b + c
,
b
c + d + a

b
a + b + c
,
c
d + a + b

c
a + b + c
Do
a
b + c + d
+
b
c + d + a
+
c
d + a + b
1
Bt ng thc c chng minh xong.

www.VNMATH.com
52 CHNG 2. SOLUTION
40 Cho cc s khng m a, b, c, chng minh bt ng thc
a
2

4a
2
+ ab + 4b
2
+
b
2

4b
2
+ bc + 4c
2
+
c
2

4c
2
+ ca + 4a
2

a + b + c
3
Li gii. Vi mi x 0, ta c
6x
2

4x
2
+ x + 4
3x 1
Tht vy, nu x
1
3
, bt ng thc l hin nhin. Nu x
1
3
, ta c
36x
4
4x
2
+ x + 4
(3x 1)
2
=
(x 1)
2
(15x 4)
4x
2
+ x + 4
0
S dng bt ng thc trn, ln lt thay x bi
a
b
,
b
c
,
c
a
, ta c
6a
2

4a
2
+ ab + 4b
2
3a b,
6b
2

4b
2
+ bc + 4c
2
3b c,
6c
2

4c
2
+ ca + 4a
2
3c a
Cng ln lt v vi v 3 bt ng thc trn, ta c pcm. ng thc xy ra khi v ch khi a = b = c.

41 Cho cc s dng a, b, c, chng minh bt ng thc


a(b + c)
a
2
+ bc
+
b(c + a)
b
2
+ ca
+
c(a + b)
c
2
+ ab

1
2

(a + b + c)
_
1
a
+
1
b
+
1
c
_
+ 27
Li gii. Bt ng thc tng ng
4

cyc
a
2
(b + c)
2
(a
2
+ bc)
2
+ 8

cyc
ab(a + c)(b + c)
(a
2
+ bc)(b
2
+ ca)
27 + (a + b + c)
_
1
a
+
1
b
+
1
c
_
Hay
4

cyc
a
2
(b + c)
2
(a
2
+ bc)
2
+ 8

cyc
ab(a + c)(b + c)
(a
2
+ bc)(b
2
+ ca)
24 +

cyc
(b + c)
2
bc

cyc
(b + c)
2
(a
2
bc)
2
bc(a
2
+ bc)
2
+ 8

cyc
c(a b)
2
(a + b)
(a
2
+ bc)(b
2
+ ca)
0
Bt ng thc c chng minh xong. ng thc xy ra khi v ch khi a = b = c.

42 Cho cc s khng m a, b, c tha a + b + c = 1, chng minh bt ng thc


a

a + 2b
+
b

b + 2c
+
c

c + 2a

_
3
2
Li gii. Xt 2 trng hp
Trng hp 1. c b a, khi s dng bt ng thc Cauchy Schwarz, ta c
_

cyc
a

a + 2b
_
2
(a + b + c)

cyc
a
a + 2b
=

cyc
a
a + 2b
www.VNMATH.com
53
Ta cn chng minh

cyc
a
a + 2b

3
2
t x =
b
a
, y =
c
b
, z =
a
c
th ta c x, y 1 z v xyz = 1, bt ng thc tng ng
1
2x + 1
+
1
2y + 1
+
1
2z + 1

3
2
Do x, y 1 nn
1
2x + 1
+
1
2y + 1
=
1
2xy + 1
+
1
3

2(x 1)(y 1)(4xy 1)
3(2x + 1)(2y + 1)(2xy + 1)

1
2xy + 1
+
1
3
=
z
z + 2
+
1
3
Ta phi chng minh
z
z + 2
+
1
2z + 1

7
6
Hay
2z
2
+ 23z + 2
6(z + 2)(2z + 1)
0 (ng)
Trng hp 2. a b c, xt 2 kh nng
Kh nng 2.1. a 4b, khi ta s chng minh
a

a + 2b

a + c

a + c + 2b
,
b

b + 2c
+
c

c + 2a

b
Tht vy, ta c
(a + c)
2
a + c + 2b

a
2
a + 2b
=
c(a
2
+ 4ab + ac + 2bc)
(a + 2b + c)(a + 2b)
0
b

b + 2c
+
c

c + 2a

b + 2c
+
c

c + 8b
Ta phi chng minh
b

b + 2c
+
c

c + 8b

b
Hay
b
2
b + 2c
+
c
2
8b + c
+
2bc
_
(b + 2c)(8b + c)
b
2b
b + 2c

c
8b + c

2b
_
(b + 2c)(8b + c)
224b
4
36b
3
c 71b
2
c
2
4bc
3
+ 4c
4
(b + 2c)
2
(8b + c)
2
0 (ng)
Kh nng 2.2. 4b a, khi ta s chng minh
a

a + 2b

a +
c
2
_
a + 2b +
3c
2
,
b

b + 2c
+
c

c + 2a

_
b +
c
2
Tht vy, ta c
_
a +
c
2
_
2
a + 2b +
3c
2

a
2
a + 2b
=
c(2a(4b a) + c(a + 2b))
2(a + 2b)(2a + 4b + 3c)
0
b

b + 2c
+
c

c + 2a

b + 2c
+
c

c + 2b
www.VNMATH.com
54 CHNG 2. SOLUTION
Ta phi chng minh
b

b + 2c
+
c

c + 2b

_
b +
c
2
Hay
b
2
b + 2c
+
c
2
2b + c
+
2bc
_
(2b + c)(b + 2c)
b +
c
2
2b
b + 2c
+
1
2

2b
_
(2b + c)(b + 2c)
+
c
2b + c
Do b c nn
c
2b + c

1
3
<
1
2
,
2b
_
(2b + c)(b + 2c)

2b
b + 2c
Nh vy, trong mi kh nng, ta ch cn xt bt ng thc trong trng hp c = 0, khi bt ng
thc tr thnh
a

a + 2b
+

b
_
3
2
Hay
1 b

1 + b

_
3
2

b
(1 b)
2
1 + b

_
_
3
2

b
_
2
1 2

6b + 9b 2b

6b
2(b + 1)
0 (ng)
Bt ng thc c chng minh xong. ng thc khng xy ra.
Nhn xt. Ngoi ra, bng cch s dng bt ng thc Holder, ta tm c kt qu sau
a

a + 2b
+
b

b + 2c
+
c

c + 2a
1

43 Cho cc s khng m a, b, c, tm hng s k tt nht bt ng thc sau ng


a
b + c
+
b
c + a
+
c
a + b

3
2
+
k max{(a b)
2
, (b c)
2
, (c a)
2
}
ab + bc + ca
Li gii. Khng mt tnh tng qut, gi s a b c, bt ng thc tr thnh

cyc
a
b + c

3
2
+
k(a c)
2
ab + bc + ca
Cho c = 0, b = 1, a =
4
3
, ta suy ra c k
7
16
, ta s chng minh l gi tr cn tm, tc l

cyc
a
b + c

3
2
+
7(a c)
2
16(ab + bc + ca)
Hay

cyc
a(a(b + c) + bc)
b + c

3
2
(ab + bc + ca) +
7
16
(a c)
2
a
2
+ b
2
+ c
2
+ abc

cyc
1
b + c

3
2
(ab + bc + ca) +
7
16
(a c)
2
www.VNMATH.com
55
S dng bt ng thc Cauchy Schwarz, ta c

cyc
1
b + c

9
2(a + b + c)
Ta cn chng minh
a
2
+ b
2
+ c
2
+
9abc
2(a + b + c)

3
2
(ab + bc + ca) +
7
16
(a c)
2
t a = c + x, b = c + y th ta c x y 0, bng bin i tng ng, ta c th a bt ng
thc v
(11x
2
32xy + 32y
2
)c + (x + y)(3x 4y)
2
0
Bt ng thc ny hin nhin ng, vy ta c pcm, do
k
max
=
7
16
.

44 Cho cc s khng m a, b, c, chng minh bt ng thc


_
a
a + b
_
3
+
_
b
b + c
_
3
+
_
c
c + a
_
3

3
8

_
a
2
+ b
2
+ c
2
ab + bc + ca
_
2
Li gii. Trc ht, ta chng minh rng vi mi x, y, z > 0, xyz = 1 th

cyc
1
(1 + x)
3
+
5
(1 + x)(1 + y)(1 + z)
1
Tht vy, t m =
1x
1+x
, n =
1y
1+y
, p =
1z
1+z
th ta c m, n, p [1, 1] v (1 m)(1 n)(1 p) =
(1 + m)(1 + n)(1 + p), suy ra
m + n + p + mnp = 0
t q = mn + np + pm, r = mnp th |r| 1, ta c
m
2
n
2
p
2
= (m + n + p)
2
= m
2
+ n
2
+ p
2
+ 2q
Suy ra
2q = m
2
(n
2
p
2
1) n
2
p
2
0
Mt khc, ta li c
2q = m
2
(n
2
p
2
1) n
2
p
2
(n
2
p
2
1) n
2
p
2
= n
2
(p
2
1) p
2
1 (p
2
1) p
2
1 = 2
Do q [1, 0], ta c bt ng thc tng ng
r
3
+ 3r
2
q(1 + 3r) 0
Nu r
1
3
th ta c pcm, xt r
1
3
, khi , ta c
r
3
+ 3r
2
q(1 + 3r) r
3
+ 3r
2
+ (1 + 3r) = (1 + r)
3
0
Bt ng thc c chng minh xong, s dng kt qu ny vi x =
a
b
, y =
b
c
, z =
c
a
, ta suy ra c

cyc
b
3
(a + b)
3
1
5abc
(a + b)(b + c)(c + a)
www.VNMATH.com
56 CHNG 2. SOLUTION
Tr li bi ton ca ta, bt ng thc c vit li nh sau

cyc
_
1
a
3
(a + b)
3
_
+
3(a
2
+ b
2
+ c
2
)
2
8(ab + bc + ca)
2
3
Hay

cyc
b
3
(a + b)
3
+ 3

cyc
ab
(a + b)
2
+
3(a
2
+ b
2
+ c
2
)
2
8(ab + bc + ca)
2
3
S dng kt qu trn, ta cn chng minh
3

cyc
ab
(a + b)
2
+
3(a
2
+ b
2
+ c
2
)
2
8(ab + bc + ca)
2
2 +
5abc
(a + b)(b + c)(c + a)
Khng mt tnh tng qut, gi s a + b + c = 1, t ab + bc + ca =
1q
2
3
, r = abc (1 q 0), khi
ta c r max
_
0,
(1+q)
2
(12q)
27
_
, bt ng thc tng ng
f(r) =
3
8
_
1 + 2q
2
1 q
2
_
2
+
108r
2
+ (15 + 20q
2
)r (1 q
2
)
2
(1 + q
2
)
(1 q
2
3r)
2
0
Ta c
f

(r) =
3((87 60q
2
)r + (1 q
2
)(2q
4
+ 4q
2
+ 3))
(1 q
2
3r)
3
0
Do f(r) l hm ng bin, suy ra
Nu 2q 1 th f(r) f(0) =
3
8
_
1+2q
2
1q
2
_
2
0, nu 1 2q th
f(r) f
_
(1 + q)
2
(1 2q)
27
_
=
3q
2
(14(2 10q + 13q
2
) + 124q
2
(1 2q) + q
2
(1 + 52q
2
+ 10q
3
) + 18q
5
(1 q))
8(1 q
2
)
2
(2 q)
4
0
Bt ng thc c chng minh xong. ng thc xy ra khi v ch khi a = b = c.

45 Cho a, b, c, d l cc s dng tha mn a, b, c 1 v abcd = 1, chng minh rng


1
(a
2
a + 1)
2
+
1
(b
2
b + 1)
2
+
1
(c
2
c + 1)
2
+
1
(d
2
d + 1)
2
4
Li gii. Ta c
1
(x
2
x + 1)
2
+
1
(y
2
y + 1)
2
1 +
1
(x
2
y
2
xy + 1)
2
vi mi x, y 1. Tht vy, bt ng thc tng ng
_
1
1
(x
2
x + 1)
2
__
1
1
(y
2
y + 1)
2
_

1
(x
2
x + 1)
2
(y
2
y + 1)
2

1
(x
2
y
2
xy + 1)
2
Hay
xy(x 1)(y 1)(x
2
x + 2)(y
2
y + 2)(x
2
y
2
xy + 1)
2
(x 1)(y 1)(x + y)[2x
2
y
2
xy(x + y) + x
2
+ y
2
x y + 2]
www.VNMATH.com
57
xy(x
2
x + 2)(y
2
y + 2)(x
2
y
2
xy + 1)
2
(x + y)[2x
2
y
2
xy(x + y) + x
2
+ y
2
x y + 2]
Do x, y 1 nn
(x
2
x + 2)(y
2
y + 2)(x
2
y
2
xy + 1) 4, 2xy x + y
Do
xy(x
2
x + 2)(y
2
y + 2)(x
2
y
2
xy + 1)
2
2(x + y)(x
2
y
2
xy + 1)
Ta cn chng minh
2(x
2
y
2
xy + 1) 2x
2
y
2
xy(x + y) + x
2
+ y
2
x y + 2
Hay
(x 1)(y 1)(x + y) 0 (ng)
S dng kt qu trn ln lt vi a, b, c, ta c
1
(a
2
a + 1)
2
+
1
(b
2
b + 1)
2
1 +
1
(a
2
b
2
ab + 1)
2
1
(a
2
b
2
ab + 1)
2
+
1
(c
2
c + 1)
2
1 +
1
(a
2
b
2
c
2
abc + 1)
2
= 1 +
d
4
(d
2
d + 1)
2
Do
1
(a
2
a + 1)
2
+
1
(b
2
b + 1)
2
+
1
(c
2
c + 1)
2
2 +
d
4
(d
2
d + 1)
2
Mt khc, ta li c
d
4
(d
2
d + 1)
2
+
1
(d
2
d + 1)
2
=
(d 1)
4
(d
2
d + 1)
2
+ 2 2
Bt ng thc c chng minh xong. ng thc xy ra khi v ch khi a = b = c = d = 1.

46 Vi mi s khng m a, b, c, chng minh rng


_
a
2
+ 4bc
b
2
+ c
2
+
_
b
2
+ 4ca
c
2
+ a
2
+
_
c
2
+ 4ab
a
2
+ b
2
2 +

2
Li gii. Khng mt tnh tng qut, gi s a b c 0. Xt 2 trng hp
Trng hp 1. 4b
3
a
2
c, khi ta c
a
2
+ 4bc
b
2
+ c
2

a
2
b
2
=
c(4b
3
a
2
c)
b
2
(b
2
+ c
2
)
0,
b
2
+ 4ca
c
2
+ a
2

b
2
a
2
=
c(4a
3
b
2
c)
a
2
(c
2
+ a
2
)
0
Suy ra

cyc
_
a
2
+ 4bc
b
2
+ c
2

a
b
+
b
a
+ 2
_
ab
a
2
+ b
2
S dng bt ng thc AMGM, ta c
a
b
+
b
a
+ 2
_
ab
a
2
+ b
2
=
_
1
1

2
_
a
2
+ b
2
ab
+
_
a
2
+ b
2

2ab
+ 2
_
ab
a
2
+ b
2
_
2
_
1
1

2
_
+ 2
4
_
2(a
2
+ b
2
)
ab
2
_
1
1

2
_
+ 2

2 = 2 +

2
www.VNMATH.com
58 CHNG 2. SOLUTION
Trng hp 2. 4b
3
a
2
c, suy ra a 2b, ta c
a
2
+ 4bc
b
2
+ c
2

a
2
+ 4b
2
2b
2
=
(b c)(a
2
(b + c) 4b
2
(b c))
2b
2
(b
2
+ c
2
)
0
Suy ra

cyc
_
a
2
+ 4bc
b
2
+ c
2

_
a
2
+ 4b
2
2b
2
+
b
a
+ 2
_
ab
a
2
+ b
2
t x =
a
b
2, ta cn chng minh
f(x) =
_
x
2
2
+ 2 +
1
x
+ 2
_
x
x
2
+ 1
2 +

2
Do x 2 nn x(x
2
+ 1) (x + 1)
2
= x
3
_
1
1
x

1
x
2

1
x
3
_
x
3
_
1
1
2

1
2
2

1
2
3
_
=
1
8
x
3
> 0, suy
ra
_
x(x
2
+ 1) > x + 1, do
f

(x) =
x
2
_
x
2
2
+ 2

1
x
2

x
2
1
(x
2
+ 1)
_
x(x
2
+ 1)
>
1
2
_
1
2
+
2
x
2

1
x
2

x
2
1
(x
2
+ 1)(x + 1)

1
2

1
x
2

x 1
x
2
+ 1
=
x
2
4
4x
2
+
(x 2)
2
+ 1
4(x
2
+ 1)
> 0
Vy f(x) l hm ng bin trn [2, +), do
f(x) f(2) =
5
2
+ 2
_
2
5
> 2 +

2
Bt ng thc c chng minh. ng thc xy ra khi v ch khi (a, b, c) (1, 1, 0).

47 Cho cc s khng m a, b, c, chng minh bt ng thc


(a b)(13a + 5b)
a
2
+ b
2
+
(b c)(13b + 5c)
b
2
+ c
2
+
(c a)(13c + 5a)
c
2
+ a
2
0
Li gii. Bt ng thc c vit li nh sau

cyc
13a
2
8ab 5b
2
a
2
+ b
2
0
Hay
4

cyc
(a b)
2
a
2
+ b
2
+ 9

cyc
a
2
b
2
a
2
+ b
2
0
Ch rng
_
1 +
a
2
b
2
a
2
+ b
2
__
1 +
b
2
c
2
b
2
+ c
2
__
1 +
c
2
a
2
c
2
+ a
2
_
=
_
1
a
2
b
2
a
2
+ b
2
__
1
b
2
c
2
b
2
+ c
2
__
1
c
2
a
2
c
2
+ a
2
_
Nn

cyc
a
2
b
2
a
2
+ b
2
=
(a
2
b
2
)(b
2
c
2
)(c
2
a
2
)
(a
2
+ b
2
)(b
2
+ c
2
)(c
2
+ a
2
)
Ta cn chng minh
4

cyc
(a b)
2
a
2
+ b
2

9(a
2
b
2
)(b
2
c
2
)(c
2
a
2
)
(a
2
+ b
2
)(b
2
+ c
2
)(c
2
+ a
2
)
www.VNMATH.com
59
S dng bt ng thc AMGM, ta c

cyc
(a b)
2
a
2
+ b
2
3
3

(a b)
2
(b c)
2
(c a)
2
(a
2
+ b
2
)(b
2
+ c
2
)(c
2
+ a
2
)
Ta phi chng minh
4
3

(a b)
2
(b c)
2
(c a)
2
(a
2
+ b
2
)(b
2
+ c
2
)(c
2
+ a
2
)

3(a
2
b
2
)(b
2
c
2
)(c
2
a
2
)
(a
2
+ b
2
)(b
2
+ c
2
)(c
2
+ a
2
)
Hay
64

cyc
(a
2
+ b
2
)
2
27

cyc
(a
2
b
2
)(a + b)
2
Bt ng thc ny l h qu ca bt ng thc sau vi mi x y 0
4(x
2
+ y
2
)
2
3(x
2
y
2
)(x + y)
2
Hay
x
4
6x
3
y + 8x
2
y
2
+ 6xy
3
+ 7y
4
0
+, Nu x 6y th bt ng thc hin nhin ng.
+, Nu x 6y th ta c
x
4
6x
3
y + 8x
2
y
2
+ 6xy
3
+ 7y
4
= x
2
(x 3y)
2
+ xy
2
(6y x) + 7y
4
0
Bt ng thc c chng minh xong. ng thc xy ra khi v ch khi a = b = c.

48 Chng minh rng vi mi s dng a, b, c, n, ta c


_
a
2
+ bc
b + c
_
n
+
_
b
2
+ ca
c + a
_
n
+
_
c
2
+ ab
a + b
_
n
a
n
+ b
n
+ c
n
Li gii. Khng mt tnh tng qut, gi s a b c > 0. Ta c
_
a
2
+ bc
a(b + c)
_
n
1
_
a
2
+ bc
a(b + c)
_
n
+
_
b
2
+ ca
b(c + a)
_
n
2
_
(a
2
+ bc)(b
2
+ ca)
ab(a + c)(b + c)
_
n/2
2
Do
(a
2
+ bc)(b
2
+ ca) ab(a + c)(b + c) = c(a b)
2
(a + b) 0
V do
_
a
2
+ bc
a(b + c)
_
n
+
_
b
2
+ ca
b(c + a)
_
n
+
_
c
2
+ ab
c(a + b)
_
n
3
t x =
_
a
2
+bc
a(b+c)
_
n
1, y =
_
b
2
+ca
b(c+a)
_
n
1, z =
_
c
2
+ab
c(a+b)
_
n
1 th ta c
x 0, x + y 0, x + y + z 0
Do

cyc
_
a
2
+ bc
b + c
_
n

cyc
a
n
= a
n
x + b
n
y + c
n
z
= (a
n
b
n
)x + (b
n
c
n
)(x + y) + c
n
(x + y + z) 0
Bt ng thc c chng minh xong. ng thc xy ra khi v ch khi a = b = c hoc n 0.

www.VNMATH.com
60 CHNG 2. SOLUTION
49 Cho cc s khng m a, b, c tha a + b + c = 1. Ty theo gi tr ca n N, hy tm gi tr ln nht
v gi tr nh nht ca biu thc
P(a, b, c) = a(b c)
n
+ b(c a)
n
+ c(a b)
n
Li gii. Trong trng hp n = 0 v n = 1 th ta c P = 1 v P = 0. Xt n 2, khi c 2 trng hp
Trng hp 1. n l, suy ra n 3, vi gi thit b l s hng nm gia a v c, ta s chng minh
P(a + c, b, 0) P(a, b, c) P(a + c, 0, b)
C 2 kh nng
Kh nng 1. a b c 0, xt hm s g(a) = P(a +c, 0, b) P(a, b, c) = (a +c)
n
b (a +c)b
n

a(b c)
n
b(c a)
n
c(a b)
n
, ta c
g

(a) = (nb(a + c)
n1
b
n
(b c)
n
) + n(b(a c)
n1
c(a b)
n1
) 0
Suy ra g(a) l hm ng bin, do
g(a) g(b) = b(b + c)((b + c)
n1
b
n1
) 0
Xt tip hm s h(a) = P(a, b, c)P(a+c, b, 0) = (a+c)
n
b(a+c)b
n
+a(bc)
n
+b(ca)
n
+c(ab)
n
,
ta c
h

(a) = nb((a + c)
n1
(a c)
n1
) b
n
+ (b c)
n
+ nc(a b)
n1
nb((b + c)
n1
(b c)
n1
) b
n
+ (b c)
n
= 2n
n3
2

i=0
C
2i+1
n1
b
n2i1
c
2i+1

n1
2

i=0
C
2i+1
n
b
n2i1
c
2i+1
+
n1
2

i=1
C
2i
n
b
n2i
c
2i
=
n3
2

i=0
b
n2i1
c
2i+1
(2nC
2i+1
n1
C
2i+1
n
) +
n1
2

i=1
C
2i
n
b
n2i
c
2i
c
n
0
Suy ra h(a) l hm ng bin, do
h(a) h(b) = b(b + c)((b + c)
n1
b
n1
) 0
Kh nng 2. a b c, khi , ta c
P(a, b, c) = a(b c)
n
+ b(c a)
n
+ c(a b)
n
= (c(b a)
n
+ b(a c)
n
+ a(c b)
n
) = P(c, b, a)
Theo trn, ta c
P(c + a, b, 0) P(c, b, a) P(c + a, 0, b)
Do
P(a + c, b, 0) = P(c + a, 0, b) P(a, b, c) P(a + c, b, 0) = P(a + c, 0, b)
Vy trong mi kh nng, ta lun c
P(a + c, b, 0) P(a, b, c) P(a + c, 0, b)
Xt hm s f(x) =
x
n
x
(x+1)
n+1
vi x 0, ta c
f

(x) =
x
n
+ nx
n1
+ nx 1
(x + 1)
n+2
f

(x) = 0 (x) = x
n
nx
n1
nx + 1 = 0
www.VNMATH.com
61
D thy x = 0, x = 1 khng l nghim ca (x) v nu x > 0 (x = 1) l 1 nghim ca (x) th
1
x
cng l nghim ca (x), do , ta ch cn xt nghim ca (x) trn [0, 1] l . Khi , ta c

(x) = (n(1 x
n1
) + n(n 1)x
n2
) 0
Suy ra (t) l hm nghch bin, li c (0) = 1 > 0, (1) = 2(1 n) < 0 nn tn ti duy nht
x
0
(0, 1) sao cho (x
0
) = 0, do phng trnh (x) = 0 ch c 2 nghim dng l x
0
v
1
x
0
. T
y, ta d dng kim tra c
f(x) max
_
f(0), f
_
1
t
0
__
= f
_
1
x
0
_
Suy ra
P(a + c, 0, b) = (a + c)
n
b (a + c)b
n
=
(a + c)
n
b (a + c)b
n
(a + b + c)
n+1
= f
_
a + c
b
_
f
_
1
x
0
_
P(a + c, 0, b) = (a + c)
n
b + (a + c)b
n
= f
_
a + c
b
_
f
_
1
x
0
_
Trng hp 2. n chn, khi d thy min P(a, b, c) = 0 v P(a, b, c) l mt biu thc i xng
vi a, b, c, do khng mt tnh tng qut, ta c th gi s a b c 0. Nu n 4, t
b = t + s, c = t s (t m 0 v xt hm s
(s) = 2
n
as
n
+ (t + s)(a + s t)
n
+ (t s)(t + s a)
n
Ta c

(s) = 2n((a + s t)
n1
(t + s a)
n1
) + n(n 1)((t + s)(a + s t)
n2
+ (t s)(t + s a)
n2
)
+ 2
n
n(n 1)as
n2
= 4n(n 1)a(b c)
n2
+ 2n((a b)
n1
+ (a c)
n1
) + n(n 1)(c(a b)
n2
+ b(a c)
n2
) 0
Suy ra (s) l hm li, do
(s) max{(t), (0)}
Nh vy, ta ch cn xt bi ton trong trng hp (a b)(b c)(c a) = 0 hoc abc = 0.
(i) abc = 0, khng mt tnh tng qut, gi s a b c = 0, ta cn tm gi tr ln nht ca
P(a, b, c) = a
n
b + ab
n
=
a
n
b+ab
n
(a+b)
n+1
=
y
n
+y
(y+1)
n+1
= u(y) vi y =
b
a
1. Ta c
u

(y) =
(1 y)(y
n1
(n 1)y
n2
. . . (n 1)y + 1)
(y + 1)
n+2
T y, ta d dng suy ra c u

(y) = 0 ch c 2 nghim l 1 v y
0
(0, 1) v nh th, ta c th
kim tra c
u(y) u(y
0
)
(ii) (a b)(b c)(c a) = 0, khng mt tnh tng qut, gi s b = c, ta c a + 2b = 1 v
P(a, b, c) = 2b(a b)
n
. Nu a b th P(a, b, c) = 2b(b a)
n
2b
n+1

1
2
n
, nu a b th s dng
bt ng thc AMGM, ta c
P(a, b, c) = 2b(a b)
n
=
2
3
n
n
3b
_
a b
n
_
n

n
n
(a b + 3b)
n+1
(n + 1)
n+1
=
2n
n
3(n + 1)
n+1
Nu n = 2 th ta c P(a, b, c) = a(b c)
2
+ b(c a)
2
+ c(a b)
2
= ab + bc + ca 9abc. Nu
ab + bc + ca
1
4
th hin nhin P(a, b, c)
1
4
, nu ab + bc + ca
1
4
th s dng bt ng thc
Schur, ta c
P(a, b, c) = ab + bc + ca 9abc ab + bc + ca (4(ab + bc + ca) 1) = 1 3(ab + bc + ca)
1
4
www.VNMATH.com
62 CHNG 2. SOLUTION
T y, ta d dng i n kt lun ca bi ton.

50 Cho cc s dng a, b, c tha a + b + c = 3, tm hng s k ln nht sao cho


a
5
+ b
5
+ c
5
3
a
3
+ b
3
+ c
3
3
k
Li gii. Cho a = b = 3

3, c = 2

3 3, ta suy ra c k
5(5

37)

3+1
, ta s chng minh y chnh
l gi tr cn tm, tc l

cyc
_
a
5

5
_
5

3 7
_

3 + 1
a
3
+
24

3 36

3 + 1
_
0
Hay

cyc
(a 1)
2
_
a
3
+ 2a
2
+
38 22

3 + 1
a +
2
_
37 23

3
_

3 + 1
_
0
Hay

cyc
(4M
a
+ M
b
+ M
c
)(a b)(a c) 0
trong
M
a
= a
3
+ 2a
2
+
38 22

3 + 1
a +
2
_
37 23

3
_

3 + 1
M
b
= b
3
+ 2b
2
+
38 22

3 + 1
b +
2
_
37 23

3
_

3 + 1
M
c
= c
3
+ 2c
2
+
38 22

3 + 1
c +
2
_
37 23

3
_

3 + 1
Ta c
4M
a
+ M
b
+ M
c
= 4a
3
+ b
3
+ c
3
+ 2(4a
2
+ b
2
+ c
2
) +
3
_
38 22

3
_

3 + 1
a +
9
_
62 38

3
_

3 + 1
4a
3
+
(b + c)
3
4
+ 8a
2
+ (b + c)
2
+
3
_
38 22

3
_

3 + 1
a +
9
_
62 38

3
_

3 + 1
= 4a
3
+
(3 a)
3
4
+ 8a
2
+ (3 a)
2
+
3
_
38 22

3
_

3 + 1
a +
9
_
62 38

3
_

3 + 1
=
15
_
a + 3 2

3
_
2
_
a 3 + 4

3
_
4
0
Tng t, ta c 4M
b
+M
c
+M
a
, 4M
c
+M
a
+M
b
0. Khng mt tnh tng qut, gi s a b c > 0,
suy ra a + b 2, ta c
(4M
a
+ M
b
+ M
c
) (4M
b
+ M
c
+ M
a
) = 3(M
a
M
b
) = 3(a
3
b
3
) + 6(a b)
_
a + b +
19 11

3 + 1
_
3(a
3
b
3
) + 6(a b)
_
2 +
19 11

3 + 1
_
0
www.VNMATH.com
63
Do ,

cyc
(4M
a
+ M
b
+ M
c
)(a b)(a c) =(a b)((4M
a
+ M
b
+ M
c
)(a c) (4M
b
+ M
c
+ M
a
)(b c))
+ (4M
c
+ M
a
+ M
b
)(a c)(b c) 0
Bt ng thc c chng minh xong. Vy
k
max
=
5
_
5

3 7
_

3 + 1
.

51 [Nguyn Phi Hng] Cho cc s khng m a, b, c tha a


2
+ b
2
+ c
2
= 8, chng minh bt ng thc
4(a + b + c 4) abc
Li gii. t x = a +b +c, y = ab +bc +ca th ta c x
2
2y = 8. S dng bt ng thc Schur bc 4,
ta c
abc
(4y x
2
)(x
2
y)
6x
=
(x
2
16)(x
2
+ 8)
12x
Ta cn chng minh
(x
2
16)(x
2
+ 8)
12x
4(x 4)
Hay
(x 4)
2
(x
2
+ 8x 8)
12x
0
Bt ng thc ny hin nhin ng, vy ta c pcm. ng thc xy ra khi v ch khi a = b = 2, c = 0
v cc hon v.

52 Cho m, n (3n
2
> m
2
) l cc s thc cho trc v a, b, c l cc s thc tha mn a + b + c =
m, a
2
+ b
2
+ c
2
= n
2
. Tm gi tr ln nht v gi tr nh nht ca biu thc sau
P = a
2
b + b
2
c + c
2
a
Li gii. t a = x +
m
3
, b = y +
m
3
, c = z +
m
3
, th th iu kin bi ton cho ta x + y + z = 0 v
x
2
+ y
2
+ z
2
=
3n
2
m
2
3
. Biu thc P tr thnh
P = x
2
y + y
2
z + z
2
x +
m
3
9
Ta c

cyc
_
3x
_
2
3n
2
m
2

18
3n
2
m
2
xy 1
_
2
= 3 +
18
3n
2
m
2
_

cyc
x
_
2
+
324
(3n
2
m
2
)
2

cyc
x
2
y
2
6
_
2
3n
2
m
2

cyc
x 54
_
2
3n
2
m
2
_
3/2

cyc
x
2
y
= 3 +
324
(3n
2
m
2
)
2

cyc
x
2
y
2
54
_
2
3n
2
m
2
_
3/2

cyc
x
2
y
Do x + y + z = 0 nn xy + yz + zx =
1
2
(x
2
+ y
2
+ z
2
) =
3n
2
m
2
6
, suy ra

cyc
x
2
y
2
=
_

cyc
xy
_
2
2xyz

cyc
x =
_

cyc
xy
_
2
=
(3n
2
m
2
)
2
36
www.VNMATH.com
64 CHNG 2. SOLUTION
Do
12 54
_
2
3n
2
m
2
_
3/2

cyc
x
2
y 0
Suy ra

cyc
x
2
y
2
9
_
3n
2
m
2
2
_
3/2
Vy
P
2
9
_
3n
2
m
2
2
_
3/2
+
m
3
9
Mt khc, cho x =

2(3n
2
m
2
)
3
cos
2
9
, y =

2(3n
2
m
2
)
3
cos
4
9
, z =

2(3n
2
m
2
)
3
cos
8
9
, ta c
P =
2
9
_
3n
2
m
2
2
_
3/2
+
m
3
9
Vy
max P =
2
9
_
3n
2
m
2
2
_
3/2
+
m
3
9
Hon ton tng t, bng cch xt biu thc

cyc
_
3x
_
2
3n
2
m
2
+
18
3n
2
m
2
xy + 1
_
2
, ta d dng
suy ra c
min P =
2
9
_
3n
2
m
2
2
_
3/2

m
3
9
Bi ton c gii quyt hon ton.

53 Tm hng s k nh nht sao cho vi mi a, b, c 0 th

a
3
ka
2
+ (b + c)
2
+

b
3
kb
2
+ (c + a)
2
+

c
3
kc
2
+ (a + b)
2

_
3(a + b + c)
k + 4
Li gii. Cho a = b = 1, c = 0, suy ra k 5. Ta s chng minh y l gi tr cn tm, tc l

cyc

a
3
5a
2
+ (b + c)
2

_
a + b + c
3
S dng bt ng thc Cauchy Schwarz, ta c
_

cyc

a
3
5a
2
+ (b + c)
2
_2

cyc
a
__

cyc
a
2
5a
2
+ (b + c)
2
_
Ta cn chng minh

cyc
a
2
5a
2
+ (b + c)
2

1
3
Khng mt tnh tng qut, gi s a +b +c = 1 v a b c 0, suy ra a
1
3
c. Bt ng thc
tr thnh

cyc
a
2
6a
2
2a + 1

1
3
www.VNMATH.com
65
Xt 2 trng hp
+, Nu c
1
8
, ta c
9

cyc
27a
2
6a
2
2a + 1
=

cyc
_
12a 1
27a
2
6a
2
2a + 1
_
=

cyc
(3a 1)
2
(8a 1)
6a
2
2a + 1
0
+, Nu c
1
8
, ta c
6(V T V P) =
2a 1
6a
2
2a + 1
+
2b 1
6b
2
2b + 1
+
6c
2
6c
2
2c + 1
=
a b c
6a
2
2a + 1
+
b c a
6b
2
2b + 1
+
6c
2
6c
2
2c + 1
=
2(a b)
2
(3c 2)
(6a
2
2a + 1)(6b
2
2b + 1)
+ c
_
6c
6c
2
2c + 1

1
6a
2
2a + 1

1
6b
2
2b + 1
_
Ta phi chng minh
1
6a
2
2a + 1
+
1
6b
2
2b + 1

6c
6c
2
2c + 1
Do c
1
8
nn
6c
6c
2
2c+1
1, suy ra ta ch cn chng minh
1
6a
2
2a + 1
+
1
6b
2
2b + 1
1
++, Nu b
1
3
th
1
6b
2
2b + 1
1
++, Nu b
1
3
, s dng bt ng thc Cauchy Schwarz, ta ch cn chng minh
4 6(a
2
+ b
2
) 2(a + b) + 2
Hay
(2(a + b) + c)(a + b + c) 3(a
2
+ b
2
)
Do b
1
3
nn 3b a, do
(2(a + b) + c)(a + b + c) 2(a + b)
2
= 3(a
2
+ b
2
) + 4ab a
2
b
2
3(a
2
+ b
2
) + 3ab a
2
3(a
2
+ b
2
)
Bt ng thc c chng minh xong. Vy
k
min
= 5.

54 Chng minh rng nu a, b, c > 0 v a + b + c = 3 th


(ab + bc + ca)
_
a
b
2
+ 9
+
b
c
2
+ 9
+
c
a
2
+ 9
_

9
10
Li gii. Bt ng thc tng ng vi

cyc
a
b
2
+ 9

9
10(ab + bc + ca)
www.VNMATH.com
66 CHNG 2. SOLUTION
Hay
9
10(ab + bc + ca)
+
1
9

cyc
ab
2
b
2
+ 9

1
3
S dng bt ng thc Cauchy Schwarz, ta c

cyc
ab
2
b
2
+ 9

(ab + bc + ca)
2
ab
2
+ bc
2
+ ca
2
+ 27
Ta cn chng minh
9
10(ab + bc + ca)
+
(ab + bc + ca)
2
9(ab
2
+ bc
2
+ ca
2
+ 27)

1
3
S dng bt ng thc AMGM,
2
5(ab + bc + ca)
+
(ab + bc + ca)
2
9(ab
2
+ bc
2
+ ca
2
+ 27)
5
5

1
9 10
4
(ab + bc + ca)
2
(ab
2
+ bc
2
+ ca
2
+ 27)
Li s dng bt ng thc Cauchy Schwarz v bt ng thc AMGM, ta c
(ab + bc + ca)
4
(ab
2
+ bc
2
+ ca
2
)
2
(ab + bc + ca)
4
(a
2
b
2
+ b
2
c
2
+ c
2
a
2
)(a
2
+ b
2
+ c
2
)
27(ab + bc + ca)
2
(a
2
b
2
+ b
2
c
2
+ c
2
a
2
)
t x = ab +bc +ca, theo bt ng thc AMGM v bt ng thc Schur, x 3, abc
4x9
3
. Suy
ra
(ab + bc + ca)
2
(a
2
b
2
+ b
2
c
2
+ c
2
a
2
) = x
2
(x
2
6abc) x
2
(x
2
8x + 18)
= (x 3)
3
(x + 1) + 27 27
Do
(ab + bc + ca)
2
(ab
2
+ bc
2
+ ca
2
) 27
Suy ra
2
5(ab + bc + ca)
+
(ab + bc + ca)
2
9(ab
2
+ bc
2
+ ca
2
+ 27)

1
6
V nh th
9
10(ab + bc + ca)
+
(ab + bc + ca)
2
9(ab
2
+ bc
2
+ ca
2
+ 27)

1
3
Bt ng thc c chng minh xong. ng thc xy ra khi v ch khi a = b = c = 1.

55 Cho cc s dng a, b, c tha a + b + c = 3, chng minh bt ng thc


ab

c
2
+ 3
+
bc

a
2
+ 3
+
ca

b
2
+ 3

3
2
Li gii. S dng bt ng thc Cauchy Schwarz, ta cn chng minh
_

cyc
ab
__

cyc
ab
c
2
+ 3
_

9
4
Khng mt tnh tng qut, gi s a b c > 0. t a + b = 2t, a b = 2m, suy ra
3
2
> t 1, c =
3 2t, xt hm s
f(m) =
t
2
m
2
3 + c
2
+
c(t + m)
3 + (t m)
2
+
c(t m)
3 + (t + m)
2
www.VNMATH.com
67
Ta c
f

(m) = 2c(t
2
m
2
)
_
1
(3 + (t m)
2
)
2

1
(3 + (t + m)
2
)
2
_

2m
3 + c
2
+ c
_
1
3 + (t m)
2

1
3 + (t + m)
2
_
=
c(a
2
b
2
)
(3 + a
2
)(3 + b
2
)
+
2abc(a
2
b
2
)(a
2
+ b
2
+ 6)
(3 + a
2
)
2
(3 + b
2
)
2

a b
3 + c
2
= (a b)
_
c(a + b)
(3 + a
2
)(3 + b
2
)
+
2abc(a + b)(a
2
+ b
2
+ 6)
(3 + a
2
)
2
(3 + b
2
)
2

1
3 + c
2
_
Ta s chng minh f

(m) 0, hay
c(a + b)
(3 + a
2
)(3 + b
2
)
+
2abc(a + b)(a
2
+ b
2
+ 6)
(3 + a
2
)
2
(3 + b
2
)
2

1
3 + c
2
Ch rng (3 +a
2
)(3 +b
2
)
_
3 + t
2
_
2
do a +b < 3 v ab(a
2
+b
2
) 2t
4
. Do , ta ch cn chng
minh
2tc
(3 + t
2
)
2
+
8t
3
c
(3 + t
2
)
3

1
3 + c
2
Hay
2t(3 2t)
(3 + t
2
)
2
+
8t
3
(3 2t)
(3 + t
2
)
3

1
3 + (3 2t)
2
9(t 1)(9t
5
31t
4
+ 42t
3
22t
2
+ 21t 3) 0
Bt ng thc ny ng do
3
2
> t 1. Do , f(m) l hm khng tng, suy ra
f(m) f(0) =
t
2
3 + c
2
+
2tc
3 + t
2
Mt khc, d thy ab + bc + ca t(t + 2c). Ta cn phi chng minh
t(t + 2c)
_
t
2
3 + c
2
+
2tc
3 + t
2
_

9
4
Hay
t(t + 2(3 2t))
_
t
2
3 + (3 2t)
2
+
2t(3 2t)
3 + t
2
_

9
4
(t 1)
2
(5t
4
24t
3
+ 33t
2
9t 9) 0
Bt ng thc ng do
3
2
> t 1.
Vy ta c pcm. ng thc xy ra khi v ch khi a = b = c = 1.

56 Chng minh rng vi mi a, b, c dng th


_
b + c
a
+
_
c + a
b
+
_
a + b
c

16(a + b + c)
3
3(a + b)(b + c)(c + a)
Li gii. S dng bt ng thc Holder, ta c
_

cyc
_
b + c
a
_
2
_

cyc
1
a
2
(b + c)
_

cyc
1
a
_
3
www.VNMATH.com
68 CHNG 2. SOLUTION
Do , ta cn chng minh
_

cyc
1
a
_
3

16(a + b + c)
3
3(a + b)(b + c)(c + a)

cyc
1
a
2
(b + c)
t x =
1
a
, y =
1
b
, z =
1
c
, bt ng thc tr thnh
(x + y + z)
3

16(xy + yz + zx)
3
3(x + y)(y + z)(z + x)

cyc
x
y + z
S dng bt ng thc AMGM, ta c
(x + y)(y + z)(z + x) = (x + y + z)(xy + yz + zx) xyz
8
9
(x + y + z)(xy + yz + zx)
Ta phi chng minh
(x + y + z)
4
xy + yz + zx
6(xy + yz + zx)

cyc
x
y + z
Hay
(x + y + z)
4
xy + yz + zx
6(x
2
+ y
2
+ z
2
) + 6xyz

cyc
1
y + z
Li s dng bt ng thc AMGM,
4
y + z

1
y
+
1
z
,
4
z + x

1
z
+
1
x
,
4
x + y

1
x
+
1
y
Suy ra

cyc
1
y + z

1
2
_
1
x
+
1
y
+
1
z
_
=
xy + yz + zx
2xyz
Ta cn chng minh
(x + y + z)
4
xy + yz + zx
6(x
2
+ y
2
+ z
2
) + 3(xy + yz + zx)
Hay
(x + y + z)
4
xy + yz + zx
6(x + y + z)
2
9(xy + yz + zx)
((x + y + z)
2
3(xy + yz + zx))
2
xy + yz + zx
0
Bt ng thc c chng minh xong. ng thc xy ra khi v ch khi a = b = c.

57 Tm hng s k ln nht sao cho bt ng thc sau ng


1
a(1 + bc)
2
+
1
b(1 + ca)
2
+
1
c(1 + ab)
2

k
(1 + ab)(1 + bc)(1 + ca)
+
3
4

k
8
trong a, b, c l cc s dng tha abc = 1.
www.VNMATH.com
69
Li gii. Cho a = 2, b = 1, c =
1
2
, ta c k 4. Ta s chng minh
4

cyc
1
a(1 + bc)
2
1 +
16
(1 + ab)(1 + bc)(1 + ca)
Hay
4

cyc
a
(a + 1)
2
1 +
16
(1 + a)(1 + b)(1 + c)
t x =
1a
1+a
, y =
1b
1+b
, z =
1c
1+c
, th x, y, z [1, 1] v
(1 x)(1 y)(1 z) = (1 + x)(1 + y)(1 + z)
Suy ra
x + y + z + xyz = 0
Bt ng thc tr thnh

cyc
(1 x
2
) 1 + 2(1 + x)(1 + y)(1 + z)
Hay
x
2
+ y
2
+ z
2
+ 2(xy + yz + zx) + 2(x + y + z + xyz) 0
(x + y + z)
2
0
Vy ta c pcm, do
k
max
= 4.

58 Cho cc s khng m a, b, c, chng minh bt ng thc sau vi k =


ln 3
ln 3ln 2
_
a
2
b
2
+ bc + c
2
_
1/k
+
_
b
2
c
2
+ ca + a
2
_
1/k
+
_
c
2
a
2
+ ab + b
2
_
1/k
2
Li gii. S dng bt ng thc Holder, ta c
_

cyc
_
a
2
b
2
+ bc + c
2
_
1/k
_
k
_

cyc
a(b
2
+ bc + c
2
)
_
(a
3/(k+1)
+ b
3/(k+1)
+ c
3/(k+1)
)
k+1
Ta phi chng minh
(a
3/(k+1)
+ b
3/(k+1)
+ c
3/(k+1)
)
k+1
2
k
(a + b + c)(ab + bc + ca)
Khng mt tnh tng qut, gi s a b c, t a = t + m, b = t m vi t m + c, m 0, xt
hm s
f(m) = (k + 1) ln ((t + m)
3/(k+1)
+ (t m)
3/(k+1)
+ c
3/(k+1)
) ln (t
2
+ 2tc m
2
)
Ta c
f

(m) =
3((t + m)
(2k)/(k+1)
(t m)
(2k)/(k+1)
)
(t + m)
3/(k+1)
+ (t m)
3/(k+1)
+ c
3/(k+1)
+
2m
t
2
+ 2tc m
2
=
3(a
(2k)/(k+1)
b
(2k)/(k+1)
)
a
3/(k+1)
+ b
3/(k+1)
+ c
3/(k+1)
+
a b
ab + bc + ca
www.VNMATH.com
70 CHNG 2. SOLUTION
Ta s chng minh f

(m) 0, t a = x
k+1
, b = y
k+1
, c = z
k+1
(x y z 0), ta phi chng
minh
x
k2
y
k2
(x
k+1
y
k+1
)
x
k+1
y
k+1
+ y
k+1
z
k+1
+ z
k+1
x
k+1

3(x
k2
y
k2
)
x
3
+ y
3
+ z
3
0
D thy rng do x y v k > 2 nn x
k+1
y
k+1

k+1
k2
y
3
(x
k2
y
k2
), nh th ta cn chng
minh
k + 1
k 2
x
k2
y
k+1
(x
3
+ y
3
+ z
3
) 3(x
k+1
y
k+1
+ y
k+1
z
k+1
+ z
k+1
x
k+1
)
Hay
7 2k
k 2
x
k+1
y
k+1
3x
k+1
z
k+1
+
k + 1
k 2
x
k2
y
k+4
+
_
k + 1
k 2
x
k2
3z
k2
_
y
k+1
z
3
0
Nh th, nu
k+1
k2
x
k2
y
k+4
x
k+1
z
k+1
th do x y v
72k
k2
> 2 nn bt ng thc ng.
Xt trng hp ngc li,
k+1
k2
x
k2
y
k+4
x
k+1
z
k+1
, suy ra x
3
_
k+1
k2
y
3

4y, khi ta c
x
k+1
y
k+1
6y
3
(x
k2
y
k2
), vy nn ta cn chng minh
2x
k2
y
k+1
(x
3
+ y
3
+ z
3
) (x
k+1
y
k+1
+ y
k+1
z
k+1
+ z
k+1
x
k+1
)
Hay
x
k+1
(y
k+1
z
k+1
) + 2x
k2
y
k+4
+ (2x
k2
z
k2
)y
k+1
z
3
0 (ng)
Vy ta lun c f

(m) 0, v nh vy, ta ch cn xt bt ng thc cho trong trng hp


a = b c l , tc l ta phi chng minh
(2a
3/(k+1)
+ c
3/(k+1)
)
k+1
2
k
a(a + 2c)(2a + c)
t u =
_
c
a
_
1/(k+1)
1, ta cn chng minh
g(u) =
(u
3
+ 2)
k+1
(u
k+1
+ 2)(2u
k+1
+ 1)
2
k
C th d dng kim tra c bt ng thc trn. Vy ta c pcm.

59 Cho cc s khng m a, b, c chng minh bt ng thc


_
a
2
+ bc
b
2
+ bc + c
2
+
_
b
2
+ ca
c
2
+ ca + a
2
+
_
c
2
+ ab
a
2
+ ab + b
2

6
Li gii. S dng bt ng thc Holder, ta c
_

cyc
_
a
2
+ bc
b
2
+ bc + c
2
_
2
_

cyc
(a
2
+ bc)
2
(b
2
+ bc + c
2
)
_
(a
2
+ b
2
+ c
2
+ ab + bc + ca)
3
Ta cn chng minh
(a
2
+ b
2
+ c
2
+ ab + bc + ca)
3
6

cyc
(a
2
+ bc)
2
(b
2
+ bc + c
2
)
Hay
(a
2
+b
2
+c
2
+ab+bc+ca)
3
12

cyc
a
2
b
2
(a
2
+b
2
)+6

cyc
a
3
b
3
+6

cyc
a
4
bc+12

cyc
a
2
b
2
c(a+b)+36a
2
b
2
c
2
www.VNMATH.com
71
Khng mt tnh tng qut, gi s a+b +c = 1, t ab +bc +ca =
1q
2
3
, r = abc th ta c 1 q 0
v r max
_
0,
(12q)(1+q)
2
27
_
, bt ng thc tr thnh
2(4q
2
+ 5)r +
17
27
q
6

8
9
q
4

20
9
q
2
+
10
27
0
Nu 2q 1 th ta c
17
27
q
6

8
9
q
4

20
9
q
2
+
10
27
0 nn bt ng thc hin nhin ng, nu 2q 1
th ta c
V T
2
27
(4q
2
+ 5)(1 2q)(q + 1)
2
+
17
27
q
6

8
9
q
4

20
9
q
2
+
10
27
=
1
27
q
2
(17q
4
+ 16q
3
+ 20q 38) 0
Bt ng thc c chng minh xong. ng thc xy ra khi v ch khi a = b = c.

60 Chng minh rng vi mi x, y [0, 1], ta c


1
x
2
x + 1
+
1
y
2
y + 1
1 +
1
x
2
y
2
xy + 1
Li gii. Bt ng thc tng ng vi
_
1
1
x
2
x + 1
__
1
1
y
2
y + 1
_

1
(x
2
x + 1)(y
2
y + 1)

1
x
2
y
2
xy + 1
Hay
xy(1 x)(1 y)
(x
2
x + 1)(y
2
y + 1)

(1 x)(1 y)(x + y)
(x
2
x + 1)(y
2
y + 1)(x
2
y
2
xy + 1)
x + y xy(x
2
y
2
xy + 1)
Do x, y 1 nn x
2
y
2
xy + 1 1, do
x + y xy(x
2
y
2
xy + 1) x + y xy = x(1 y) + y 0
Bt ng thc c chng minh xong.

61 Cho cc s dng a, b, c, chng minh bt ng thc


_
a
a + b
+
_
b
b + c
+
_
c
c + a

3

2

_
ab + bc + ca
a
2
+ b
2
+ c
2
Li gii. Trc ht ta chng minh rng
3(a
2
+ b
2
+ c
2
)
a + b + c
2

cyc
a
2
a + c
Tht vy, ta c
(V T V P)(a + b + c) = 3(a
2
+ b
2
+ c
2
) (a + b + c)

cyc
a
2
+ b
2
a + b
= a
2
+ b
2
+ c
2

cyc
c(a
2
+ b
2
)
a + b
=

cyc
ab(a b)
2
(a + c)(b + c)
0
www.VNMATH.com
72 CHNG 2. SOLUTION
S dng bt ng thc ny v bt ng thc Holder, ta c
3(a
2
+ b
2
+ c
2
)
a + b + c
_

cyc
_
a
a + b
_
2
_

cyc
a(a + b)(a + c)
_
2
_

cyc
a
2
a + c
__

cyc
_
a
a + b
_
2
_

cyc
a(a + b)(a + c)
_
2(a + b + c)
4
Ta cn chng minh
4(a + b + c)
5
27(ab + bc + ca)

cyc
a(a + b)(a + c)
Khng mt tnh tng qut, gi s a + b + c = 1, t ab + bc + ca =
1q
2
3
, r = abc (1 q 0), th
th ta c r
(1q)
2
(1+2q)
27
, bt ng thc tr thnh
4 (1 q
2
)(6q
2
+ 3 + 27r)
Ta c
4 (1 q
2
)(6q
2
+3 +27r) 4 (1 q
2
)(6q
2
+3 +(1 q)
2
(1 +2q)) = q
2
(2q
3
+2q
2
+(q 1)
2
) 0
Bt ng thc c chng minh xong. ng thc xy ra khi v ch khi a = b = c.

62 Chng minh rng vi mi a, b, c 0, ta c bt ng thc


a
2
(b + c)
(b
2
+ c
2
)(2a + b + c)
+
b
2
(c + a)
(c
2
+ a
2
)(2b + c + a)
+
c
2
(a + b)
(a
2
+ b
2
)(2c + a + b)

2
3
Li gii. S dng bt ng thc Cauchy Schwarz, ta c
_

cyc
a
2
(b + c)
(b
2
+ c
2
)(2a + b + c)
__

cyc
a
2
(b
2
+ c
2
)(2a + b + c)
b + c
_
(a
2
+ b
2
+ c
2
)
2
Do , ta cn chng minh
3(a
2
+ b
2
+ c
2
)
2
2

cyc
a
2
(b
2
+ c
2
)(2a + b + c)
b + c
Hay
3

cyc
a
4
+ 2

cyc
a
2
b
2
4

cyc
a
3
(b
2
+ c
2
)
b + c
3

cyc
_
a
4

a
3
(b
2
+ c
2
)
b + c
_
+

cyc
_
a
2
(b
2
+ c
2
)
a
3
(b
2
+ c
2
)
b + c
_
0
3

cyc
a
3
b(a b) ca
3
(c a)
b + c
+

cyc
a
2
(b
2
+ c
2
)(b + c a)
b + c
0
3

cyc
ab(a b)
2
(a
2
+ b
2
+ ab + bc + ca)
(a + c)(b + c)
+

cyc
a
2
(b
2
+ c
2
)(b + c a)
b + c
0
www.VNMATH.com
73
Khng mt tnh tng qut, gi s a b c 0. Khi , ta c
V T
2ab(a b)
2
(a
2
+ b
2
+ ab + bc + ca)
(a + c)(b + c)
+
a
2
(b
2
+ c
2
)(b a)
b + c
+
b
2
(a
2
+ c
2
)(a b)
a + c
=
(a b)
2
(2ab(a
2
+ b
2
+ ab + bc + ca) (a
2
b
2
+ (a
2
+ b
2
+ ab)c
2
+ (a + b)c
3
))
(a + c)(b + c)

(a b)
2
(2ab(a
2
+ b
2
+ ab + bc + ca) (a
2
b
2
+ (a
2
+ b
2
+ ab)ab + (a + b)abc))
(a + c)(b + c)
=
ab(a b)
2
(a
2
+ b
2
+ ac + bc)
(a + c)(b + c)
0
Bt ng thc c chng minh xong. ng thc xy ra khi v ch khi (a, b, c) (1, 1, 0).

63 Cho cc s dng a, b, c, chng minh rng vi mi k 2, ta c bt ng thc


a + b + c
3

abc

k
_
a + c
b + c
+
k
_
c + b
a + b
+
k
_
b + a
c + a
Li gii. S dng bt ng thc Holder, ta c
_

cyc
k
_
a + c
b + c
_k
2
3
k
2
1

cyc
_
a + c
b + c
Mt khc, theo bt ng thc AMGM th
_

cyc
k
_
a + c
b + c
_k
2
=
_

cyc
k
_
a + c
b + c
__

cyc
k
_
a + c
b + c
_k
2
1
3
k
2
1

cyc
k
_
a + c
b + c
Suy ra

cyc
k
_
a + c
b + c

cyc
_
a + c
b + c
Do , ta ch cn chng minh

cyc
a
3

abc

cyc
_
a + c
b + c
S dng bt ng thc Cauchy Schwarz, ta c
_

cyc
_
a + c
b + c
_
2
2
_

cyc
a
__

cyc
1
a + b
_
Nh vy, ta ch cn chng minh
a + b + c
3

a
2
b
2
c
2

2
a + b
+
2
b + c
+
2
c + a
Hay
(a + b + c)(a + b)(b + c)(c + a) 2
3

a
2
b
2
c
2
(a
2
+ b
2
+ c
2
+ 3ab + 3bc + 3ca)
S dng bt ng thc AMGM, ta c
ab + bc + ca 3

a
2
b
2
c
2
, (a + b + c)(ab + bc + ca) 9abc
www.VNMATH.com
74 CHNG 2. SOLUTION
Suy ra
(a + b)(b + c)(c + a) = (a + b + c)(ab + bc + ca) abc

8
9
(a + b + c)(ab + bc + ca)

8
3
3

a
2
b
2
c
2
(a + b + c)
Do
(a + b + c)(a + b)(b + c)(c + a)
8
3
3

a
2
b
2
c
2
(a + b + c)
2
Nh vy, ta ch cn chng minh
4
3
(a + b + c)
2
a
2
+ b
2
+ c
2
+ 3ab + 3bc + 3ca
Hay
(a + b + c)
2
3(ab + bc + ca) (ng theo AMGM)
Vy bt ng thc cn chng minh ng. ng thc xy ra khi v ch khi a = b = c.

64 Cho cc s khng m a, b, c, chng minh bt ng thc


3
_
a
b + c
+
3
_
b
c + a
+
3
_
c
a + b
2

abc
(a + b)(b + c)(c + a)
+ 1
Li gii. Trc ht, ta chng minh
_
a
b + c
+
_
b
c + a
+
_
c
a + b
2

abc
(a + b)(b + c)(c + a)
+ 1
Tht vy, bt ng thc tng ng

cyc
_
a(a + b)(a + c) 2
_
(a + b + c)(ab + bc + ca)
Hay
_

cyc
_
a(a + b)(a + c)
_
2
4(a + b + c)(ab + bc + ca)

cyc
a
3
+ 2

cyc
(a + b)
_
ab(a + c)(b + c) 3

cyc
ab(a + b) + 9abc
S dng cc bt ng thc AMGM, Cauchy Schwarz v Schur, ta c
V T

cyc
a
3
+ 2

cyc
(a + b)
_
ab + c

ab
_
=

cyc
a
3
+ 2

cyc
ab(a + b) + 2

abc

cyc

c(a + b)

cyc
a
3
+ 2

cyc
ab(a + b) + 12abc =
_

cyc
a
3
+ 3abc
_
+ 2

cyc
ab(a + b) + 9abc
3

cyc
ab(a + b) + 9abc
www.VNMATH.com
75
Tip theo, ta s chng minh
3
_
a
b + c

a
2/3
b
2/3
+ c
2/3
Tht vy, bt ng thc tng ng
a
2
(b + c)
2

a
2
(b
2/3
+ c
2/3
)
3
Hay
(b
2/3
+ c
2/3
)
3
(b + c)
2
3b
2/3
c
2/3
(b
2/3
+ c
2/3
) 2bc (ng)
T bt ng thc ny, suy ra

cyc
3
_
a
b + c

cyc

a
2/3
b
2/3
+ c
2/3
Theo trn th

cyc

a
2/3
b
2/3
+ c
2/3
2

a
2/3
b
2/3
c
2/3
(a
2/3
+ b
2/3
)(b
2/3
+ c
2/3
)(c
2/3
+ a
2/3
)
+ 1
Suy ra

cyc
3
_
a
b + c
2

a
2/3
b
2/3
c
2/3
(a
2/3
+ b
2/3
)(b
2/3
+ c
2/3
)(c
2/3
+ a
2/3
)
+ 1
Do , ta ch cn chng minh

a
2/3
b
2/3
c
2/3
(a
2/3
+ b
2/3
)(b
2/3
+ c
2/3
)(c
2/3
+ a
2/3
)
+ 1

abc
(a + b)(b + c)(c + a)
+ 1
Hay
(a + b)(b + c)(c + a) a
1/3
b
1/3
c
1/3
(a
2/3
+ b
2/3
)(b
2/3
+ c
2/3
)(c
2/3
+ a
2/3
)
(x
3
+ y
3
)(y
3
+ z
3
)(z
3
+ x
3
) xyz(x
2
+ y
2
)(y
2
+ z
2
)(z
2
+ x
2
)
trong x = a
1/3
, y = b
1/3
, z = c
1/3
.
Bt ng thc ny chnh l h qu ca cc bt ng thc hin nhin sau
x + y 2

xy, x
2
xy + y
2

1
2
(x
2
+ y
2
) x, y 0
Vy ta c pcm. ng thc xy ra khi v ch khi (a, b, c) (1, 1, 0).

65 Cho cc s thc a, b, c, d tha a


2
+ b
2
+ c
2
+ d
2
= 4, chng minh bt ng thc
9(a + b + c + d) 4abcd + 32
Li gii. D dng kim tra c max{abc, bcd, cda, dab} <
9
4
, do nu tn ti 1 trong 4 s a, b, c, d
khng ln hn 0, chng hn d 0, ta c
9(a+b+c+d)4abcd = 9(a+b+c)+d(94abc) 9(a+b+c) 9
_
3(a
2
+ b
2
+ c
2
+ d
2
) = 18

3 < 32
www.VNMATH.com
76 CHNG 2. SOLUTION
Nh vy, ta ch cn phi xt bt ng thc trong trng hp a, b, c, d > 0. Khng mt tnh tng
qut, gi s d = min{a, b, c, d}, suy ra 1 d > 0, t P(a, b, c, d) = V T V P, x =
_
a
2
+b
2
+c
2
3
v
p = a +b +c, th th ta c 2

3 3x p x

3, ta s chng minh P(a, b, c, d) P(x, x, x, d), tht


vy bt ng thc tng ng
9(3x p) 4d(x
3
abc)
T bt ng thc Schur bc 4

cyc
a
2
(ab)(ac) 0, ta suy ra c abc
(p
2
6x
2
)(p
2
+3x
2
)
12p
, nh
vy, ta ch cn chng minh
9(3x p) 4d
_
x
3

(p
2
6x
2
)(p
2
+ 3x
2
)
12p
_
Hay
(3x p)
_
27
d(p
3
+ 3p
2
x + 6px
2
+ 6x
3
)
p
_
0
Ch rng 3x p x

3 nn
81
3d(p
3
+ 3p
2
x + 6px
2
+ 6x
3
)
p
81 78x
2
d = 81 26d(4 d
2
) = 3 + 26(1 d)(3 d d
2
) 0
Nh vy bt ng thc ng v ta cn phi chng minh
9(3x + d) 4x
3
d 32
Hay
(9 4x
3
)
_
4 3x
2
= d(9 4x
3
) 32 27x
f(x) =
32 27x
(9 4x
3
)

4 3x
2
1
Ta c
f

(x) =
12(x 1)(81x
4
47x
3
119x
2
+ 9x + 81)
(9 4x
3
)
2
(4 3x
2
)
3/2
0
Suy ra f(x) l hm ng bin, do f(x) f(1) = 1, vy ta c pcm. ng thc xy ra khi v
ch khi a = b = c = d = 1.

66 Cho cc s khng m a, b, c, chng minh bt ng thc


_
a
2
+ 256bc
b
2
+ c
2
+
_
b
2
+ 256ca
c
2
+ a
2
+
_
c
2
+ 256ab
a
2
+ b
2
12
Li gii. Khng mt tnh tng qut, gi s a b c 0, xt cc trng hp sau
Trng hp 1. 256b
3
a
2
c, suy ra a
2
256b
2
, do
_
a
2
+ 256bc
b
2
+ c
2
16
_
b
2
+ bc
b
2
+ c
2
16 > 12
Trng hp 2. 256b
3
a
2
c, khi ta c
a
2
+ 256bc
b
2
+ c
2
=
a
2
b
2
+
c(256b
3
a
2
c)
b
2
(b
2
+ c
2
)

a
2
b
2
,
b
2
+ 256ca
c
2
+ a
2
=
b
2
a
2
+
c(256a
3
b
2
c)
a
2
(a
2
+ c
2
)

b
2
a
2
Do
V T
a
b
+
b
a
+ 16
_
ab
a
2
+ b
2
=
a
2
+ b
2
ab
+ 2 8
_
ab
a
2
+ b
2
3
3

8
2
= 12
Bt ng thc c chng minh xong. ng thc xy ra khi v ch khi (a, b, c)
_
2 +

3, 1, 0
_
.

www.VNMATH.com
77
67 Cho cc s dng x, y, z c tch bng 1, chng minh rng
x
y
4
+ 2
+
y
z
4
+ 2
+
z
x
4
+ 2
1
Li gii. Do x, y, z > 0, xyz = 1 nn tn ti cc s dng a, b, c sao cho x =
a
b
, y =
c
a
, z =
b
c
, khi bt
ng thc tr thnh

cyc
a
5
b(c
4
+ 2a
4
)
1
S dng bt ng thc Cauchy Schwarz, ta c
V T
(a
3
+ b
3
+ c
3
)
2
2(a
5
b + b
5
c + c
5
a) + abc(a
3
+ b
3
+ c
3
)
Ta cn chng minh
(a
3
+ b
3
+ c
3
)
2
2(a
5
b + b
5
c + c
5
a) + abc(a
3
+ b
3
+ c
3
)
Hay
(a
3
+ b
3
+ c
3
)
2
(ab + bc + ca)(a
4
+ b
4
+ c
4
)

cyc
a
5
b

cyc
ab
5
2
_

cyc
a
3
_
2

cyc
a
2
__

cyc
a
4
_
+
_

cyc
a
4
__
2

cyc
a
2
2

cyc
ab
_
2
_

cyc
a
5
b

cyc
ab
5
_

cyc
(a
4
+ b
4
+ c
4
2a
2
b
2
)(a b)
2
2(a b)(b c)(a c)((a + b + c)(a
2
+ b
2
+ c
2
) + abc)
T y, khng mt tnh tng qut, ta ch cn xt a b c l . t a = a
1
+t, b = b
1
+t, c = c
1
+t
vi t c
1
, xt hm s
f(t) =

cyc
((a
1
+ t)
4
+ (b
1
+ t)
4
+ (c
1
+ t)
4
2(a
1
+ t)
2
(b
1
+ t)
2
)(a b)
2
+ 2

cyc
(a b)
__

cyc
(a
1
+ t)
__

cyc
(a
1
+ t)
2
_
+ (a
1
+ t)(b
1
+ t)(c
1
+ t)
_
Ta c
f

(t) = 4

cyc
((a
1
+ t)
3
+ (b
1
+ t)
3
+ (c
1
+ t)
3
(a
1
+ t)
2
(b
1
+ t) (a
1
+ t)(b
1
+ t)
2
)(a b)
2
+ 2

cyc
(a b)
_
_
3

cyc
(a
1
+ t)
2
+ 2
_

cyc
(a
1
+ t)
_
2
+

cyc
(a
1
+ t)(b
1
+ t)
_
_
f

(t) = 4

cyc
(2(a
1
+ t)
2
+ 2(b
1
+ t)
2
+ 3(c
1
+ t)
2
4(a
1
+ t)(b
1
+ t))(a b)
2
+ 24

cyc
(a b)

cyc
(a
1
+ t)
= 4

cyc
(2a
2
+ 2b
2
+ 3c
2
4ab)(a b)
2
24(a b)(b c)(a c)(a + b + c)
= 2

cyc
(2a
2
2b
2
+ 5bc 3ca 2ab)
2
+ 6

cyc
c
2
(a b)
2
0
www.VNMATH.com
78 CHNG 2. SOLUTION
Suy ra f

(t) l hm ng bin, do
f

(t) f

(c
1
) = 2(4x
5
11x
4
y + 6x
3
y
2
+ 6x
2
y
3
xy
4
+ 4y
5
) 0
trong x = a
1
c
1
, y = b
1
c
1
. Nh vy f(t) l hm ng bin, suy ra
f(t) f(c
1
) = x
6
2x
5
y + 2x
3
y
3
+ y
6
0
Bt ng thc c chng minh xong. ng thc xy ra khi v ch khi x = y = z = 1.

68 Chng minh rng vi mi s dng a, b, c, d ta c bt ng thc


_
1
a
+
1
b
+
1
c
+
1
d
__
1
a + b
+
1
b + c
+
1
c + d
+
1
d + a
_

16
abcd + 1
Li gii. S dng bt ng thc AMGM, ta c
V T =
_
1
ab
+
1
cd
_
+
_
a + b
ab(c + d)
+
c + d
cd(a + b)
_
+
a + b
ab(d + a)
+
a + b
ab(b + c)
+
c + d
cd(b + c)
+
c + d
cd(d + a)

abcd
+
a + b
ab(d + a)
+
a + b
ab(b + c)
+
c + d
cd(b + c)
+
c + d
cd(d + a)
Tng t, ta c
V T
4

abcd
+
b + c
bc(a + b)
+
b + c
bc(c + d)
+
a + d
ad(a + b)
+
a + d
ad(c + d)
Do
2V T
8

abcd
+
_
a + b
ab(d + a)
+
a + d
ad(a + b)
_
+
_
a + b
ab(b + c)
+
b + c
bc(a + b)
_
+
_
c + d
cd(b + c)
+
b + c
bc(c + d)
_
+
_
c + d
cd(d + a)
+
a + d
ad(c + d)
_

abcd
+
2
a

bd
+
2
b

ca
+
2
c

bd
+
2
d

ca
=
8

abcd
+
2

bd
_
1
a
+
1
c
_
+
2

ac
_
1
b
+
1
d
_

16

abcd
Suy ra
V T
8

abcd

16
abcd + 1
Vy ta c pcm. ng thc xy ra khi v ch khi a = b = c = d = 1.

69 Cho cc s dng a, b, c, d tha a


2
+ b
2
+ c
2
+ d
2
= 4, chng minh bt ng thc
a + b + c + d
2

3

(abcd + 1)
_
1
a
+
1
b
+
1
c
+
1
d
_
Li gii. S dng kt qu bi ton 65, ta c
9(a + b + c + d) 4abcd + 32
www.VNMATH.com
79
Mt khc, theo bt ng thc AMGM th
abcd 1,
1
a
+
1
b
+
1
c
+
1
d

4
4

abcd
Nh vy, ta cn chng minh
2(x
4
+ 8) 9
3
_
4(x
4
+ 1)
x
vi x =
4

abcd 1.
Hay
f(x) =
x(x
4
+ 8)
3
x
4
+ 1

729
2
Ta c f

(x) =
(x
4
+8)
2
(9x
8
11x
4
+8)
(x
4
+1)
2
> 0, suy ra f(x) l hm ng bin, do f(x) f(1) =
729
2
.
Bt ng thc c chng minh xong. ng thc xy ra khi v ch khi a = b = c = d = 1.

70 Cho cc s dng a
1
, a
2
, . . . , a
n
tha a
1
a
2
a
n
= 1. Khi , vi mi k R, ta c
1
(1 + a
1
)
k
+
1
(1 + a
2
)
k
+ +
1
(1 + a
n
)
k
min
_
1,
n
2
k
_
Li gii. Nhn xt rng ta ch cn chng minh trong trng hp k > 0 l . t f(t) =
1
(t+1)
k
. Gi
M l trung bnh nhn ca a
1
, a
2
, . . . , a
n
. Khi , bt ng thc cn chng minh tng ng vi
f(a
1
) + f(a
2
) + + f(a
n
) min {nf(M), 1}
Ta c B sau
B . Nu 0 < a b c d v ad = bc th
f(a) + f(d) min {f(b) + f(c), 1}
Tht vy, t m =

ad =

bc va g(t) = f(mt) + f
_
m
t
_
= (mt + 1)
k
+
_
m
t
+ 1
_
k
vi mi t > 0.
Li t t
1
=
c
m
, t
2
=
d
m
th t
2
t
1
1. Ta cn chng minh
g(t
2
) min {g(t
1
), 1}
Xt tnh n iu ca g trn [1, +), ta c
g

(t) = mk
_
1
t
2
_
m
t
+ 1
_
k1
(mt + 1)
k1
_
g

(t) > 0
1
t
2
_
m
t
+ 1
_
k1
> (mt + 1)
k1
h(t) = t
2
k+1
mt + mt
1k
1+k
1 < 0
Li c
h

(t) =
2
k + 1
t
1k
1+k
m +
1 k
1 + k
mt

2k
k+1
h

(t) =
2(1 k)
(k + 1)
2
t

3k+1
k+1
(t mk)
h(1) = 0, h

(1) =
2(1 km)
k + 1
www.VNMATH.com
80 CHNG 2. SOLUTION
Ty thuc vo cc gi tr ca m v k, xt cc trng hp sau
(i) k = 1, m 1, ta c h(t) = (1 m)(t 1) 0 t > 1, do h 0 trn (1, +).
(ii) k = 1, m > 1, ta c h(t) = (1 m)(t 1) < 0 t > 1, do h < 0 trn (1, +).
(iii) k < 1, m
1
k
, khi , ta c h

> 0 t > 1, v h

(1) 0 nn h

> 0 trn (1, +). V h(1) = 0


v h lin tc nn h > 0 trn (1, +).
(iv) k < 1, m >
1
k
, khi , ta c h

(1) < 0 v h

< 0 t (1, mk), suy ra h

< 0 t (1, mk). V


h(1) = 0 v h lin tc nn h < 0 t (1, mk]. Trn (mk, +), ta c h

> 0, tc h l hm lm
trn (mk, +). Ta li c h(mk) < 0 v lim
t+
h(t) = + nn tn ti duy nht p > 1 sao cho h < 0
t (1, p) v h > 0 t (p, +).
(v) k > 1, m
1
k
, khi , ta c h

< 0 t > 1, tc h l hm li trn (1, +). Do h l hm lin tc


nn h(t) min
_
h(1), lim
t+
h(t)
_
= 0 t > 1.
(vi) k > 1, m >
1
k
, khi , ta c h

> 0 t (1, mk), tc h l hm lm trn (1, mk) v h

< 0
t (mk, +), tc h l hm li trn (mk, +). Nu h(mk) < 0 th do h(1) = 0 v h lin tc
nn h 0 t (1, mk], li do lim
t+
h(t) = + nn tn ti duy nht u > mk sao cho h < 0
t (mk, u) v h 0 t (u, +). Nu h(mk) 0 th do h l hm li trn (mk, +) nn
h(t) min
_
h(mk), lim
t+
h(t)
_
0 t > mk, do h l hm lm trn (1, mk) v h(1) = 0 nn tn
ti v [1, mk] sao cho h 0 t (1, v] v h 0 t [v, mk].
T cc trng hp ni trn
+, Nu h(t
2
) 0 th h 0 t (t
2
, +), tc l g

0. Suy ra, g l hm khng tng trn [t


2
, +).
Do
g(t
2
) lim
t+
g(t) = 1
+, Nu h(t
2
) < 0 th h 0 t (1, t
2
), tc l g

0. Suy ra, g l hm khng gim trn (1, t


2
). Do

g(t
2
) g(t
1
)
Vy, ta c
g(t
2
) min {g(t
1
), 1}
B c chng minh hon ton.
Tr li bi ton ca ta, ta s chng minh bng quy np theo n. Trng hp n = 1, n = 2 th bt
ng thc hin nhin ng. Gi s bt ng thc ng cho s bin b hn n (n 3). Ta s chng
minh minh n cng ng cho s bin bng n. Ta cn chng minh
f(a
1)
+ f(a
2
) + + f(a
n
) min {nf(M), 1}
D thy rng trong dy a
1
, a
2
, . . . , a
n
lun tn ti t nht mt s khng ln hn M v t nht
mt s khng nh hn M. Khng mt tnh tng qut, ta c th gi s a
1
M a
2
. K hiu
x
1
= min
_
M,
a
1
a
2
M
_
v x
2
= max
_
M,
a
1
a
2
M
_
. Khi , ta c a
1
x
1
x
2
a
2
v x
1
x
2
= a
1
a
2
, do
s dng B trn, ta c
f(a
1
) + f(a
2
) min {f(x
1
) + f(x
2
), 1} = min
_
f(M) + f
_
a
1
a
2
M
_
, 1
_
Ch rng
a
1
a
2
M
, a
3
, a
4
, . . . , a
n
cng c trung bnh nhn l M v s bin l n 1 < n nn theo gi
thit quy np, ta c
f
_
a
1
a
2
M
_
+ f(a
3
) + . . . + f(a
n
) min {(n 1)f(M), 1}
www.VNMATH.com
81
Do
n

i=1
f(a
i
) min
_
f(M) + f
_
a
1
a
2
M
_
, 1
_
+ f(a
3
) + + f(a
n
)
min
_
f(M) + f
_
a
1
a
2
M
_
+ f(a
3
) + + f(a
n
), 1
_
min{nf(M), 1}
Vy bt ng thc trn cng ng cho s bin bng n. Theo nguyn l quy np, ta suy ra n ng
vi mi n. Bi ton c gii quyt hon ton.

71 Cho a, b, c l cc s dng, chng minh rng


1.
a
9
bc
+
b
9
ca
+
c
9
ab
+
2
abc
a
5
+ b
5
+ c
5
+ 2
2.
a
9
bc
+
b
9
ca
+
c
9
ab
+
3
abc
a
4
+ b
4
+ c
4
+ 3
Li gii. (1) S dng bt ng thc AMGM, ta c
a
9
bc
+ abc 2a
5
,
b
9
ca
+ abc 2b
5
,
c
9
ab
+ abc 2c
5
Suy ra
a
9
bc
+
b
9
ca
+
c
9
ab
2(a
5
+ b
5
+ c
5
) 3abc a
5
+ b
5
+ c
5
+ 3
3

a
5
b
5
c
5
3abc
Do chng minh bt ng thc cho, ta ch cn chng minh
3
3

a
5
b
5
c
5
3abc +
2
abc
2
Hay
3t
5
3t
3
+
2
t
3
2
vi t =
3

abc > 0.
(t 1)
2
(3t
6
+ 6t
5
+ 6t
4
+ 6t
3
+ 6t
2
+ 4t + 2)
t
3
0 (ng)
Vy bt ng thc cn chng minh ng. ng thc xy ra khi v ch khi a = b = c = 1.
(2) Tng t nh trn, p dng bt ng thc AMGM, ta cng c
a
9
bc
+ abc + a
2
3a
4
,
b
9
ca
+ abc + b
2
3b
4
,
c
9
ab
+ abc + c
2
3c
4
Suy ra
a
9
bc
+
b
9
ca
+
c
9
ab
3(a
4
+ b
4
+ c
4
) (a
2
+ b
2
+ c
2
) 3abc
Li p dng bt ng thc AMGM, ta c
1
2
(a
4
+ 1) a
2
,
1
2
(b
4
+ 1) b
2
,
1
2
(c
4
+ 1) c
2
Suy ra
1
2
(a
4
+ b
4
+ c
4
) a
2
+ b
2
+ c
2

3
2
www.VNMATH.com
82 CHNG 2. SOLUTION
Do t trn, ta c
a
9
bc
+
b
9
ca
+
c
9
ab

5
2
(a
4
+ b
4
+ c
4
) 3abc
3
2

9
2
3

a
4
b
4
c
4
+ (a
4
+ b
4
+ c
4
) 3abc
3
2
Nh vy, chng minh bt ng thc cho, ta ch cn chng minh
9
2
3

a
4
b
4
c
4
3abc
3
2
+
3
abc
3
Hay

9
2
t
4
3t
3
+
3
t
3

9
2
vi t =
3

abc > 0.
3
2
(t 1)
2
(t + 1)(3t
4
+ t
3
+ 4t
2
+ 2t + 2) 0 (ng)
Vy bt ng thc cn chng minh ng. ng thc xy ra khi v ch khi a = b = c = 1.

72 Cho x, y, z, t l cc s dng tha xyzt = 1, chng minh rng


1
xy + yz + zx + 1
+
1
yz + zt + ty + 1
+
1
zt + tx + xz + 1
+
1
tx + xy + yt + 1
1
Li gii. t a =
1
x
, b =
1
y
, c =
1
z
, d =
1
t
, th ta c a, b, c, d > 0 v abcd = 1. Bt ng thc cn chng
minh tr thnh
1
a(b + c + d) + 1
+
1
b(c + d + a) + 1
+
1
c(d + a + b) + 1
+
1
d(a + b + c) + 1
1
Khng mt tnh tng qut, ta c th gi s a b c d > 0, th cd 1. Khi , theo bt ng
thc AMGM, ta c
1
c(d + a + b) + 1
+
1
d(a + b + c) + 1

1
c
_
d + 2

ab
_
+ 1
+
1
d
_
2

ab + c
_
+ 1
Mt khc
1
a(b + c + d) + 1
+
1
b(c + d + a) + 1
=
(a + b)(c + d) + 2(ab + 1)
(a + b)(c + d)(ab + 1) + ab(c + d)
2
+ (ab + 1)
2
=
m + 2(ab + 1)
m(ab + 1) + ab(c + d)
2
+ (ab + 1)
2
= f(m)
trong m = (a + b)(c + d) 2

ab(c + d) > 0. Ta c
f

(m) =
ab(c
2
+ d
2
+ cd ab 2)
(m(ab + 1) + ab(c + d)
2
+ (ab + 1)
2
)
2
0
Do f(m) l hm ngch bin, suy ra
f(m) f
_
2

ab(c + d)
_
=
2

ab
_
c + d +

ab
_
+ 1
t A =

ab, th ta c A 1 v A
2
cd = 1. Do
V T
2
A(c + d + A) + 1
+
1
c (d + 2A) + 1
+
1
d (2A + c) + 1
www.VNMATH.com
83
Nh vy, chng minh bt ng thc cho, ta ch cn chng minh
2
A(c + d + A) + 1
+
1
c (d + 2A) + 1
+
1
d (2A + c) + 1
1
t p = A(c +d), khi sau mt vi tnh ton n gin (vi ch rng A
2
cd = 1), ta c bt ng
thc trn tng ng vi
2cdp
2
+ (cd 1)
2
p + 3A
2
c
2
d
2
3cd 7 0
Hay
2A
2
cd(c + d)
2
+ A(c + d)(cd 1)
2
+ 3A
2
c
2
d
2
3cd 7 0
2(c + d)
2
+ A(c + d)(cd 1)
2
+ 3A
2
c
2
d
2
3cd 7 0
2(c d)
2
+ A(c + d)(cd 1)
2
+ 3A
2
c
2
d
2
+ 5cd 7 0
2(c d)
2
+ A(c + d)(cd 1)
2
+
3
cd
c
2
d
2
+ 5cd 7 0
2(c d)
2
+ A(c + d)(cd 1)
2
+
(cd 1)
2
(3 cd)
cd
0
Bt ng thc ny hin nhin ng v cd 1. Vy bt ng thc cn chng minh ng. ng thc
xy ra khi v ch khi a = b = c = d = 1.

73 Chng minh rng vi mi x, y, z, t > 0 th


(x + y)(x + z)(x + t)(y + z)(y + t)(z + t) 4xyzt(x + y + z + t)
2
Li gii. t a =
1
x
, b =
1
y
, c =
1
z
, d =
1
t
th ta c a, b, c, d > 0. Khi , bt ng thc trn tr thnh
(a + b)(a + c)(a + d)(b + c)(b + d)(c + d) 4(abc + abd + acd + bcd)
2
S dng bt ng thc Cauchy Schwarz, ta c
(abc + abd + acd + bcd)
2
=
_

ac b

ac +

bd a

bd +

ad c

ad +

bc d

bc
_
2
(ac + bd + ad + bc)(ab
2
c + a
2
bd + ac
2
d + bcd
2
)
= (a + b)(c + d)(ab
2
c + a
2
bd + ac
2
d + bcd
2
)
(abc + abd + acd + bcd)
2
=
_

bc a

bc +

ad b

ad +

ac d

ac +

bd c

bd
_
2
(bc + ad + ac + bd)(a
2
bc + ab
2
d + acd
2
+ bc
2
d)
= (a + b)(c + d)(a
2
bc + ab
2
d + acd
2
+ bc
2
d)
Cng 2 bt ng thc trn v vi v, ta c
2(abc + abd + acd + bcd)
2
(a + b)
2
(c + d)
2
(ab + cd)
Tng t, ta c
2(abc + abd + acd + bcd)
2
(a + c)
2
(b + d)
2
(ac + bd)
2(abc + abd + acd + bcd)
2
(a + d)
2
(b + c)
2
(ad + bc)
Do
8(abc +abd+acd +bcd)
6
(a+b)
2
(a+c)
2
(a+d)
2
(b +c)
2
(b +d)
2
(c +d)
2
(ab +cd)(ac +bd)(ad +bc)
Nh vy, ta ch cn chng minh
(a + b)(a + c)(a + d)(b + c)(b + d)(c + d) 8(ab + cd)(ac + bd)(ad + bc)
www.VNMATH.com
84 CHNG 2. SOLUTION
S dng bt ng thc AMGM, ta c
4(ab + cd)(ac + bd) (ab + cd + ac + bd)
2
= (a + d)
2
(b + c)
2
Tng t
4(ac + bd)(ad + bc) (a + b)
2
(c + d)
2
, 4(ab + cd)(ad + bc) (a + c)
2
(b + d)
2
Suy ra
4
3
(ab + cd)
2
(ac + bd)
2
(ad + bc)
2
(a + b)
2
(a + c)
2
(a + d)
2
(b + c)
2
(b + d)
2
(c + d)
2
Hay
(a + b)(a + c)(a + d)(b + c)(b + d)(c + d) 8(ab + cd)(ac + bd)(ad + bc)
Vy bt ng thc cn chng minh ng. ng thc xy ra khi v ch khi x = y = z = t.

74 Chng minh rng vi mi s dng a


1
, a
2
, . . . , a
n
tha a
1
a
2
a
n
= 1 ta c bt ng thc
_
a
2
1
+ 1 +
_
a
2
2
+ 1 + +
_
a
2
n
+ 1

2(a
1
+ a
2
+ + a
n
)
Li gii. Xt hm s f(x) =

x
2
+ 1

2x +
_

2
1

2
_
ln x vi x > 0. Ta c
f

(x) =
(x 1)
_
2x
2
+ x 1 2x
2
_
2(x
2
+ 1)
_
x
_
2(x
2
+ 1)
_
2x
2
+

x
2
+ 1
_
f

(x) = 0 x = 1
Qua 1 th f

(x) i du t dng sang m nn


f(x) f(1) = 0 x > 0
Hay
_
x
2
+ 1

2x
_

2
1

2
_
ln x x > 0
S dng bt ng thc ny cho n s a
1
, a
2
, . . . , a
n
ri cng li, ta c
n

i=1
_
a
2
i
+ 1

2
n

i=1
a
i

2
1

2
_
_
n

i=1
ln a
i
_
=

2
n

i=1
a
i

2
1

2
_
ln(a
1
a
2
a
n
) =

2
n

i=1
a
i
Vy bt ng thc cn chng minh ng. ng thc xy ra khi v ch khi a
1
= a
2
= = a
n
= 1.

75 Chng minh rng vi mi s dng a, b, c ta c bt ng thc


a +

ab +
3

abc
3

3
_
a
a + b
2

a + b + c
3
www.VNMATH.com
85
Li gii. t a = x
6
, b = y
6
, c = z
6
(x, y, z > 0). Ta c bt ng thc cn chng minh tng ng vi
(x
4
+ xy
3
+ y
2
z
2
)
3

9
2
(x
6
+ y
6
)(x
6
+ y
6
+ z
6
)
S dng bt ng thc Holder, ta c
(x
4
+ xy
3
+ y
2
z
2
)
3
= (x
2
x
2
1 + xy y
2
1 + y
2
z
2
1)
3
3(x
6
+ x
3
y
3
+ y
6
)(x
6
+ y
6
+ z
6
)
Mt khc, theo bt ng thc AMGM th
x
3
y
3

x
6
+ y
6
2
Suy ra
_
x
4
+ xy
3
+ y
2
z
2
_
3
3(x
6
+ x
3
y
3
+ y
6
)(x
6
+ y
6
+ z
6
)
9
2
(x
6
+ y
6
)(x
6
+ y
6
+ z
6
)
Bt ng thc c chng minh xong. ng thc xy ra khi v ch khi a = b = c.

76 Cho cc s khng m a, b, c, chng minh bt ng thc


a
3

b
2
bc + c
2
+
b
3

c
2
ca + a
2
+
c
3

a
2
ab + b
2
a
2
+ b
2
+ c
2
Li gii. S dng bt ng thc Cauchy Schwarz, ta c

cyc
a
3

b
2
bc + c
2

(a
2
+ b
2
+ c
2
)
2

cyc
a

b
2
bc + c
2
Do , ta ch cn chng minh

cyc
a
_
b
2
bc + c
2
a
2
+ b
2
+ c
2
Li s dng bt ng thc Cauchy Schwarz, ta c
_

cyc
a
_
b
2
bc + c
2
_
2

cyc
a
__

cyc
a(b
2
bc + c
2
)
_
Nh vy, ta ch cn chng minh
(a
2
+ b
2
+ c
2
)
2

cyc
a
__

cyc
a(b
2
bc + c
2
)
_
Hay

cyc
a
4
+ abc

cyc
a

cyc
ab(a
2
+ b
2
)
y chnh l bt ng thc Schur. Vy bt ng thc cn chng minh ng. ng thc xy ra khi
v ch khi (a, b, c) (1, 1, 1), hoc (a, b, c) (1, 1, 0).

www.VNMATH.com
86 CHNG 2. SOLUTION
77 Chng minh rng vi mi a, b, c khng m
_
a
2
a
2
+ 6ab + 2b
2
+
_
b
2
b
2
+ 6bc + 2c
2
+
_
c
2
c
2
+ 6ca + 2a
2
1
Li gii. t x =
b
a
, y =
c
b
, z =
a
c
th ta c x, y, z 0 v xyz = 1, khi bt ng thc tr thnh

cyc
1

2x
2
+ 6x + 1
1
Do x, y, z 0, xyz = 1 nn tn ti cc s m, n, p 0 sao cho x =
np
m
2
, y =
pm
n
2
, z =
mn
p
2
, bt ng
thc c vit li nh sau

cyc
m
2
_
m
4
+ 6m
2
np + 2n
2
p
2
1
S dng bt ng thc Holder, ta c
V T
2
_

cyc
m
2
(m
4
+ 6m
2
np + n
2
p
2
)
_
(m
2
+ n
2
+ p
2
)
3
Ta phi chng minh
(m
2
+ n
2
+ p
2
)
3

cyc
m
2
(m
4
+ 6m
2
np + 2n
2
p
2
)
Hay
3

cyc
m
4
(n p)
2
0
Bt ng thc ny hin nhin ng. Vy ta c pcm. ng thc xy ra khi v ch khi a = b = c
hoc cc s a, b, c tha
b
a
0,
c
b
0 v cc hon v.

78 Cho cc s khng m a, b, c, chng minh bt ng thc


_
a
b + c
+
_
b
c + a
+
_
c
a + b
+ 3
_
3(ab + bc + ca)
a
2
+ b
2
+ c
2

7

2
2
Li gii. Khng mt tnh tng qut, gi s a b c, ta s chng minh
_
b
c + a
+
_
c
a + b

_
b + c
a
Tht vy, bt ng thc tng ng
b
a + c
+
c
a + b
+ 2

bc
(a + b)(a + c)

b + c
a
Hay
2

bc
(a + b)(a + c)

bc
a(a + c)
+
bc
a(a + b)
2a

bc

2a + b + c
_
(a + b)(a + c)
www.VNMATH.com
87
2
_
a

bc
_

bc

(b c)
2
_
(a + b)(a + c)
_
a + b +

a + c
_
2
Ta c
V T V P
2
_

c
_

c

(b c)
2
4(a + b)(a + c)
=
_

c
_
_
_
_
2

c

_

c
__

b +

c
_
2
4(a + b)(a + c)
_
_
_ 0
V
_

b +

c
_
2
2(b + c) 2(a + c),

c
_

c
_

bc a
Mt khc, d thy
ab + bc + ca a(b + c), a
2
+ b
2
+ c
2
a
2
+ (b + c)
2
Suy ra
V T
_
a
b + c
+
_
b + c
a
+ 3

3a(b + c)
a
2
+ (b + c)
2
= x +
3

x
2
2
vi x =
_
a
b+c
+
_
b+c
a
2.
Ta cn chng minh
x +
3

x
2
2

7

2
2
Nu x
7

2
2
th bt ng thc hin nhin ng, xt trng hp ngc li x
7

2
2
, khi ta c
27
x
2
2

_
7

2
2
x
_
2
=
_
x 2

2
_
2
_
19 + 6

2x 2x
2
_
2(x
2
2)
0 (do x
7

2
2
)
Bt ng thc c chng minh xong. ng thc xy ra khi v ch khi (a, b, c)
_
3 + 2

2, 1, 0
_
.
Nhn xt. Mt cch tng qut, ta c kt qu sau
_
a
b + c
+
_
b
c + a
+
_
c
a + b
+ k
_
ab + bc + ca
a
2
+ b
2
+ c
2
min
x2
_
x +
k

x
2
2
_
a, b, c, k 0
Tuy nhin, bng tnh ton thc t, ta nhn thy vi k = 3

3 th bi ton c p s p nht.

79 Cho cc s khng m a, b, c, chng minh bt ng thc


a
b + c
+
b
c + a
+
c
a + b
+
16(ab + bc + ca)
a
2
+ b
2
+ c
2
8
Li gii. Khng mt tnh tng qut, gi s a +b +c = 1, t ab +bc +ca = x, bt ng thc tr thnh
3abc + 1 2x
x abc
+
16x
1 2x
8
Ta c
V T
1 2x
x
+
16x
1 2x
=
(6x 1)
2
x(1 2x)
+ 8 8
Bt ng thc c chng minh xong. ng thc xy ra khi v ch khi (a, b, c)
_
2 +

3, 1, 0
_
.

www.VNMATH.com
88 CHNG 2. SOLUTION
80 Cho cc s khng m a, b, c, chng minh bt ng thc
3(a
3
+ b
3
+ c
3
) + 2abc 11
_
a
2
+ b
2
+ c
2
3
_
3/2
Li gii. Khng mt tnh tng qut, gi s a+b+c = 1, t q = ab+bc+ca, r = abc th ta c
1
3
q 0,
ngoi ra, s dng bt ng thc Schur, ta cng c r max
_
0,
4q1
9
_
, bt ng thc tr thnh
11r + 3 9q 11
_
1 2q
3
_
3/2
Nu 1 4q, ta c
V T V P 3 9q 11
_
1 2q
3
_
3/2
> 0 (do
1
4
q 0)
Nu 4q 1, ta c
V T V P
16 37q
9
11
_
1 2q
3
_
3/2
=
(1 3q)(673q 107 968q
2
)
9
_
16 37q + 11
_
3(1 2q)
3
_ 0
Bt ng thc c chng minh xong. ng thc xy ra khi v ch khi a = b = c.

81 Cho cc s khng m a, b, c, d tha a


2
+ b
2
+ c
2
+ d
2
= 1, chng minh bt ng thc
a
3
1 bcd
+
b
3
1 cda
+
c
3
1 dab
+
d
3
1 abc

4
7
Li gii. S dng bt ng thc Cauchy Schwarz, ta suy ra c ta ch cn chng minh
7(a
3
+ b
3
+ c
3
+ d
3
)
2
+ 4abcd 4(a
3
+ b
3
+ c
3
+ d
3
) 0
Khng mt tnh tng qut, gi s d = min{a, b, c} suy ra d
1
2
, t t =
_
a
2
+b
2
+c
2
3
, ta s chng
minh
V T 7(3t
3
+ d
3
)
2
+ 4t
3
d 4(3t
3
+ d
3
)
Hay
7(a
3
+ b
3
+ c
3
3t
3
)(a
3
+ b
3
+ c
3
+ 2d
3
+ 3t
3
) 4(a
3
+ b
3
+ c
3
3t
3
) + 4d(t
3
abc)
T kt qu bi ton trn, ta d dng suy ra c 0 2(t
3
abc) 3(a
3
+ b
3
+ c
3
3t
3
), ta cn
chng minh
7(a
3
+ b
3
+ c
3
3t
3
)(a
3
+ b
3
+ c
3
+ 2d
3
+ 3t
3
) 4(a
3
+ b
3
+ c
3
3t
3
) + 6d(a
3
+ b
3
+ c
3
3t
3
)
Hay
7(a
3
+ b
3
+ c
3
+ 2d
3
+ 3t
3
) 4 + 6d
Theo bt ng thc Holder, ta c
a
3
+ b
3
+ c
3
+ d
3

1
2
, 3t
3
+ d
3

1
2
Suy ra
7(a
3
+ b
3
+ c
3
+ 2d
3
+ 3t
3
) 4 6d 3(1 2d) 0
www.VNMATH.com
89
Tip theo, ta phi chng minh
7(3t
3
+ d
3
)
2
+ 4t
3
d 4(3t
3
+ d
3
) 0
Hay
7(3t
3
+ d
3
)
2
+ 4t
3
d(3t
2
+ d
2
)
(3t
3
+ d
3
)(3t
2
+ d
2
)
3/2
4
f(x) =
7(x
3
+ 3)
2
+ 4x(x
2
+ 3)
(x
3
+ 3)(x
2
+ 3)
3/2
4
vi x =
d
t
1.
Ta c th d dng chng minh bt ng thc trn. ng thc xy ra khi v ch khi a = b = c = d =
1
2
.

82 Cho cc s khng m a, b, c, d tha a


3
+ b
3
+ c
3
+ d
3
= 1, chng minh bt ng thc
1
a
3
1 bcd
+
b
3
1 cda
+
c
3
1 dab
+
d
3
1 abc

4
3
Li gii. Ta c

cyc
a
3
1 bcd

cyc
a
3
= 1
Mt khc, s dng bt ng thc AMGM,

cyc
3a
3
3 3bcd

cyc
3a
3
3 (b
3
+ c
3
+ d
3
)
= 3

cyc
a
3
a
3
+ 2
= 12 6

cyc
1
a
3
+ 2
12
96
a
3
+ b
3
+ c
3
+ d
3
+ 8
=
4
3
Bt ng thc c chng minh xong.

83 Cho cc s dng a, b, c, d tha a + b + c + d = 4, chng minh rng


1
ab
+
1
bc
+
1
cd
+
1
da
a
2
+ b
2
+ c
2
+ d
2
Li gii. S dng bt ng thc AMGM, ta suy ra c vi mi x, y > 0 th
x
2
+ y
2

(x + y)
4
8xy
S dng bt ng thc ny, ta c
a
2
+ c
2

(a + c)
4
8ac
=
(a + c)
4
bd
8abcd

(a + c)
4
(b + d)
2
32abcd
Tng t, ta c
b
2
+ d
2

(b + d)
4
(a + c)
2
32abcd
Ta cn chng minh
32(a + c)(b + d) (a + c)
2
(b + d)
2
((a + c)
2
+ (b + d)
2
)
Hay
32 (a + c)(b + d)((a + c)
2
+ (b + d)
2
)
((a + c) + (b + d))
4
8(a + c)(b + d)((a + c)
2
+ (b + d)
2
) (ng)
Vy ta c pcm. ng thc xy ra khi v ch khi a = b = c = d = 1.

www.VNMATH.com
90 CHNG 2. SOLUTION
84 Cho cc s dng x, y, z, tm hng s k ln nht sao cho
x
y
+
y
z
+
z
x
+ 3k (k + 1)
x + y + z
3

xyz
Li gii. Khng mt tnh tng qut, gi s xyz = 1, khi tn ti a, b, c > 0 sao cho x =
a
b
, y =
c
a
, z =
b
c
,
bt ng thc tr thnh
a
3
+ b
3
+ c
3
+ 3kabc (k + 1)(ab
2
+ bc
2
+ ca
2
)
Cho a = 1, b =
3

2, c 0, ta suy ra c k
3
3

4
1, ta s chng minh y l gi tr cn tm.
Tht vy, khng mt tnh tng qut, gi s c = min{a, b, c}, t a = c + x, b = c + y (x, y 0),
bng tnh ton n gin, ta c bt ng thc tng ng vi
3
_
1
1
3

4
_
(x
2
xy + y
2
)c + x
3
+ y
3

3
3

4
xy
2
0
S dng bt ng thc AMGM, ta c
x
3
+ y
3
= x
3
+
y
3
2
+
y
3
2

3
3

4
xy
2
Vy ta c pcm, do gi tr k phi tm l
k
max
=
3
3

4
1.

85 Cho cc s khng m a, b, c, d, chng minh bt ng thc


_
a
a + b + c
+
_
b
b + c + d
+
_
c
c + d + a
+
_
d
d + a + b

4

3
Li gii. S dng bt ng thc Cauchy Schwarz, ta c
_

cyc
_
a
a + b + c
_
2

cyc
(a + b + d)(a + c + d)
__

cyc
a
(a + b + c)(a + b + d)(a + c + d)
_
=
2(2(a + b + c + d)
2
+ (a + c)(b + d))((a + c)(b + d) + ac + bd)
(a + b + c)(b + c + d)(c + d + a)(d + a + b)
Ta cn chng minh
8(a+b+c)(b+c+d)(c+d+a)(d+a+b) 3(2(a+b+c+d)
2
+(a+c)(b+d))((a+c)(b+d)+ac+bd)
t P(a, b, c, d) = V T V P = f(x) vi x = bd, r rng y l 1 hm bc nht theo x, nn ta phi
c
f(x) min
_
f(0), f
_
(b + d)
2
4
__
Do P(a, b, c, d) min
_
P(a, b + d, c, 0), P
_
a,
b+d
2
, c,
b+d
2
__
, tng t, ta c
P(a, b + d, c, 0) min
_
P(a + c, b + d, 0, 0), P
_
a + c
2
, b + d,
a + c
2
, 0
__
P
_
a,
b + d
2
, c,
b + d
2
_
min
_
P
_
a + c,
b + d
2
, 0,
b + d
2
_
, P
_
a + c
2
,
b + d
2
,
a + c
2
,
b + d
2
__
www.VNMATH.com
91
Nh vy, ta ch cn xt bi ton trong cc trng hp sau l
+, c = d = 0, bt ng thc tr thnh
8ab(a + b)
2
3ab(2a
2
+ 5ab + 2b
2
)
Hay
ab(2a
2
+ ab + 2b
2
) 0 (ng)
+, a = c, d = 0, bt ng thc tr thnh
16a(2a + b)(a + b)
2
6a(a + 2b)(a + b)(4a + b)
Hay
2a(a + b)(4a
2
3ab + 2b
2
) 0 (ng)
+, a = c, b = d, bt ng thc tr thnh
8(2a + b)
2
(a + 2b)
2
12(2a
2
+ 5ab + 2b
2
)(a
2
+ 4ab + b
2
)
Hay
4(2a + b)(a + 2b)(a b)
2
0 (ng)
Bt ng thc c chng minh xong. ng thc xy ra khi v ch khi a = b = c = d.
Nhn xt. Vi cch lm tng t, ta c th gii c bi ton sau ca Vasile Cirtoaje trn MR 1/2007

2(4 ab bc c d da)
_

2 + 1
_
(4 a b c d)
vi mi s thc khng m a, b, c, d tha a
2
+ b
2
+ c
2
+ d
2
= 4.

86 Chng minh rng vi mi a, b, c, d [1, 2], ta c


a + b
c + d
+
c + d
a + b

a + c
b + d

3
2
Li gii. t V T = f(a, c), ta c

2
f
a
2
=
2(c + d)
(a + b)
2
> 0,

2
f
c
2
=
2(a + b)
(c + d)
2
> 0
Do , f l hm li vi a, c. Xt cc trng hp sau
Trng hp 1. b d, khi do a, b, c, d [1, 2] nn 2d a, b, c, d
1
2
b, do f l hm li vi a, c
nn
f(a, c) max
_
f(2d, 2d), f
_
2d,
b
2
_
, f
_
b
2
, 2d
_
, f
_
b
2
,
b
2
__
Ta li c
f(2d, 2d) =
(2b + d)(b
2
4d
2
) 9d
2
(b + 4d)
6d(b + d)(b + 2d)
+
3
2

3
2
f
_
2d,
b
2
_
=
b 2d
2(b + d)
+
3
2

3
2
f
_
b
2
, 2d
_
=
b
2
(b 2d) + d(d b)(b + 4d))
2bd
+
3
2

3
2
f
_
b
2
,
b
2
_
=
(b d)(b 2d)(5b + 4d)
6b(b + d)(b + 2d)
+
3
2

3
2
www.VNMATH.com
92 CHNG 2. SOLUTION
Trng hp 2. d b, khi , do a, b, c, d [1, 2] nn 2b a, b, c, d
1
2
d, do f l hm li vi a, c
nn
f(a, c) max
_
f(2b, 2b), f
_
2b,
d
2
_
, f
_
d
2
, 2b
_
, f
_
d
2
,
d
2
__
Ta li c
f(2b, 2b) =
(b + 2d)(d
2
4b
2
) 9b
2
(4b + d)
6b(b + d)(2b + d)
+
3
2

3
2
f
_
2b,
d
2
_
=
d
2
(d 2b) + b(b d)(4b + d)
2bd(b + d)
+
3
2

3
2
f
_
d
2
, 2b
_
=
d 2b
2(b + d)
+
3
2

3
2
f
_
d
2
,
d
2
_
=
(d b)(d 2b)(5d + 4b)
6d(b + d)(2b + d)
+
3
2

3
2
Bt ng thc c chng minh xong.

87 Chng minh rng vi mi a, b, c > 0, ta lun c


a
2
b
c(b + c)
+
b
2
c
a(c + a)
+
c
2
a
b(a + b)

3
2

a
2
+ b
2
+ c
2
a + b + c
Li gii. t x =
1
a
, y =
1
b
, c =
1
c
th th bt ng thc tr thnh

cyc
x
2
y
2
(z + x)

3
2

x
2
y
2
+ y
2
z
2
+ z
2
x
2
xyz(xy + yz + zx)
Hay

cyc
x
2
(x + y + z)
y
2
(z + x)

3(x
2
y
2
+ y
2
z
2
+ z
2
x
2
)(x + y + z)
2xyz(xy + yz + zx)

cyc
x
2
y
2
+

cyc
x
2
y(z + x)

3(x
2
y
2
+ y
2
z
2
+ z
2
x
2
)(x + y + z)
2xyz(xy + yz + zx)
S dng kt qu bi ton 17 v bt ng thc Cauchy Schwarz, ta c

cyc
x
2
y
2

3
2

cyc
x
2
+ y
2
xy
6,

cyc
x
2
y(z + x)

(x + y + z)
2
2(xy + yz + zx)
Ta cn chng minh
3
2

cyc
x
2
+ y
2
xy
6 +
(x + y + z)
2
2(xy + yz + zx)

3(x
2
y
2
+ y
2
z
2
+ z
2
x
2
)(x + y + z)
2xyz(xy + yz + zx)
Chun ha cho x + y + z = 1 v t u = xy + yz + zx, v = xyz th ta c
1
3
q 0, r 0 v bt
ng thc tr thnh
3(u 3v)
2v
+
1
2u
6
3(u
2
2v)
2uv
Ta c
V T V P =
7(1 3u)
2u
0
Bt ng thc c chng minh xong. ng thc xy ra khi v ch khi a = b = c.

www.VNMATH.com
93
88 Cho cc s khng m a, b, c, tha a
2
+ b
2
+ c
2
= 3, chng minh rng
1 + 4abc 5 min{a, b, c}
Li gii. Khng mt tnh tng qut, gi s a = min{a, b, c}, suy ra 1 a 0, bt ng thc tr thnh
1 + 4abc 5a
Do a = min{a, b, c} nn ta c bc a

b
2
+ c
2
a
2
= a

3 2a
2
, ta phi chng minh
4a
2
_
3 2a
2
5a 1
Nu a
1
2
, ta c V T V P 2

10a
2
5a + 1 0, nu a
1
2
, bt ng thc tng ng vi
16a
2
(3 2a
2
) (5a 1)
2
Hay
(a 1)(32a
5
+ 32a
4
16a
3
16a
2
+ 9a 1) 0
32a
5
+ 32a
4
16a
3
16a
2
+ 9a 1 0
Ta c
32a
5
+ 32a
4
16a
3
16a
2
+ 9a 1 =
1
2
(2a 1)(32a
3
+ 3) + 2a(2a 1)
2
(4a
2
+ 4a + 3) +
1
2
> 0
Bt ng thc c chng minh xong. ng thc xy ra khi v ch khi a = b = c = 1.

89 Vi mi a, b, c 0 v ab + bc + ca = 1, ta c
1

2a
2
+ 3bc
+
1

2b
2
+ 3ca
+
1

2c
2
+ 3ab

6
3
Li gii. t x = bc, y = ca, z = ab, suy ra x + y + z = 1, bt ng thc tr thnh
_
x
3x
2
+ 2yz
+
_
y
3y
2
+ 2zx
+
_
z
3z
2
+ 2xy

2

2
_
3(x + y + z)
Ta c

cyc
x
3x
2
+ 2yz

4
3(x + y + z)
(2.2)

cyc
xy
(3x
2
+ 2yz)(3y
2
+ 2zx)

4
9(x + y + z)
2
(2.3)
v

cyc
x
3x
2
+ 2yz

4
3(x + y + z)
=
18A + 21B + 7C
3(x + y + z)(3x
2
+ 2yz)(3y
2
+ 2zx)(3z
2
+ 2xy)
0

cyc
xy
(3x
2
+ 2yz)(3y
2
+ 2zx)

4
9(x + y + z)
2
=
18A + 36B + 22C + 15xyz(x + y + z)
3
9(x + y + z)
2
(3x
2
+ 2yz)(3y
2
+ 2zx)(3z
2
+ 2xy)
0
vi A = (x y)
2
(y z)
2
(z x)
2
, B = xyz(x + y)(y + z)(z + x), C = (xyz)
2
. Tip theo, vi mi
m, n, p 0,
m + p + n =
_
m
2
+ n
2
+ p
2
+ 2(mn + np + pm)
=
_
m
2
+ n
2
+ p
2
+ 2
_
m
2
n
2
+ n
2
p
2
+ p
2
m
2
+ 2mnp(m + n + p)

_
m
2
+ n
2
+ p
2
+ 2
_
m
2
n
2
+ n
2
p
2
+ p
2
m
2
www.VNMATH.com
94 CHNG 2. SOLUTION
S dng bt ng thc ny v cc bt ng thc (2.2), (2.3), ta c

cyc
_
x
3x
2
+ 2yz

cyc
x
3x
2
+ 2yz
+ 2

cyc
xy
(3x
2
+ 2yz)(3y
2
+ 2zx)

_
4
3(x + y + z)
+ 2

4
9(x + y + z)
2
=
2

2
_
3(x + y + z)
Bt ng thc c chng minh xong.

90 Cho a, b, c l cc s thc khc 0 tha a


2
+ b
2
+ c
2
= (a b)
2
+ (b c)
2
+ (c a)
2
, chng minh bt
ng thc
1.
a
b
+
b
c
+
c
a
5 2.
1
12

a
2
b + b
2
c + c
2
a
(a + b + c)
3

5
36
Li gii. Khng mt tnh tng qut gi s a = max{a, b, c}. Ch rng iu kin bi v cc bt
ng thc trn s khng i nu ta thay (a, b, c) bi (a, b, c). Nh vy, ta ch cn xt a > 0,
khi
Nu b > 0 > c th
(a b)
2
+ (b c)
2
+ (c a)
2
(a
2
+ b
2
+ c
2
) = (a b)
2
+ c
2
2c(a + b) > 0
Nu c > 0 > b th
(a b)
2
+ (b c)
2
+ (c a)
2
(a
2
+ b
2
+ c
2
) = (a c)
2
+ b
2
2b(a + c) > 0
Nu b < 0, c < 0 th
(a b)
2
+ (b c)
2
+ (c a)
2
(a
2
+ b
2
+ c
2
) = (b c)
2
+ a
2
2a(b + c) > 0
Nh th, ta ch cn xt b > 0 v c > 0 l . T
0 = (a b)
2
+ (b c)
2
+ (c a)
2
(a
2
+ b
2
+ c
2
)
=
_

a +

b +

c
__

a +

c
__

b +

a
__

c +

b
_
Ta suy ra c

a =

b +

c, do bt ng thc tng ng vi
1.
_

b +

c
_
2
b
+
b
c
+
c
_

b +

c
_
2
5
2.
1
12

_

b +

c
_
4
b + b
2
c + c
2
_

b +

c
_
2
_
_

b +

c
_
2
+ b + c
_
3

5
36
t t =
_
b
c
> 0, ta phi chng minh
1.
(t + 1)
2
t
2
+ t
2
+
1
(t + 1)
2
5
2.
2
3

(t + 1)
4
t
2
+ t
4
+ (t + 1)
2
(t
2
+ t + 1)
3

10
9
www.VNMATH.com
95
Tht vy, bt ng thc (1) tng ng vi (t
3
+ t
2
2t 1)
2
0, v tri ca bt ng thc (2)
tng ng vi (t
3
+ 3t
2
1)
2
0, v v phi tng ng vi (t
3
3t
2
6t 1)
2
0.
Vy ta c pcm.

91 Tm hng s k > 0 nh nht sao cho bt ng thc


_
a + k(b c)
2
+
_
b + k(c a)
2
+
_
c + k(a b)
2

3
ng vi mi a, b, c 0 v a + b + c = 1.
Li gii. Cho a = b =
1
2
, c = 0, ta c k 1. Ta s chng minh k = 1 l gi tr cn tm, tc l
_
a + (b c)
2
+
_
b + (c a)
2
+
_
c + (a b)
2

3
Bnh phng 2 v, ta c th vit bt ng thc li nh sau

cyc
(b c)
2
+ 2

cyc
_
(a + (b c)
2
)(b + (a c)
2
) 2
S dng bt ng thc Cauchy Schwarz, ta c

cyc
_
(a + (b c)
2
)(b + (a c)
2
)

cyc

ab +

cyc
|(a c)(b c)|
Ta cn chng minh

cyc
(b c)
2
+ 2

cyc

ab + 2

cyc
|(a c)(b c)| 2
Khng mt tnh tng qut, gi s a b c 0, khi

cyc
(b c)
2
+ 2

cyc

ab + 2

cyc
|(a c)(b c)| 2 = 4(a c)
2
2
_
1

cyc

ab
_
= 4(a c)
2

_
a

c
_
2

_
_

b
_
2
+
_

c
_
2
_
4(a c)
2
2
_
a

c
_
2
= 2
_
a

c
_
2
_
2
_
a +

c
_
2
1
_
2
_
a

c
_
2
(2(a + c) 1) = 2
_
a

c
_
2
(a b + c) 0
Bt ng thc c chng minh xong. Vy ta c
k
min
= 1.

92 Chng minh rng vi mi a, b, c 0 th

a
3
+ abc
(b + c)
3
+

b
3
+ abc
(c + a)
3
+

c
3
+ abc
(a + b)
3

a
b + c
+
b
c + a
+
c
a + b
Li gii. Bnh phng 2 v, ta c bt ng thc tng ng

cyc
a
3
+ abc
(b + c)
3
+ 2

cyc

(a
3
+ abc)(b
3
+ abc)
(a + c)
3
(b + c)
3

cyc
a
2
(b + c)
2
+ 2

cyc
ab
(a + c)(b + c)
www.VNMATH.com
96 CHNG 2. SOLUTION
Ch rng (a
2
+ bc)(b
2
+ ca) ab(a + c)(b + c) = c(a b)
2
(a + b) 0, nn
(a
3
+ abc)(b
3
+ abc)
(a + c)
3
(b + c)
3

a
2
b
2
(a + c)
2
(b + c)
2
v do

(a
3
+ abc)(b
3
+ abc)
(a + c)
3
(b + c)
3

ab
(a + c)(b + c)
Nh vy, ta cn phi chng minh

cyc
a
3
+ abc
(b + c)
3

cyc
a
2
(b + c)
2
Ta c

cyc
_
a
3
+ abc
(b + c)
3

a
2
(b + c)
2
_
=

cyc
a(a b)(a c)
(b + c)
3
0
Bt ng thc c chng minh xong. ng thc xy ra khi v ch khi a = b = c hoc a = b, c = 0
v cc hon v.

93 Cho cc s dng a, b, c, chng minh rng


ab
2
c
2
+
bc
2
a
2
+
ca
2
b
2
+ a + b + c
6(a
2
+ b
2
+ c
2
)
a + b + c
Li gii. S dng bt ng thc Cauchy Schwarz, ta c
_

cyc
ab
2
c
2
+

cyc
a
__

cyc
1
a(b
2
+ c
2
)
_
=
_

cyc
a(b
2
+ c
2
)
c
2
__

cyc
1
a(b
2
+ c
2
)
_

cyc
1
a
_
2
Ta cn chng minh
_

cyc
a
__

cyc
1
a
_
2
6
_

cyc
a
2
__

cyc
1
a(b
2
+ c
2
)
_
Hay
_

cyc
a
__

cyc
1
a
_
2
9

cyc
1
a
3
_

cyc
2a
b
2
+ c
2

cyc
1
a
_

cyc
(a b)
2
ab
_
1
a
+
1
b
+
1
c
+
3(a + b)(c
2
ab)
(a
2
+ c
2
)(b
2
+ c
2
)
_
0
+, Nu
1
c

2(a+b)
ab
, th
1
a
+
1
b
+
1
c
+
3(a + b)(c
2
ab)
(a
2
+ c
2
)(b
2
+ c
2
)

1
a
+
1
b
+
1
c

3(a + b)
ab
0
www.VNMATH.com
97
+, Nu
1
c

2(a+b)
ab
hay c
ab
2(a+b)
, th
1
a
+
1
b
+
1
c
+
3(a + b)(c
2
ab)
(a
2
+ c
2
)(b
2
+ c
2
)
=
c
4
(ab + bc + ca) + abc
2
(a
2
+ b
2
) + (a + b)(a
2
+ b
2
+ 3ab)c
3
+ a
3
b
3
2a
2
b
2
c(a + b)
abc(a
2
+ c
2
)(b
2
+ c
2
)

abc
2
(a
2
+ b
2
) + (a + b)
3
c
3
+ a
3
b
3
2a
2
b
2
c(a + b)
abc(a
2
+ c
2
)(b
2
+ c
2
)

a
3
b
3
(a
2
+b
2
)
4(a+b)
2
+ (a + b)
3
c
3
+ a
3
b
3
2a
2
b
2
c(a + b)
abc(a
2
+ c
2
)(b
2
+ c
2
)

(a + b)
3
c
3
+
9
16
a
3
b
3
+
9
16
a
3
b
3
2a
2
b
2
c(a + b)
abc(a
2
+ c
2
)(b
2
+ c
2
)

3
3
_
81
256
a
2
b
2
c(a + b) 2a
2
b
2
c(a + b)
abc(a
2
+ c
2
)(b
2
+ c
2
)
0
Bt ng thc c chng minh xong. ng thc xy ra khi v ch khi a = b = c.

94 Tm gi tr ln nht ca biu thc


P = (a b)(b c)(c a)(a + b + c)
vi a, b, c 0 tha a
2
+ b
2
+ c
2
= 1.
Li gii. Nu a b c 0, ta c P 0. Nu c b a 0 th
P = (c b)(b a)(c a)(a + b + c) = (c b)(b a)(c
2
+ bc a
2
ab)
b(c b)(c
2
+ bc) = (c
2
bc)(b
2
+ bc)
1
4
(b
2
+ c
2
)
2

1
4
Cho a = 0, b = sin

8
, c = cos

8
, ta c P =
1
4
. Vy
max P =
1
4
.

95 Vi mi s dng a, b, c, d,
b(a + c)
c(a + b)
+
c(b + d)
d(b + c)
+
d(c + a)
a(c + d)
+
a(d + b)
b(d + a)
4
Li gii. Bt ng thc c vit li nh sau
(a + c)
_
b
c(a + b)
+
d
a(c + d)
_
+ (b + d)
_
c
d(b + c)
+
a
b(d + a)
_
4
Hay
(abc + abd + acd + bcd)
_
a + c
ac(a + b)(c + d)
+
b + d
bd(b + c)(d + a)
_
4
_
1
a
+
1
b
+
1
c
+
1
d
_
_
1
a
+
1
c
_
1
a
+
1
b
_ _
1
c
+
1
d
_ +
1
b
+
1
d
_
1
b
+
1
c
_ _
1
d
+
1
a
_
_
4
www.VNMATH.com
98 CHNG 2. SOLUTION
S dng bt ng thc AMGM, ta c
1
a
+
1
c
_
1
a
+
1
b
_ _
1
c
+
1
d
_ +
1
b
+
1
d
_
1
b
+
1
c
_ _
1
d
+
1
a
_
4
_
1
a
+
1
c
_
_
1
a
+
1
b
+
1
c
+
1
d
_
2
+
4
_
1
b
+
1
d
_
_
1
a
+
1
b
+
1
c
+
1
d
_
2
=
4
1
a
+
1
b
+
1
c
+
1
d
Bt ng thc c chng minh xong. ng thc xy ra khi v ch khi a = c, b = d.

96 Chng mnh rng vi mi s thc a, b, c th


a
2
bc
a
2
+ 2b
2
+ 3c
2
+
b
2
ca
b
2
+ 2c
2
+ 3a
2
+
c
2
ca
c
2
+ 2a
2
+ 3b
2
0
Li gii. Ta c

cyc
4(a
2
bc)
a
2
+ 2b
2
+ 3c
2
=

cyc
_
4(a
2
bc)
a
2
+ 2b
2
+ 3c
2
+ 1
_
3
= 2

cyc
(b c)
2
a
2
+ 2b
2
+ 3c
2
+

cyc
5a
2
+ c
2
a
2
+ 2b
2
+ 3c
2
3

cyc
5a
2
+ c
2
a
2
+ 2b
2
+ 3c
2
3
Ta cn chng minh

cyc
5a
2
+ c
2
a
2
+ 2b
2
+ 3c
2
3
Hay

cyc
5x + z
x + 2y + 3z
3 x, y, z > 0
S dng bt ng thc Cauchy Schwarz, ta c

cyc
5x + z
x + 2y + 3z

36(x + y + z)
2

cyc
(5x + z)(x + 2y + 3z)
=
9(x + y + z)
2
2(x
2
+ y
2
+ z
2
) + 7(xy + yz + zx)
Mt khc, ta li c
3(x + y + z)
2
2(x
2
+ y
2
+ z
2
) 7(xy + yz + zx) = x
2
+ y
2
+ z
2
xy yz zx 0
Bt ng thc c chng minh xong. ng thc xy ra khi v ch khi a = b = c.

97 Cho cc s khng m x, y, z, chng minh bt ng thc


x
4
x
4
+ x
2
yz + y
2
z
2
+
y
4
y
4
+ y
2
zx + z
2
x
2
+
z
4
z
4
+ z
2
xy + x
2
y
2
1
www.VNMATH.com
99
Li gii. S dng bt ng thc Cauchy Schwarz, ta c

cyc
x
4
x
4
+ x
2
yz + y
2
z
2

(x
2
+ y
2
+ z
2
)
2

cyc
x
4
+

cyc
x
2
yz +

cyc
y
2
z
2

(x
2
+ y
2
+ z
2
)
2

cyc
x
4
+ 2

cyc
y
2
z
2
= 1
Bt ng thc c chng minh xong.
Nhn xt. t a =
yz
x
2
, b =
zx
y
2
, c =
xy
z
2
, ta c

cyc
1
a
2
+ a + 1
1 a, b, c > 0, abc = 1 (2.4)
T y, ta li t a =
n
m
, b =
p
n
, c =
m
p
(m, n, p > 0), nh th ta c

cyc
m
2
m
2
+ mn + n
2
1 (2.5)

98 Cho cc s dng a, b, c tha abc = 1, chng minh rng


1
a
2
a + 1
+
1
b
2
b + 1
+
1
c
2
c + 1
3
Li gii. S dng 97(2.4) khi thay (a, b, c) ln lt bi
_
1
a
2
,
1
b
2
,
1
c
2
_
, ta c

cyc
a
4
a
4
+ a
2
+ 1
1
Hay

cyc
2(a
2
+ 1)
a
4
+ a
2
+ 1
4

cyc
1
a
2
+ a + 1
+

cyc
1
a
2
a + 1
4
Li s dng 97(2.4), ta c pcm. ng thc xy ra khi v ch khi a = b = c = 1.

99 Chng minh rng vi mi s dng a, b, c,


3a
2
2ab b
2
3a
2
+ 2ab + 3b
2
+
3b
2
2bc c
2
3b
2
+ 2bc + 3c
2
+
3c
2
2ca a
2
3c
2
+ 2ca + 3a
2
0
Li gii. Bt ng thc tng ng vi

cyc
_
1
3a
2
2ab b
2
3a
2
+ 2ab + 3b
2
_
3
Hay

cyc
b(a + b)
3a
2
+ 2ab + 3b
2

3
4
www.VNMATH.com
100 CHNG 2. SOLUTION
Do 3a
2
+ 2ab + 3b
2

8
3
(a
2
+ ab + b
2
) nn ta ch cn chng minh

cyc
b(a + b)
8
3
(a
2
+ ab + b
2
)

3
4
Hay

cyc
a
2
a
2
+ ab + b
2
1
y chnh l 97(2.5). Bt ng thc c chng minh xong. ng thc xy ra khi v ch khi
a = b = c.

100 Cho cc s dng a, b, c tha a


4
+ b
4
+ c
4
= 3, chng minh bt ng thc
a
2
b
3
+ 1
+
b
2
c
3
+ 1
+
c
2
a
3
+ 1

3
2
Li gii. Ta c

cyc
a
2
1 + b
3
=

cyc
a
2

cyc
a
2
b
3
1 + b
3

cyc
a
2

1
2

cyc
a
2
b
3/2

cyc
a
2

1
4

cyc
a
2
b(b + 1)
=

cyc
a
2

1
4

cyc
a
2
b
2

1
4

cyc
a
2
b

cyc
a
2

1
4

cyc
a
2
b
2

1
8

cyc
a
2
(b
2
+ 1)
=
7
8

cyc
a
2

3
8

cyc
a
2
b
2
t x = a
2
+ b
2
+ c
2
th ta c 3 x

3 v a
2
b
2
+ b
2
c
2
+ c
2
a
2
=
x
2
3
2
, ta cn chng minh
7

cyc
a
2
3

cyc
a
2
b
2
12
Hay
7x
3(x
2
3)
2
12
(x 3)(3x 5) 0 (ng)
Bt ng thc c chng minh xong. ng thc xy ra khi v ch khi a = b = c = 1.

101 Cho cc s dng a, b, c, chng minh bt ng thc


9
2

(a
2
+ b
2
+ c
2
)
3
(a + b + c)
4

a
3
a + b
+
b
3
b + c
+
c
3
c + a
Li gii. Ta c

cyc
a
3
b
3
a + b
=

cyc
(a b)((a + b)
2
ab)
a + b
=

cyc
(a b)(a + b)

cyc
ab(a b)
a + b
=
(ab + bc + ca)(a b)(b c)(c a)
(a + b)(b + c)(c + a)
www.VNMATH.com
101
Suy ra
2

cyc
a
3
a + b
=

cyc
a
3
+ b
3
a + b
+

cyc
a
3
b
3
a + b
= 2

cyc
a
2

cyc
ab +
(ab + bc + ca)(a b)(b c)(c a)
(a + b)(b + c)(c + a)
Ta phi chng minh
9(a
2
+ b
2
+ c
2
)
3
(a + b + c)
4
+

cyc
ab 2

cyc
a
2

(ab + bc + ca)(a b)(b c)(c a)


(a + b)(b + c)(c + a)
t x = a
2
+ b
2
+ c
2
, y = ab + bc + ca th ta c x y v
V T =
9x
3
(x + 2y)
2
+ y 2x =
(x y)(7x
2
4y
2
)
(x + 2y)
2

3x
2
(x y)
(x + 2y)
2

x(x y)
x + 2y
Mt khc, s dng bt ng thc AMGM, ta c
(a + b)(b + c)(c + a) = (a + b + c)(ab + bc + ca) abc
8
9
(a + b + c)(ab + bc + ca)
Ngoi ra, ta cng d thy rng ta ch cn xt bt ng thc trong trng hp c b a l . Nh
th, t cc lp lun trn, ta suy ra ta ch cn chng minh rng
x(x y)
x + 2y

9(a b)(b c)(c a)
8(a + b + c)
Hay
8(a
2
+ b
2
+ c
2
)(a
2
+ b
2
+ c
2
ab bc ca) 9(a b)(b c)(c a)(a + b + c)
1
16

cyc
(8a
2
8b
2
+ 6ab + 5bc 11ca)
2
+
101
16

cyc
c
2
(a b)
2
0
Bt ng thc cui cng hin nhin ng, vy ta c pcm. ng thc xy ra khi v ch khi a = b = c.

102 Cho cc s dng a, b, c, d tha a + b + c + d = 4, tm hng s k tt nht sao cho


1
a
+
1
b
+
1
c
+
1
d
4 k(a
2
+ b
2
+ c
2
+ d
2
4)
Li gii. Cho a = b = c =
2
3
, d = 2, ta suy ra c k
3
4
. Ta s chng minh y l gi tr cn tm, tc
l
1
a
+
1
b
+
1
c
+
1
d
1 +
3
4
(a
2
+ b
2
+ c
2
+ d
2
)
Khng mt tnh tng qut, gi s d c b a, suy ra a + b a + c 2, b + c
8
3
, t
t =
a+b+c
3
1, ta s chng minh
1
a
+
1
b
+
1
c

3
4
(a
2
+ b
2
+ c
2
)
3
t

9
4
t
2
Hay
4(a + b + c)
_
1
a
+
1
b
+
1
c
_
36 (a + b + c)(3(a
2
+ b
2
+ c
2
) (a + b + c)
2
)

cyc
x(b c)
2
0
www.VNMATH.com
102 CHNG 2. SOLUTION
vi x =
4
bc
a b c, y =
4
ca
a b c, z =
4
ab
a b c. Ta c
z =
4
ab
a b c
16
(a + b)
2
3 4 3 = 1 > 0
y =
4
ca
a b c
16
(a + c)
2
3 4 3 = 1 > 0
x + y =
4
ac
+
4
bc
2(a + b + c)
16
(a + c)
2
+
16
(b + c)
2
6 4 +
9
4
6 =
1
4
> 0
Do bt ng thc trn ng. Nh vy, chng minh bt ng thc cho, ta ch cn chng
minh
3
t
+
1
d
1 +
3
4
(3t
2
+ d
2
)
Hay
3
t
+
1
4 3t
1 +
3
4
(3t
2
+ (4 3t)
2
)
Ta c
V T V P =
3(3t 2)
2
(t 1)
2
t(4 3t)
0
Vy ta c pcm, t ta i n kt lun
k
max
=
3
4
.
Nhn xt. C th thy kt qu ny mnh hn kt qu sau ca Phm Kim Hng
1
a
2
+
1
b
2
+
1
c
2
+
1
d
2
a
2
+ b
2
+ c
2
+ d
2
Tht vy, t x = a
2
+ b
2
+ c
2
+ d
2
4, s dng bt ng thc Cauchy Schwarz, ta c
64V T 16
_

cyc
1
a
_
2
(3x + 4)
2
Li c (3x + 4)
2
64x = (x 4)(9x 4) 0 nn bt ng thc ng.

103 Cho cc s dng x, y, z tha xy + yz + zx = 1, chng minh bt ng thc


x(y + z)
2
(1 + yz)
2
+
y(z + x)
2
(1 + zx)
2
+
z(x + y)
2
(1 + xy)
2

3

3
4
Li gii. t a = yz, b = zx, c = xy th ta c a + b + c = 1 v bt ng thc tr thnh

cyc

a(b + c)
2

bc(1 + a)
2

3
4
Hay

cyc
a(1 a)
2
(1 + a)
2

3

3
4

abc

cyc
_
a(1 a)
2
(1 + a)
2
a
_
+ 1
3

3
4

abc
www.VNMATH.com
103
1
3

3
4

abc 4

cyc
a
2
(a + 1)
2
t q = ab + bc + ca, R =

3abc, s dng bt ng thc AMGM v bt ng thc Schur, ta c


q = ab + bc + ca
_
3abc(a + b + c) =

3abc = R
q = ab + bc + ca
1 + 9abc
4
=
3R
2
+ 1
4
9R
2
= 27abc (a + b + c)
3
= 1
Suy ra
3R
2
+1
4
q R v 1 3R, bt ng thc cho tng ng vi
f(q) =
12(6q
2
+ 4R
2
q + R
4
6R
2
+ 3)
(3q + R
2
+ 6)
2
+
3
4
R 1 0
Ta c
f

(q) =
216(33 26R
2
24q + R
4
)
(3q + R
2
+ 6)
4

216
_
33 26
1
9
24
1
3
_
(3q + R
2
+ 6)
4
=
4776
(3q + R
2
+ 6)
4
> 0
Suy ra f(q) l hm li, do ta c
f(q) max
_
f(R), f
_
3R
2
+ 1
4
__
Li c
f(R) =
R(3R 1)(R
3
+ 21R
2
+ 84R + 36)
4(R
2
+ 3R + 6)
2
0
f
_
3R
2
+ 1
4
_
=
(3R 1)(169R
4
+ 1719R
3
+ 546R
2
+ 81(3R 1)(3R 4))
4(13R
2
+ 27)
2
0
Bt ng thc c chng minh xong. ng thc xy ra khi v ch khi a = b = c =
1
3
hoc
a = 1, b = c = 0 v cc hon v.
Nhn xt. Bt ng thc trn c dng lng gic nh sau
sin A
cos
2
BC
2
+
sin B
cos
2
CA
2
+
sin C
cos
2
AB
2

3
2
Kt qu ny c tc gi t ra t kt qu sau sin A+sin B +sin C
3

3
2
, tt nhin cc bn c th chng
minh "dng lng gic" bng k thut dn bin trong tam gic nhng xem ra c v di hn! Li gii trn
tuy phc tp nhng li c v ngn gn!

104 Cho cc s khng m a, b, c tha a + b + c = 3, chng minh bt ng thc


_
a +
_
b
2
+ c
2
+
_
b +
_
c
2
+ a
2
+
_
c +
_
a
2
+ b
2
3
_

2 + 1
Li gii. Bnh phng hai v, ta c th vit li bt ng thc nh sau

cyc
_
a
2
+ b
2
+ 2

cyc
_
_
a +
_
b
2
+ c
2
__
b +
_
a
2
+ c
2
_
9

2 + 6
www.VNMATH.com
104 CHNG 2. SOLUTION
S dng bt ng thc Cauchy Schwarz, ta c
2

cyc
_
_
a +
_
b
2
+ c
2
__
b +
_
a
2
+ c
2
_
2

cyc

_
a +
b + c

2
__
b +
a + c

2
_
=

cyc
_
__

2 1
_
a + 3
___

2 1
_
b + 3
_

cyc
__

2 1
_

ab + 3
_
=
_
2

2
_

cyc

ab + 9

2
Nh vy, chng minh bt ng thc cho, ta ch cn chng minh bt ng thc sau vi mi
x, y 0
_
x
4
+ y
4
+
_
2

2
_
xy x
2
+ y
2
Tht vy, ta c

2(V T V P) = (x y)
2
_
(x + y)
2
_
2(x
4
+ y
4
) + x
2
+ y
2

2 + 1
_
(x y)
2
_
(x + y)
2
_
2 + 1
_
(x
2
+ y
2
)

2 + 1
_
=
_
2 1
_
xy(x y)
2
x
2
+ y
2
0
Bt ng thc c chng minh xong. ng thc xy ra khi v ch khi a = b = c = 1.

105 Cho a, b, c l di ba cnh ca mt tam gic, chng minh rng


a
3a + b c
+
b
3b + c a
+
c
3c + a b
1
Li gii. S dng bt ng thc Cauchy Schwarz, ta c

cyc
4a
3a + b c
3 =

cyc
_
4a
3a + b c
1
_
=

cyc
a b + c
3a + b c

(a + b + c)
2

cyc
(a b + c)(3a + b c)
=
(a + b + c)
2

cyc
a
2
+ 2

cyc
ab
= 1
Bt ng thc c chng minh xong. ng thc xy ra khi v ch khi a = b = c.

106 Cho cc s dng a, b, c tha a


2
+ b
2
+ c
2
= 3, chng minh bt ng thc
a
ab + 3
+
b
bc + 3
+
c
ca + 3

3
4
Li gii. Ta c bt ng thc tng ng
4abc

cyc
ab + 12

cyc
ab
2
+ 36abc + 36

cyc
a 3a
2
b
2
c
2
+ 9abc

cyc
a + 27

cyc
ab + 81
Khng mt tnh tng qut, gi s b l s hng nm gia a v c, suy ra
a(b a)(b c) 0
www.VNMATH.com
105
hay
ab
2
+ ca
2
a
2
b + abc
Nh th

cyc
ab
2
b(a
2
+ c
2
) + abc = b(a + c)
2
abc
4
27
(a + b + c)
3
abc
Nh vy, ta ch cn chng minh rng
4abc

cyc
ab + 12
_
4
27
(a + b + c)
3
abc
_
+ 36abc + 36

cyc
a 3a
2
b
2
c
2
+ 9abc

cyc
a + 27

cyc
ab + 81
t p = a +b +c, q = ab +bc +ca, r = abc th ta c p
2
2q = 3. Mt khc, s dng bt ng thc
AMGM, ta c
(p
2
3)
2
4
= q
2
3pr, bt ng thc tr thnh
f(r) = 3r
2
(2p
2
9p + 18)r
16
9
p
3
+
27
2
p
2
36p +
81
2
0
Ta c
f

(r) = 6r 2p
2
+ 9p 18
(p
2
3)
2
2p
2p
2
+ 9p 18
=
(p 1)(p 3)(p
2
+ 2) 18
2p
0
Do f(r) l hm nghch bin, suy ra
f(r) f
_
(p
2
3)
2
12p
_
=
(p 3)(3p
7
15p
6
+ 27p
5
247p
4
+ 717p
3
1953p
2
+ 621p 81)
144p
2
Mt khc, ta li c
3p
7
15p
6
+ 27p
5
247p
4
+ 717p
3
1953p
2
+ 621p 81
6p
6
+ 27p
5
247p
4
+ 717p
3
1953p
2
+ 621p 81
21p
5
247p
4
+ 717p
3
1953p
2
+ 621p 81
184p
4
+ 717p
3
1953p
2
+ 621p 81
533p
3
1953p
2
+ 621p 81
3(118p
2
207p + 27) 0 (do p

3)
Bt ng thc c chng minh xong. ng thc xy ra khi v ch khi a = b = c = 1.

107 Cho cc s khng m a, b, c, chng minh bt ng thc

a
2
b
2
+ (c + a)
2
+

b
2
c
2
+ (a + b)
2
+

c
2
a
2
+ (b + c)
2

3

5
Li gii. S dng bt ng thc Cauchy Schwarz, ta c
_

cyc

a
2
b
2
+ (c + a)
2
_2

cyc
a
(b
2
+ (c + a)
2
)(c
2
+ (a + b)
2
)
__

cyc
a(c
2
+ (a + b)
2
)
_
=
((a
2
+ b
2
+ c
2
)(a + b + c) + 6abc)((a
2
+ b
2
+ c
2
)(a + b + c) + 2(a
2
b + b
2
c + c
2
a))
(a
2
+ b
2
+ c
2
+ 2ab)(a
2
+ b
2
+ c
2
+ 2bc)(a
2
+ b
2
+ c
2
+ 2ca)
www.VNMATH.com
106 CHNG 2. SOLUTION
Khng mt tnh tng qut, gi s a + b + c = 1, khi d dng chng minh c
a
2
b + b
2
c + c
2
a
4
27
abc
Nh th, ta ch cn chng minh
(a
2
+ b
2
+ c
2
+ 6abc)
_
a
2
+ b
2
+ c
2
+
8
27
2abc
_
(a
2
+ b
2
+ c
2
+ 2ab)(a
2
+ b
2
+ c
2
+ 2bc)(a
2
+ b
2
+ c
2
+ 2ca)

9
5
t a
2
+ b
2
+ c
2
=
1+2q
2
3
, r = abc (1 q 0) th ta c
(1q)
2
(1+2q)
27
r max
_
0,
(1+q)
2
(12q)
27
_
,
bt ng thc tng ng vi
2673r
2
+ 72(3q
2
1)r + 36q
4
+ 16q
2
1 0
Nu 3q
2
1 th bt ng thc l hin nhin. Nu 1 2q
2

3
th
V T 72(3q
2
1)
(1 q)
2
(1 + 2q)
27
+ 36q
4
+ 16q
2
1
= 16q
5
+ 4q
3
_
3q
4
3
_
+
_
32q
2

11
3
_
> 0
Nu 1 2q th
V T 2673
_
(1 + q)
2
(1 2q)
27
_
2
+ 72(3q
2
1)
(1 q)
2
(1 + 2q)
27
+ 36q
4
+ 16q
2
1
=
q
2
(44q
4
+ 180q
3
+ 135q
2
+ 30(1 2q))
3
0
Bt ng thc c chng minh xong. ng thc xy ra khi v ch khi a = b = c.
Nhn xt. Ngoi ra, ta cng c kt qu sau

a
2
b
2
+ (c + a)
2
+

b
2
c
2
+ (a + b)
2
+

c
2
a
2
+ (b + c)
2
1
Tht vy, s dng bt ng thc Holder, ta c
_

cyc

a
2
b
2
+ (c + a)
2
_
2
_

cyc
a(b
2
+ (c + a)
2
)
_
(a + b + c)
3
Li c
(a + b + c)
3

cyc
a(b
2
+ (c + a)
2
) = 2

cyc
a
2
b + 6abc 0
T y ta d dng suy ra pcm. ng thc xy ra khi v ch khi (a, b, c) (1, 0, 0).

108 Cho a, b, c l di ba cnh ca mt tam gic, chng minh bt ng thc


a(a b)
a
2
+ 2bc
+
b(b c)
b
2
+ 2ca
+
c(c a)
c
2
+ 2ab
0
Li gii. Bt ng thc tng ng

cyc
_
a(a b)
a
2
+ 2bc
+ 1
_
3
www.VNMATH.com
107
Hay

cyc
2a
2
ab + 2bc
a
2
+ 2bc
3
Do a, b, c l di 3 cnh ca 1 tam gic nn c ba, do ta c 2a
2
ab+2bc 2a
2
ab+2b(ba) =
2(a b)
2
+ ab 0. Nh th, s dng bt ng thc Cauchy Schwarz, ta c

cyc
2a
2
ab + 2bc
a
2
+ 2bc

(2(a
2
+ b
2
+ c
2
) + ab + bc + ca)
2

cyc
(2a
2
ab + 2bc)(a
2
+ 2bc)
Ta cn chng minh
(2(a
2
+ b
2
+ c
2
) + ab + bc + ca)
2
3

cyc
(2a
2
ab + 2bc)(a
2
+ 2bc)
Hay
7

cyc
a
3
b + 4

cyc
ab
3
2

cyc
a
4
+ 3

cyc
a
2
b
2
+ 6

cyc
a
2
bc
Li do a, b, c l di 3 cnh ca mt tam gic nn tn ti cc s dng x, y, z sao cho a = y+z, b =
z + x, c = x + y, khi bt ng thc tr thnh
2

cyc
x
4
+ 2

cyc
xy(x
2
+ y
2
) + 3

cyc
xy
3
6

cyc
x
2
y
2
+ 3

cyc
x
2
yz
S dng bt ng thc AMGM, ta c
2

cyc
x
4
2

cyc
x
2
y
2
, 2

cyc
xy(x
2
+ y
2
) 4

cyc
x
2
y
2
, 3

cyc
xy
3
3

cyc
x
2
yz
Bt ng thc c chng minh xong. ng thc xy ra khi v ch khi a = b = c.

109 Cho cc s dng a, b, c, chng minh


_
a
2
a
2
+ 7ab + b
2
+
_
b
2
b
2
+ 7bc + c
2
+
_
c
2
c
2
+ 7ca + a
2
1
Li gii. t x =
b
a
, y =
c
b
, e =
a
c
th ta c xyz = 1, bt ng thc tr thnh

cyc
1

x
2
+ 7x + 1
1
Do x, y, z > 0 v xyz = 1 nn tn ti m, n, p > 0 sao cho x =
n
2
p
2
m
4
, y =
p
2
m
2
n
4
, z =
m
2
n
2
p
4
, ta phi
chng minh

cyc
m
4
_
m
8
+ 7m
4
n
2
p
2
+ n
4
p
4
1
S dng bt ng thc Holder, ta c
_

cyc
m
4
_
m
8
+ 7m
4
n
2
p
2
+ n
4
p
4
_
2
_

cyc
m(m
8
+ 7m
4
n
2
p
2
+ n
4
p
4
)
_
(m
3
+ n
3
+ p
3
)
3
Nh th, ta ch cn chng minh
(m
3
+ n
3
+ p
3
)
3

cyc
m(m
8
+ 7m
4
n
2
p
2
+ n
4
p
4
)
www.VNMATH.com
108 CHNG 2. SOLUTION
Hay

sym
(5m
6
n
3
+ 2m
3
n
3
p
3
7m
5
n
2
p
2
) +

sym
(m
6
n
3
m
4
n
4
p) 0
Bt ng thc cui cng hin nhin ng, vy ta c pcm. ng thc xy ra khi v ch khi a = b = c
hoc
b
a
0,
c
b
0 v cc hon v tng ng.
Nhn xt. Tng qut hn, ta c kt qu sau
_
a
2
a
2
+ kab + b
2
+
_
b
2
b
2
+ kbc + c
2
+
_
c
2
c
2
+ kca + a
2
min
_
1,
3

k + 2
_
k 2
Tht vy, theo chng minh trn r rng ta ch cn chng minh bt ng thc cho trng hp k 7, khi
s dng bt ng thc Holder, ta c
_

cyc
m
4
_
m
8
+ km
4
n
2
p
2
+ n
4
p
4
_
2
_

cyc
m(m
8
+ km
4
n
2
p
2
+ n
4
p
4
)
_
(m
3
+ n
3
+ p
3
)
3
Ta s chng minh
(k + 2)(m
3
+ n
3
+ p
3
)
3
9

cyc
m(m
8
+ km
4
n
2
p
2
+ n
4
p
4
)
Hay
k(m
3
+ n
3
+ p
3
)((m
3
+ n
3
+ p
3
)
2
9m
2
n
2
p
2
) + 2(m
3
+ n
3
+ p
3
)
3
9

cyc
m
9
9

cyc
m
4
n
4
p 0
Do k 7 v (m
3
+ n
3
+ p
3
)
2
9m
2
n
2
p
2
0 nn ta ch cn chng minh
7(m
3
+ n
3
+ p
3
)((m
3
+ n
3
+ p
3
)
2
9m
2
n
2
p
2
) + 2(m
3
+ n
3
+ p
3
)
3
9

cyc
m
9
9

cyc
m
4
n
4
p 0
Hay
(m
3
+ n
3
+ p
3
)
3

cyc
m(m
8
+ 7m
4
n
2
p
2
+ n
4
p
4
)
Bt ng thc ny c chng minh trn.

110 Cho cc s khng m a, b, c, chng minh bt ng thc


1

a
2
+ bc
+
1

b
2
+ ca
+
1

c
2
+ ab

2
_
1
a + b
+
1
b + c
+
1
c + a
_
Li gii. S dng bt ng thc Cauchy Schwarz, ta c
_

cyc
1

a
2
+ bc
_
2
=
_

cyc
_
(a + b)(a + c)
a
2
+ bc

1
_
(a + b)(a + c)
_
2

cyc
(a + b)(a + c)
a
2
+ bc
__

cyc
1
(a + b)(a + c)
_
=
2(a + b + c)
(a + b)(b + c)(c + a)
_

cyc
a(b + c)
a
2
+ bc
+ 3
_
Nh th, ta ch cn chng minh
2(a + b + c)
(a + b)(b + c)(c + a)
_

cyc
a(b + c)
a
2
+ bc
+ 3
_
2
_

cyc
1
b + c
_
2
www.VNMATH.com
109
Hay

cyc
a(b + c)
a
2
+ bc
+ 3
(a
2
+ b
2
+ c
2
+ 3ab + 3bc + 3ca)
2
(a + b)(b + c)(c + a)(a + b + c)

cyc
a(b + c)
a
2
+ bc
3
a
4
+ b
4
+ c
4
a
2
b
2
b
2
c
2
c
2
a
2
(a + b)(b + c)(c + a)(a + b + c)

cyc
(a b)(a c)
_
1
a
2
+ bc
+
1
(b + c)(a + b + c)
_
0
Khng mt tnh tng qut, gi s a b c, khi ta c a c
a
b
(b c) 0, do

cyc
(a b)(a c)
_
1
a
2
+ bc
+
1
(b + c)(a + b + c)
_

(a b)(b c)
b
_
a
_
1
a
2
+ bc
+
1
(b + c)(a + b + c)
_
b
_
1
b
2
+ ca
+
1
(c + a)(a + b + c)
__
=
c(a b)
2
(a + b)(b c)(a
2
+ b
2
ab + ac + bc)
b(a + c)(b + c)(a
2
+ bc)(b
2
+ ac)
0
Bt ng thc c chng minh xong. ng thc xy ra khi v ch khi a = b = c.

111 Cho a, b, c l di 3 cnh ca 1 tam gic, chng minh rng


3
_
a
b
+
b
c
+
c
a
3
_
2
_
b
a
+
c
b
+
a
c
3
_
Li gii. Ch rng
a
b
+
b
c
+
c
a
3 =
(a b)
2
ab
+
(a c)(b c)
ac
b
a
+
c
b
+
a
c
3 =
(a b)
2
ab
+
(a c)(b c)
bc
Do bt ng thc tng ng vi
(a b)
2
ab

_
3
ac

2
bc
_
(a c)(c b)
Vi gi s c l s hng nm gia a v b, ta c
(a b)
2
= (a c + c b)
2
4(a c)(c b) 0
Nh th, ta ch cn chng minh
4
ab

3
ac

2
bc
Hay
4c
ab
+
2
b

3
a
Nu b c a th ta c
4c
ab
+
2
b

4c
a
2
+
2
a

2a
a
2
+
2
a
=
4
a
>
3
a
Nu b c a th ta c
4c
ab
+
2
b

3
a

4c
a(a + c)
+
2
(a + c)

3
a
=
c a
a(a + c)
0
Bt ng thc c chng minh xong. ng thc xy ra khi v ch khi a = b = c.

www.VNMATH.com
110 CHNG 2. SOLUTION
112 Chng minh rng nu a, b, c l di 3 cnh ca 1 tam gic th
a
2
b
c
+
b
2
c
a
+
c
2
a
b
a
2
+ b
2
+ c
2
Li gii. Bt ng thc tng ng vi
c
2
(a b)
2
ab
+
(a
2
+ ac bc)(a c)(b c)
ac
0
T y, vi gi s c = min{a, b, c}, ta c a
2
+ac bc = a(a +c) bc b(a c) 0. Bt ng thc
c chng minh xong. ng thc xy ra khi v ch khi a = b = c.

113 Cho cc s khng m a, b, c chng minh bt ng thc


a
2
b
2
+
b
2
c
2
+
c
2
a
2
+
9(ab + bc + ca)
a
2
+ b
2
+ c
2
12
Li gii. Cch 1. Ta c b sau
B . Vi mi a, b, c > 0, ta c
2
_
a
b
+
b
c
+
c
a
_
+ 1
21(a
2
+ b
2
+ c
2
)
(a + b + c)
2
Tht vy, bt ng thc tng ng
2(a + b + c)
2
_
a
b
+
b
c
+
c
a
_
+ (a + b + c)
2
21(a
2
+ b
2
+ c
2
)
Hay

cyc
a
3
b
+ 4

cyc
ab +

cyc
c
2
a
b
+ 2

cyc
ca
2
b
8

cyc
a
2
Hay
_

cyc
a
3
b
+

cyc
ab 2

cyc
a
2
_
+
_

cyc
c
2
a
b

cyc
ab
_
+ 2
_

cyc
ca
2
b

cyc
ab
_
6
_

cyc
a
2

cyc
ab
_

cyc
S
c
(a b)
2
0
trong
S
a
=
b
c
+
2a
c
+
a
b
3, S
b
=
c
a
+
2b
a
+
b
c
3, S
c
=
a
b
+
2c
b
+
c
a
3
Khng mt tnh tng qut, ta ch cn xt bt ng thc cho trong trng hp a b c l ,
khi d thy S
a
S
c
v S
a
0, ta c
S
b
+ S
c
=
a
b
+
2b
a
+
2c(a + b)
ab
+
b
c
6
a
b
+
2b
a
+ 2
_
2(a + b)
a
6 0
S
c
+ 2S
b
=
a
b
+
4b
a
+
c(2a + 3b)
ab
+
2b
c
9
a
b
+
4b
a
+ 2
_
2(2a + 3b)
a
9 0
Do
www.VNMATH.com
111
+, Nu S
b
0, ta c (a c)
2
(a b)
2
nn
V T (S
b
+ S
c
)(a b)
2
0
+, Nu S
b
0, theo bt ng thc Cauchy Schwarz, ta c (a c)
2
(a b)
2
+ (b c)
2
nn
V T (S
a
+ 2S
b
)(b c)
2
+ (S
c
+ 2S
b
)(a b)
2
(S
c
+ 2S
b
)(b c)
2
+ (S
c
+ 2S
b
)(a b)
2
0
B c chng minh xong. ng thc xy ra khi v ch khi a = b = c.
Tr li bi ton ca ta, s dng b trn v kt qu bi ton 1.17, ta c
a
2
b
2
+
b
2
c
2
+
c
2
a
2

3
2

cyc
a + b
c
6 v
a
2
b
2
+
b
2
c
2
+
c
2
a
2

10(a
4
+ b
4
+ c
4
) a
2
b
2
b
2
c
2
c
2
a
2
(a
2
+ b
2
+ c
2
)
2
Suy ra
a
2
b
2
+
b
2
c
2
+
c
2
a
2

3
4

cyc
a + b
c
+
10(a
4
+ b
4
+ c
4
) a
2
b
2
b
2
c
2
c
2
a
2
2(a
2
+ b
2
+ c
2
)
2
3
Nh th, ta ch cn chng minh
3
4

cyc
a + b
c
+
10(a
4
+ b
4
+ c
4
) a
2
b
2
b
2
c
2
c
2
a
2
2(a
2
+ b
2
+ c
2
)
2
+
9(ab + bc + ca)
a
2
+ b
2
+ c
2
15
Khng mt tnh tng qut, gi s a +b +c = 1, t ab +bc +ca =
1q
2
3
, r = abc (1 q 0) th ta
c
(1q)
2
(1+2q)
27
r
(1+q)
2
(12q)
27
, bt ng thc tr thnh
_
1 q
2
r
+
729r
(1 + 2q
2
)
2
_
+
27r 42(1 q
2
)
2
(1 + 2q
2
)
2
+
36(1 q
2
)
1 + 2q
2
49
D dng chng minh c f(r) =
1q
2
r
+
729r
(1+2q
2
)
2
l hm nghch bin theo r, nn ta c
1 q
2
r
+
729r
(1 + 2q
2
)
2

27(1 q
2
)
(1 q)
2
(1 + 2q)
+
27(1 q)
2
(1 + 2q)
(1 + 2q
2
)
2
=
27(1 + q)
(1 q)(1 + 2q)
+
27(1 q)
2
(1 + 2q)
(1 + 2q
2
)
2
Li c
27r 42(1 q
2
)
2
(1 + 2q
2
)
2

(1 + q)
2
(1 2q) 42(1 q
2
)
2
(1 + 2q
2
)
2
Nh vy, ta ch cn chng minh
27(1 + q)
(1 q)(1 + 2q)
+
27(1 q)
2
(1 + 2q)
(1 + 2q
2
)
2
+
(1 + q)
2
(1 2q) 42(1 q
2
)
2
(1 + 2q
2
)
2
+
36(1 q
2
)
1 + 2q
2
49
Hay
2q
2
(q
2
(11q 7)
2
+ 189q
4
+ q
3
+ 36q
2
+ 1)
(1 q)(1 + 2q)(1 + 2q
2
)
2
0
Bt ng thc cui cng hin nhin ng. Vy ta c pcm. ng thc xy ra khi v ch khi
a = b = c.
Cch 2. Ta chng minh b sau
B . Vi mi x, y, z > 0 tha xyz = 1, ta c
(x + y + z)
2
+
15
2

11
4
(x + y + z + xy + yz + zx)
www.VNMATH.com
112 CHNG 2. SOLUTION
Tht vy, khng mt tnh tng qut, gi s z = min{x, y, z} suy ra t
2
= xy 1 (t > 0), t
P(x, y, z) = V T th ta c
P(x, y, z) P(t, t, z) =
_
4xy(x + y) + 8xy

xy 11xy 3
_ _
x

y
_
2
4xy
0
Li c
P(t, t, z) = P
_
t, t,
1
t
2
_
=
(5t
4
12t
3
+ t
2
+ 8t + 4)(t 1)
2
4t
4
0
B c chng minh xong. S dng kt qu ny, ta c
_
a
2
b
2
+
b
2
c
2
+
c
2
a
2
_
2

11
4

cyc
a
2
+ b
2
c
2

15
2
Nh th, ta ch cn chng minh
11

cyc
a
2
+ b
2
c
2
30 36
_
4
3(ab + bc + ca)
a
2
+ b
2
+ c
2
_
2
Khng mt tnh tng qut, gi s a + b + c = 1, t ab + bc + ca =
p
2
q
2
3
(1 q 0) v r = abc
th ta c
(1q)
2
(1+2q)
27
r 0, bt ng thc tr thnh
f(r) =
11(1 + 2q
2
)((1 q
2
)
2
18r)
27r
2
63
36(11q
2
+ 1)
2
(1 + 2q
2
)
2
0
R rng y l hm nghch bin theo r nn ta c
f(r) f
_
(1 q)
2
(1 + 2q)
27
_
=
18q
2
(8 28q + 61q
2
148q
3
+ 778q
4
+ 1112q
5
892q
6
)
(1 + 2q
2
)
2
(1 q)
2
(1 + 2q)
2
0
Bt ng thc c chng minh xong.

114 Cho cc s khng m a, b, c, chng minh bt ng thc


a
b
+
b
c
+
c
a
3
_
a
2
+ b
2
+ c
2
ab + bc + ca
_
2/3
Li gii. Ta c bt ng thc tng ng
a
3
b
3
+
b
3
c
3
+
c
3
a
3
+
3(a
3
+ b
3
+ c
3
)
abc
+
3(a
3
b
3
+ b
3
c
3
+ c
3
a
3
)
a
2
b
2
c
2
+ 6 27
_
a
2
+ b
2
+ c
2
ab + bc + ca
_
2
S dng bt ng thc Cauchy Schwarz, ta c
a
3
b
3
+
b
3
c
3
+
c
3
a
3

(a
3
+ b
3
+ c
3
)
2
a
3
b
3
+ b
3
c
3
+ c
3
a
3
Nh th, ta ch cn chng minh
(a
3
+ b
3
+ c
3
)
2
a
3
b
3
+ b
3
c
3
+ c
3
a
3
+
3(a
3
+ b
3
+ c
3
)
abc
+
3(a
3
b
3
+ b
3
c
3
+ c
3
a
3
)
a
2
b
2
c
2
+ 6 27
_
a
2
+ b
2
+ c
2
ab + bc + ca
_
2
Khng mt tnh tng qut, gi s a +b +c = 1, t ab +bc +ca =
1q
2
3
(1 q 0) v r = abc th
ta c
(1q)
2
(1+2q)
27
r 0, bt ng thc tr thnh
f(r) =
27(3r + q
2
)
2
81r
2
27(1 q
2
)r + (1 q
2
)
3
+
3q
2
r
+
(1 q
2
)((1 q
2
)
2
27r)
9r
2
+ 24
27(1 + 2q
2
)
2
(1 q
2
)
2
0
www.VNMATH.com
113
Ta c
f

(r) =
81(q
2
+ 3r)((1 q
2
)(1 + 2q
2
)(2 + q
2
) 27(1 + q
2
)r)
(81r
2
27(1 q
2
)r + (1 q
2
)
3
)
2

3
r
2

2(1 q
2
)((1 q
2
)
2
27r)
9r
3
Ta c
(1 q
2
)
2
27r (1 q
2
)
2
(1 q)
2
(1 + 2q) = q
2
(1 q)
2
0
Nh th, ta s chng minh f

(r) 0 bng cch chng minh


27(q
2
+ 3r)((1 q
2
)(1 + 2q
2
)(2 + q
2
) 27(1 + q
2
)r)
(81r
2
27(1 q
2
)r + (1 q
2
)
3
)
2

1
r
2
D chng minh c (1 q
2
)(1 + 2q
2
)(2 + q
2
)
3

3
2
nn ta ch cn chng minh
g(r) =
r
2
(q
2
+ 3r)
(81r
2
27(1 q
2
)r + (1 q
2
)
3
)
2

2
81

3
Ta c
g

(r) =
r(243r
3
+ 81(1 + q
2
)r
2
9(1 q
2
)
3
r 2q
2
(1 q
2
)
3
)
81r
2
27(1 q
2
)r + (1 q
2
)
3
)
3
D thy h(r) = 243r
3
+ 81(1 + q
2
)r
2
9(1 q
2
)
3
r 2q
2
(1 q
2
)
3
l hm li nn
h(r) max
_
h(0), h
_
(1 q)
2
(1 + 2q)
27
__
Li c
h(0) = 2q
2
(1 q
2
)
3
0
h
_
(1 q)
2
(1 + 2q)
27
_
=
(q 1)
3
(62q
6
+ 267q
5
+ 399q
4
+ 344q
3
+ 156q
2
+ 51q + 17)
81
0
Do h(r) 0, suy ra g

(r) 0, vy g(r) l hm ng bin nn


g(r) g
_
(1 q)
2
(1 + 2q)
27
_
=
(1 q)(1 + 2q)
2
(2q
3
+ 6q
2
+ 1)
729(5q
3
+ 9q
2
+ 3q + 1)
Ch rng

3 <
7
4
nn ta ch cn chng minh rng
(1 q)(1 + 2q)
2
(2q
3
+ 6q
2
+ 1)
729(5q
3
+ 9q
2
+ 3q + 1)

8
567
Hay
56q
6
+ 168q
5
42q
4
+ 248q
3
+ 606q
2
+ 195q + 65
5103(5q
3
+ 9q
2
+ 3q + 1)
0
Bt ng thc ny hin nhin ng, do f(r) l hm nghch bin, suy ra
f(r) f
_
(1 q)
2
(1 + 2q)
27
_
=
27q
2
(79q
8
+ 140q
7
+ 67q
6
+ 52q
5
7q
4
14q
3
+ 4q
2
+ 2q + 1)
(1 q)
3
(q + 1)
2
(2q + 1)
2
(5q
3
+ 9q
2
+ 3q + 1)
0
Bt ng thc c chng minh xong. Vy ta c pcm. ng thc xy ra khi v ch khi a = b = c.

115 Cho cc s khng m a, b, c, chng minh bt ng thc


a
b
+
b
c
+
c
a
2
3

9(a
3
+ b
3
+ c
3
)
(a + b)(b + c)(c + a)
www.VNMATH.com
114 CHNG 2. SOLUTION
Li gii. S dng kt qu bi ton 1.113, ta ch cn chng minh
11
4

cyc
a + b
c

15
2
4
_
9(a
3
+ b
3
+ c
3
)
(a + b)(b + c)(c + a)
_
2/3
Khng mt tnh tng qut, gi s a +b +c = 1, t ab +bc +ca =
1q
2
3
(1 q 0) v r = abc th
ta c
(1q)
2
(1+2q)
27
r 0, bt ng thc tr thnh f(r) g(r) vi
f(r) =
11(1 q
2
)
12r

63
4
v g(r) = 36
_
q
2
+ 3r
1 q
2
3r
_
2/3
R rng f(r) l hm nghch bin theo r v g(r) l hm ng bin theo r nn ta ch cn chng minh
f
_
(1 q)
2
(1 + 2q)
27
_
g
_
(1 q)
2
(1 + 2q)
27
_
t x =
1+2q
1q
1 th bt ng thc tng ng
11x
2
4x + 11
2x
4
_
9(x
3
+ 2)
2(x + 1)
2
_
2/3
Hay
h(x) = ln
(11x
2
4x + 11)
3
(x + 1)
4
x
3
(x
3
+ 2)
2
7 ln 2 4 ln 3 0
Ta c
h

(x) =
(x 1)(11x
5
14x
4
45x
3
+ 10x
2
+ 44x + 66)
x(x + 1)(x
3
+ 2)(11x
2
4x + 11)
T y, ta c th d dng kim tra c bt ng thc trn. Bt ng thc c chng minh xong.
ng thc xy ra khi v ch khi a = b = c.

116 Cho cc s khng m x, y, z tha x + y


2
+ z
2
= 1, chng minh bt ng thc
x
3
x
2
+ xy + y
2
+
y
3
y
2
+ yz + z
2
+
z
3
z
2
+ zx + x
2

1
2
Li gii. Ta c
3V T =

cyc
3(x
3
+ y
3
)
2(x
2
+ xy + y
2
)
=

cyc
(x y)
2
_
x + y
x
2
+ xy + y
2

1
_
3(x
2
+ y
2
+ z
2
) + x + y + z
_
+
_
3(x
2
+ y
2
+ z
2
)

_
3(x
2
+ y
2
+ z
2
)
Mt khc, ta c
x
2
+ y
2
+ z
2
=
_
x
1
2
_
2
+ x + y
2
+ z
2

1
4
=
_
x
1
2
_
2
+
3
4

3
4
Suy ra 3V T
3
2
, tc l V T
1
2
. Bt ng thc c chng minh xong. ng thc xy ra khi v
ch khi a = b = c.

www.VNMATH.com
115
117 Cho a, b, c l di 3 cnh ca mt tam gic, chng minh bt ng thc
a
2
+ b
2
a
2
+ c
2
+
b
2
+ c
2
b
2
+ a
2
+
c
2
+ a
2
c
2
+ b
2

a + b
a + c
+
b + c
b + a
+
c + a
c + b
Li gii. Khng mt tnh tng qut, gi s c = min{a, b, c}, ta c
a
2
+ b
2
a
2
+ c
2
+
b
2
+ c
2
b
2
+ a
2
+
c
2
+ a
2
c
2
+ b
2
3 =
(a
2
b
2
)
2
(a
2
+ c
2
)(b
2
+ c
2
)
+
(a
2
c
2
)(b
2
c
2
)
(a
2
+ b
2
)(a
2
+ c
2
)
a + b
a + c
+
b + c
b + a
+
c + a
c + b
3 =
(a b)
2
(a + c)(b + c)
+
(a c)(b c)
(a + b)(a + c)
Nh th, bt ng thc tng ng vi
(a b)
2
_
(a + b)
2
(a
2
+ c
2
)(b
2
+ c
2
)

1
(a + c)(b + c)
_
+ (a c)(b c)
_
(a + c)(b + c)
(a
2
+ b
2
)(a
2
+ c
2
)

1
(a + b)(a + c)
_
0
Ta c
(a + b)
2
(a
2
+ c
2
)(b
2
+ c
2
)

1
(a + c)(b + c)

(a + b)
2
(a + c)
2
(b + c)
2

1
(a + c)(b + c)
=
(a + b)
2
(a + c)(b + c)
(a + c)
2
(b + c)
2
0
Ta phi chng minh
(a + c)
2
(a + b)(b + c)
(a
2
+ b
2
)(a
2
+ c
2
)
1
Do (a + c)
2
a
2
+ c
2
nn ta ch cn chng minh
(a + b)(b + c) a
2
+ b
2
Nu b a c th ta c (b + c)(a + b) b(a + b) = b
2
+ ab a
2
+ b
2
. Nu a b c th ta c
(b + c)(a + b) a(a + b) = a
2
+ ab a
2
+ b
2
Bt ng thc c chng minh xong. ng thc xy ra khi v ch khi (a, b, c) (1, 1, 1) hoc
(a, b, c) (1, 1, 0).

118 Cho a, b, c l di 3 cnh ca mt tam gic, chng minh rng


3(a
3
b + b
3
c + c
3
a) (a
2
+ b
2
+ c
2
)(ab + bc + ca)
Li gii. Khng mt tnh tng qut gi s c l s hng nm gia a v b, suy ra 2c max{a, b, c}.
Ch rng
a
3
b + b
3
c + c
3
a abc(a + b + c) = c(a b)
2
(a + b) + a(a + c)(a c)(b c)
ab
3
+ bc
3
+ ca
3
abc(a + b + c) = c(a b)
2
(a + b) + b(b + c)(a c)(b c)
Nn bt ng thc tng ng vi
c(a b)
2
(a + b) (a c)(c b)(2a(a + c) b(b + c))
www.VNMATH.com
116 CHNG 2. SOLUTION
Do c l s hng nm gia a v b nn theo bt ng thc AMGM, ta c
(a b)
2
= ((a c) + (c b))
2
4(a c)(c b) 0
Nh th, ta cn chng minh
4c(a + b) 2a(a + c) b(b + c)
Hay
b
2
+ 5bc + 2a(c a) 0
Nu b c a th bt ng thc l hin nhin, nu a c b th
b
2
+ 5bc + 2a(c a) (a c)
2
+ 5(a c)c 2a(a c) = (a c)(4c a) 0
Bt ng thc c chng minh xong. ng thc xy ra khi v ch khi a = b = c.

119 Cho cc s thc a, b, c, chng minh bt ng thc


15a
2
b
2
c
2
+ 12(a
4
+ b
4
+ c
4
)(a
2
+ b
2
+ c
2
) 11(a
6
+ b
6
+ c
6
) + 30abc(a
3
+ b
3
+ c
3
)
Li gii. Ch rng abc(a
3
+b
3
+c
3
) |a||b||c|(|a|
3
+|b|
3
+|c|
3
) nn khng mt tnh tng qut, ta ch
cn xt a, b, c 0 l . Ta c bt ng thc tng ng

cyc
a
6
+ 12

cyc
c
4
(a
2
+ b
2
) + 15a
2
b
2
c
2
30

cyc
a
4
bc 0
Hay
_

cyc
a
6
3a
2
b
2
c
2
_
+ 12

cyc
c
4
(a b)
2
6abc
_

cyc
a
3
3abc
_
0
D thy

cyc
a
6
3a
2
b
2
c
2
=
1
2
(a
2
+ b
2
+ c
2
)

cyc
(a
2
b
2
)
2

cyc
a
3
3abc =
1
2
(a + b + c)

cyc
(a b)
2
Nn bt ng thc tng ng

cyc
(a b)
2
((a + b)
2
(a
2
+ b
2
+ c
2
) + 24c
4
6abc(a + b + c)) 0
Nh th, ta ch cn chng minh rng
(a + b)
2
(a
2
+ b
2
+ c
2
) + 24c
4
6abc(a + b + c) 0
t 2t = a + b th ta c
(a + b)
2
(a
2
+ b
2
+ c
2
) + 24c
4
6abc(a + b + c) 4t
2
(2t
2
+ c
2
) + 24c
4
6t
2
c(2t + c)
= 2(4t
4
6t
3
c t
2
c
2
+ 12c
4
)
= 2(t
2
(t 3c)
2
+ 3(t
2
2c
2
)
2
+ 2t
2
c
2
) 0
Bt ng thc c chng minh xong. ng thc xy ra khi v ch khi a = b = c.

www.VNMATH.com
117
120 Cho cc s khng m a, b, c, d tha a + b + c + d = 3, chng minh bt ng thc
ab(b + c) + bc(c + d) + cd(d + a) + da(a + b) 4
Li gii. Do vai tr hon v vng quanh nn khng mt tnh tng qut, ta c th gi s b = max{a, b, c, d},
khi ta c
ab(b + c) + bc(c + d) + cd(d + a) + da(a + b) ab
2
+ b(c + d)
2
+ (c + d)a
2
+ ab(c + d)
Tht vy, bt ng thc tng ng
cd(b a) + d
2
(b c) + a
2
c 0
Bt ng thc ny hin nhin ng do b = max{a, b, c, d}. Nh th ta ch cn chng minh vi mi
x, y, z 0 tha x + y + z = 3 th
xy
2
+ yz
2
+ zx
2
+ xyz 4
Khng mt tnh tng qut gi s y l s hng nm gia x v z, khi ta c
x(y z)(y x) 0
Suy ra xy
2
+ zx
2
xyz + x
2
y, do
xy
2
+ yz
2
+ zx
2
+ xyz y(x + z)
2
4
_
y + (x + z)
3
_
3
= 4
Bt ng thc c chng minh xong. ng thc xy ra khi v ch khi (a, b, c, d) = (1, 2, 0, 0) v
cc hon v tng ng.

121 Cho a, b, c l cc s khn m tha a


2
+ b
2
+ c
2
= 1, chng minh rng
_
1
_
a + b
2
_
2
__
1
_
b + c
2
_
2
__
1
_
c + a
2
_
2
_

8
27
Li gii. t a +b +c = q, ab +bc +ca = q, abc = r khi ta c p
2
= 2q +1, q 1. Mt khc, theo bt
ng thc Schur r
q(4qp
2
)
9
=
q(2q1)
9
. Bt ng thc tr thnh
19
27

cyc
(a + b)
2
4
+

cyc
(a + b)
2
(a + c)
2
16

(a + b)
2
(b + c)
2
(c + a)
2
64
0
Hay
460
27
24q + 3q
2
2q
3
+ 16pr + 2pqr r
2
0
D thy y l hm ng bin theo r nn
V T
460
27
24q + 3q
2
2q
3
+
16p
2
(4q p
2
)
9
+
2p
2
q(4q p
2
)
9

p
2
(4q p
2
)
2
81
=
1
81
(1 q)(98q
2
725q + 1235) 0
Bt ng thc c chng minh xong. ng thc xy ra khi v ch khi a = b = c =
1

3
.

www.VNMATH.com
118 CHNG 2. SOLUTION
122 Cho cc s khng m a, b, c, d, chng minh bt ng thc
ab
a + b
+
bc
b + c
+
cd
c + d
+
da
d + a

_
(a + c)(b + d)
Li gii. Ta c bt ng thc tng ng

cyc
_
a + b
2

2ab
a + b
_
a + b + c + d 2
_
(a + c)(b + d)
Hay

cyc
(a b)
2
2(a + b)

(a b + c d)
2
_
a + c +

b + d
_
2
S dng bt ng thc Cauchy Schwarz, ta c
(a b)
2
2(a + b)
+
(c d)
2
2(c + d)

(a b + c d)
2
2(a + b + c + d)

(a b + c d)
2
2
_
a + c +

b + d
_
2
Tng t, ta c
(b c)
2
2(b + c)
+
(d a)
2
2(d + a)

(a b + c d)
2
2(a + b + c + d)

(a b + c d)
2
2
_
a + c +

b + d
_
2
Cng 2 bt ng thc trn li v vi v, ta c pcm. ng thc xy ra khi v ch khi a = b = c = d.

123 Chng minh rng vi mi s dng a, b, c ta c bt ng thc


a
b
+
b
c
+
c
a

_
a
2
+ c
2
b
2
+ c
2
+
_
c
2
+ b
2
a
2
+ b
2
+
_
b
2
+ a
2
c
2
+ a
2
Li gii. Trc ht, ta chng minh

cyc
a
b
+

cyc
b
a

cyc
_
a
2
+ c
2
b
2
+ c
2
+

cyc
_
b
2
+ c
2
a
2
+ c
2
Tht vy, chng minh bt ng thc trn, ta ch cn chng minh
a
b
+
b
a

_
a
2
+ c
2
b
2
+ c
2
+
_
b
2
+ c
2
a
2
+ c
2
Hay
c
2
(a
2
b
2
)
2
_
a
2
+ b
2
+ c
2
_
0 (ng)
Tr li bi ton ca ta, gi s bt ng thc cho khng ng, tc l tn ti a, b, c > 0 sao cho

cyc
a
b
<

cyc
_
a
2
+ c
2
b
2
+ c
2
Khi , theo trn,

cyc
a
b
+

cyc
b
a

cyc
_
a
2
+ c
2
b
2
+ c
2
+

cyc
_
b
2
+ c
2
a
2
+ c
2
www.VNMATH.com
119
T y v t gi thit phn chng, ta suy ra c

cyc
b
a
>

cyc
_
b
2
+ c
2
a
2
+ c
2
Mt khc, bt ng thc

cyc
a
b
<

cyc
_
a
2
+c
2
b
2
+c
2
tng ng vi
_

cyc
a
b
_
2
<
_

cyc
_
a
2
+ c
2
b
2
+ c
2
_
2
Hay

cyc
a
2
b
2
+ 2

cyc
b
a
<

cyc
a
2
+ c
2
b
2
+ c
2
+ 2

cyc
_
b
2
+ c
2
a
2
+ c
2
T y, suy ra

cyc
a
2
b
2
<

cyc
a
2
+ c
2
b
2
+ c
2
Nh vy, dn n iu mu thun, ta s chng minh bt ng thc sau

cyc
u
v

cyc
u + t
v + t
u, v, t > 0
Tht vy, t x =
u
v
, y =
v
t
, z =
t
u
th ta c x, y, z > 0 v xyz = 1. Khi , ta c
u + t
v + t
=
1 + xy
1 + y
= x +
1 x
1 + y
,
t + v
u + v
= z +
1 z
1 + x
,
v + u
t + u
= y +
1 y
1 + z
Do bt ng thc tng ng
x 1
y + 1
+
y 1
z + 1
+
z 1
x + 1
0
Hay

cyc
(x 1)(z + 1)(x + 1) 0

cyc
xy
2
+

cyc
x
2

cyc
x + 3
S dng bt ng thc AMGM v bt ng thc Cauchy Schwarz, ta c

cyc
xy
2
3xyz = 3,

cyc
x
2

cyc
x
Bt ng thc c chng minh xong. ng thc xy ra khi v ch khi a = b = c.
Nhn xt. Ta c kt qu sau vi mi a, b, c, x > 0
a
b
+
b
c
+
c
a
=
_
a
x
+ c
x
b
x
+ c
x
_
1/x
+
_
c
x
+ b
x
a
x
+ b
x
_
1/x
+
_
b
x
+ a
x
c
x
+ a
x
_
1/x
chng minh kt qu ny, ta s dng mt kt qu "rt p" sau
Nu a, b, c, x, y, z l cc s dng tha mn abc = xyz, max {a, b, c} max {x, y, z} , min {a, b, c} min {x, y, z}
th
a + b + c x + y + z
www.VNMATH.com
120 CHNG 2. SOLUTION
Tht vy, khng mt tnh tng qut, gi s a b c, x y z, suy ra a x, c z. Do c z nn ab xy.
Do , s dng bt ng thc AMGM, ta c
a + b + c x y z = (x y)
_
a
x
1
_
+ (y z)
_
a
x
+
b
y
2
_
+ z
_
a
x
+
b
y
+
c
z
3
_
(y z)
_
2

ab
xy
2
_
+ z
_
3
3

abc
xyz
3
_
0
Suy ra
a + b + c x + y + z
T kt qu ny, vi ch rng
a
b

b
c

c
a
=
_
a
x
+ c
x
b
x
+ c
x
_
1/x

_
c
x
+ b
x
a
x
+ b
x
_
1/x

_
b
x
+ a
x
c
x
+ a
x
_
1/x
= 1
max
_
a
b
,
b
c
,
c
a
_
max
_
_
a
x
+ c
x
b
x
+ c
x
_
1/x
,
_
c
x
+ b
x
a
x
+ b
x
_
1/x
,
_
b
x
+ a
x
c
x
+ a
x
_
1/x
_
min
_
a
b
,
b
c
,
c
a
_
max
_
_
a
x
+ c
x
b
x
+ c
x
_
1/x
,
_
c
x
+ b
x
a
x
+ b
x
_
1/x
,
_
b
x
+ a
x
c
x
+ a
x
_
1/x
_
Ta i n kt qu nh trn.

124 Cho cc s khng m a, b, c tha a + b + c = 5, chng minh bt ng thc


16(a
3
b + b
3
c + c
3
a) + 640 11(ab
3
+ bc
3
+ ca
3
)
Li gii. Khng mt tnh tng qut, ta ch cn xt trng hp a b c l . Khi , bt ng thc
tng ng
f(a) = 16(a
3
b + b
3
c + c
3
a) +
128
125
(a + b + c)
4
11(ab
3
+ bc
3
+ ca
3
) 0
Ta c
f

(a) = 16(3a
2
b + c
3
) +
512
125
(a + b + c)
3
11(b
3
+ 3a
2
c) = g(c)
Li c
g

(c) = 48c
2
+
1536
125
(a + b + c)
2
33a
2
0
Suy ra g(c) l hm ng bin. Do ,
f

(a) = g(c) g(b) = 5b


3
+ 15a
2
b +
128
125
(a + 2b)
3
0
Do f(a) l hm ng bin. Vy,
f(a) f(0) =
1
125
(4b c)
2
(8b
2
+ 16bc + 128c
2
) 0
Bt ng thc c chng minh xong. ng thc xy ra khi v ch khi a = 0, b = 1, c = 4, v cc
hon v tng ng.

125 Cho cc s dng a, b, c, chng minh bt ng thc


1
a + b + c

_
1
a + b
+
1
b + c
+
1
c + a
_

1
ab + bc + ca
+
1
2(a
2
+ b
2
+ c
2
)
www.VNMATH.com
121
Li gii. Ta c bt ng thc cn chng minh tng ng vi

cyc
ab + bc + ca
b + c

cyc
a +
(a + b + c)(ab + bc + ca)
2(a
2
+ b
2
+ c
2
)
Hay

cyc
bc
b + c

(a + b + c)(ab + bc + ca)
2(a
2
+ b
2
+ c
2
)
S dng bt ng thc Cauchy Schwarz, ta c

cyc
bc
b + c

(ab + bc + ca)
2
bc(b + c) + ca(c + a) + ab(a + b)
Do , ta ch cn chng minh
2
_

cyc
a
2
__

cyc
bc
_

cyc
a
__

cyc
bc(b + c)
_
Hay

cyc
ab(a
2
+ b
2
) 2

cyc
a
2
b
2
Bt ng thc ny hin nhin ng theo bt ng thc AMGM.
Bt ng thc c chng minh xong. ng thc xy ra khi v ch khi (a, b, c) (1, 1, 1) hoc
(a, b, c) (1, 1, 0).

126 Chng minh rng vi mi s khng m a, b, c, d ta c


1
a
3
+ b
3
+
1
a
3
+ c
3
+
1
a
3
+ d
3
+
1
b
3
+ c
3
+
1
b
3
+ d
3
+
1
c
3
+ d
3

243
2(a + b + c + d)
3
Li gii. Khng mt tnh tng qut, gi s a b c d 0. Khi , ta c
a
3
+ b
3

_
a +
d
3
_
3
+
_
b +
d
3
_
3
Suy ra
1
a
3
+ b
3

1
_
a +
d
3
_
3
+
_
b +
d
3
_
3
Tng t, ta c
1
a
3
+ c
3

1
_
a +
d
3
_
3
+
_
c +
d
3
_
3
,
1
b
3
+ c
3

1
_
b +
d
3
_
3
+
_
c +
d
3
_
3
Mt khc, d thy
_
a +
d
3
_
3
a
3
+ d
3
Suy ra
1
a
3
+ d
3

1
_
a +
d
3
_
3
Tng t
1
b
3
+ d
3

1
_
b +
d
3
_
3
,
1
c
3
+ d
3

1
_
c +
d
3
_
3
www.VNMATH.com
122 CHNG 2. SOLUTION
Do
V T

cyc
1
x
3
+

cyc
1
x
3
+ y
3
trong x = a +
d
3
, y = b +
d
3
, z = c +
d
3
.
Ta cn chng minh

cyc
1
x
3
+

cyc
1
x
3
+ y
3

243
2(x + y + z)
3
Hay

cyc
_
1
x
3
+
1
y
3
+
2
x
3
+ y
3
_

243
(x + y + z)
3
S dng bt ng thc AMGM, ta c
1
x
3
+
1
y
3
+
2
x
3
+ y
3
3
3

2
x
3
y
3
(x
3
+ y
3
)
= 3
3

2
3
y
3
(x
2
xy + y
2
)(x + y)
3
3

_
2
_
3xy+(x
2
xy+y
2
)
4
_
4
(x + y)
=
24
(x + y)
3
Tng t
1
y
3
+
1
z
3
+
2
y
3
+ z
3

24
(y + z)
3
,
1
z
3
+
1
x
3
+
2
z
3
+ x
3

24
(z + x)
3
Suy ra

cyc
_
1
x
3
+
1
y
3
+
2
x
3
+ y
3
_
24

cyc
1
(x + y)
3

84
(x + y)(y + z)(z + x)

243
(x + y + z)
3
Bt ng thc c chng minh xong. ng thc xy ra khi v ch khi (a, b, c, d) (1, 1, 1, 0).

127 Chng minh rng vi mi s khng m a, b, c, d ta c


1
a
2
+ b
2
+ c
2
+
1
b
2
+ c
2
+ d
2
+
1
c
2
+ d
2
+ a
2
+
1
d
2
+ a
2
+ b
2

12
(a + b + c + d)
2
Li gii. Khng mt tnh tng qut, gi s a b c d 0. Khi , ta c
_
a +
c + d
2
_
2
+
_
b +
c + d
2
_
2
(a
2
+ b
2
+ c
2
) =
(2(a + b) + c + d)(c + d)
2
c
2
0
Suy ra
1
a
2
+ b
2
+ c
2

1
_
a +
c+d
2
_
2
+
_
b +
c+d
2
_
2
Tng t, ta c
1
d
2
+ a
2
+ b
2

1
_
a +
c+d
2
_
2
+
_
b +
c+d
2
_
2
Li c
_
a +
c + d
2
_
2
(c
2
+ d
2
+ a
2
) = a(c + d) c
2
d
2
+
(c + d)
2
4
0
www.VNMATH.com
123
Suy ra
1
c
2
+ d
2
+ a
2

1
_
a +
c+d
2
_
2
Tng t
1
d
2
+ a
2
+ b
2

1
_
b +
c+d
2
_
2
Do
V T
1
_
a +
c+d
2
_
2
+
1
_
b +
c+d
2
_
2
+
2
_
a +
c+d
2
_
2
+
_
b +
c+d
2
_
2
t u = a +
c+d
2
, v = b +
c+d
2
th ta c u, v 0. Ta cn chng minh
1
u
2
+
1
v
2
+
2
u
2
+ v
2

12
(u + v)
2
S dng bt ng thc AMGM v bt ng thc Cauchy Schwarz, ta c
1
u
2
+
1
v
2
+
2
u
2
+ v
2

2
u
2
+ v
2
+
2
uv
= 2
_
1
u
2
+ v
2
+
1
2uv
_
+
1
uv

8
u
2
+ v
2
+ 2uv
+
4
(u + v)
2
=
12
(u + v)
2
Bt ng thc c chng minh xong. ng thc xy ra khi v ch khi (a, b, c, d) (1, 1, 0, 0).
Nhn xt. Bng cch lm hon ton tng t, bn hy gii bi ton sau: Tm gi tr nh nht nht ca
biu thc
P(a, b, c, d) =

cyc
1
a
n
+ b
n
+ c
n
vi a, b, c, d 0, a + b + c + d = 2, n
ln 2
ln 3ln 2
.

128 Cho cc s dng a, b, c, chng minh bt ng thc


_
a(b + c)
a
2
+ bc
+
_
b(c + a)
b
2
+ ca
+
_
c(a + b)
c
2
+ ab

a +

b +

c
_
_
1

a
+
1

b
+
1

c
_
Li gii. Bnh phng hai v ca bt ng thc trn, ta c bt ng thc tng ng
_

cyc
_
a(b + c)
a
2
+ bc
_
2

cyc

a
__

cyc
1

a
_
Hay

cyc
c(a + b)
c
2
+ ab
+ 2

cyc

ab(a + c)(b + c)
(a
2
+ bc)(b
2
+ ac)
3 +

cyc
a + b

ab
Ta c
(a
2
+ bc)(b
2
+ ac) ab(a + c)(b + c) = c(a b)
2
(a + b) 0
Suy ra

ab(a + c)(b + c)
(a
2
+ bc)(b
2
+ ac)
1
V nh th

cyc

ab(a + c)(b + c)
(a
2
+ bc)(b
2
+ ac)
3
www.VNMATH.com
124 CHNG 2. SOLUTION
Do , ta ch cn chng minh

cyc
a + b

ab

cyc
c(a + b)
c
2
+ ab
+ 3
Nhng bt ng thc ny hin nhin ng v theo bt ng thc AMGM, ta c

cyc
a + b

ab

cyc
c(a + b)
c
2
+ ab
3 =

cyc
_
a + b

ab

c(a + b)
c
2
+ ab
1
_

cyc
_
a + b

ab

c(a + b)
2

abc
2
1
_
=

cyc
_

b
_
2
2

ab
0
Bt ng thc c chng minh xong. ng thc xy ra khi v ch khi a = b = c.

129 Chng minh rng vi mi s dng a, b, c th


a
2
bc

a
2
+ 2b
2
+ 3c
2
+
b
2
ca

b
2
+ 2c
2
+ 3a
2
+
c
2
ab

c
2
+ 2a
2
+ 3b
2
0
Li gii. S dng bt ng thc Cauchy Schwarz, ta c

cyc
8(a
2
bc)
_
6(a
2
+ 2b
2
+ 3c
2
)
=

cyc
_
8(a
2
bc)
_
6(a
2
+ 2b
2
+ 3c
2
)
+ b + c
_
2

cyc
a
=

cyc
8(a
2
bc) + (b + c)
_
6(a
2
+ 2b
2
+ 3c
2
)
_
6(a
2
+ 2b
2
+ 3c
2
)
2

cyc
a

cyc
8(a
2
bc) + (b + c)(a + 2b + 3c)
_
6(a
2
+ 2b
2
+ 3c
2
)
2

cyc
a
=

cyc
8a
2
+ ab + bc + ca + c
2
_
6(a
2
+ 2b
2
+ 3c
2
)
+ 2

cyc
(b c)
2
_
6(a
2
+ 2b
2
+ 3c
2
)
2

cyc
a
Do , chng minh bt ng thc cho, ta ch cn chng minh

cyc
8a
2
+ ab + bc + ca + c
2

a
2
+ 2b
2
+ 3c
2
2

cyc
a
S dng bt ng thc Holder, ta c
V T
2
_
_
11
_

cyc
a
2
_
2
+ 21

cyc
a
2
b
2
+ 6
_

cyc
a
2
__

cyc
ab
_
_
_
= V T
2
_

cyc
(8a
2
+ ab + bc + ca + c
2
)(a
2
+ 2b
2
+ 3c
2
)
_

cyc
(8a
2
+ ab + bc + ca + c
2
)
_
3
= 27
_
3

cyc
a
2
+

cyc
ab
_
3
www.VNMATH.com
125
Do , ta ch cn chng minh
9
_
3

cyc
a
2
+

cyc
ab
_
3
8
_

cyc
a
_
2
_
_
11
_

cyc
a
2
_
2
+ 21

cyc
a
2
b
2
+ 6
_

cyc
a
2
__

cyc
ab
_
_
_
Do y l mt bt ng thc ng bc vi a, b, c nn khng mt tnh tng qut, gi s a+b+c = 1.
t q = ab + bc + ca, r = abc th ta c
1
3
q 9r 0. Ngoi ra, s dng bt ng thc Schur, ta
c r
4q1
9
. Bt ng thc trn tr thnh
9(3 5q)
3
8(11(1 2q)
2
+ 21(q
2
2r) + 6q(1 2q))
Hay
1125q
3
+ 1601q
2
911q + 336r + 155 0
Bt ng thc ny hin nhin ng v
1125q
3
+ 1601q
2
911q + 336r + 155 1125q
3
+ 1601q
2
911q + 336
4q 1
9
+ 155
=
1
3
(1 3q)(1125q
2
1226q + 353) 0
Vy bt ng thc cn chng minh ng. ng thc xy ra khi v ch khi a = b = c.

130 Cho cc s dng a, b, c tha a + b + c = 1, chng minh bt ng thc


_
1
a
2
_
2
+
_
1
b
2
_
2
+
_
1
c
2
_
2

8(a
2
+ b
2
+ c
2
)
2
(1 a)(1 b)(1 c)
Li gii. t x = a
2
+ b
2
+ c
2
th d thy 1 x
1
3
, do (x 1)(3x 1) 0, suy ra 4x 1 3x
2
.
Ta li c

cyc
a
2
b
2
abc

cyc
a = abc
Do
(4x 1)
_

cyc
a
2
b
2
_
3abcx
2
Mt khc, ta li c
4x 1 =

cyc
(b + c a)
2
S dng bt ng thc Chebyshev, ta c
3

cyc
b
2
c
2
(b + c a)
2

cyc
(b + c a)
2
__

cyc
b
2
c
2
_
Do

cyc
b
2
c
2
(b + c a)
2
abcx
2
Hay

cyc
b
2
c
2
(1 2a)
2
abcx
2

cyc
_
1
a
2
_
2

x
2
abc
www.VNMATH.com
126 CHNG 2. SOLUTION
Theo bt ng thc AMGM th
(1 a)(1 b)(1 c) = (a + b)(b + c)(c + a) 8abc
Do

cyc
_
1
a
2
_
2

x
2
abc

8x
2
(1 a)(1 b)(1 c)
=
8
_

cyc
a
2
_
2
(1 a)(1 b)(1 c)
Bt ng thc c chng minh xong. ng thc xy ra khi v ch khi a = b = c =
1
3
.

131 Cho cc s khng m a, b, c, d tha a + b + c + d = 1, chng minh bt ng thc

a
4
b
4
+ c
4
d
4
2a
2
c
2
+ 2b
2
d
2
+ 4ab
2
c + 4cd
2
a 4bc
2
d 4da
2
b

1
Li gii. Ta c bt ng thc cn chng minh tng ng

_
a
2
c
2
_
2

_
b
2
d
2
_
2
+ 4ac(b d)
2
4bd(a c)
2

1
Hay
1 f(a, b, c, d) 1
trong f(a, b, c, d) =
_
a
2
c
2
_
2

_
b
2
d
2
_
2
+ 4ac(b d)
2
4bd(a c)
2
. Khng mt tnh tng
qut, gi s a + c b + d th ta c 1 b + d
1
2
a + c 0. Khi , ta c
f(a, b, c, d) f
_
a + c
2
, b,
a + c
2
, d
_
= (a c)
2
((a + c)
2
(b + d)
2
) 0
Hay
f(a, b, c, d) f
_
a + c
2
, b,
a + c
2
, d
_
= (b d)
2
((a + c)
2
(b + d)
2
) 0
Tng t, ta c
f(a, b, c, d) f(a + c, b, 0, d) = 4ac((b + d)
2
(a + c)
2
) 0
Suy ra
f(a, b, c, d) f(a + c, b, 0, d) = (a + c)
4
(b
2
d
2
)
2
4bd(a + c)
2
= (1 S)
4
S
2
(S
2
4P) 4P(1 S)
2
= 4S
2
(1 S) + 2(S 1)
2
+ 4P(2S 1) 1 1
Bt ng thc c chng minh xong. ng thc xy ra khi v ch khi (a, b, c, d) = (1, 0, 0, 0).

132 Cho cc s dng a, b, c, chng minh bt ng thc


ab(a
2
+ bc)
b + c
+
bc(b
2
+ ca)
c + a
+
ca(c
2
+ ab)
a + b

_
3abc(ab
2
+ bc
2
+ ca
2
)
Li gii. S dng bt ng thc AMGM, ta c
_

cyc
ab(a
2
+ bc)
b + c
_
2
3

cyc
_
ab(a
2
+ bc)
b + c

bc(b
2
+ ca)
c + a
_
= 3abc

cyc
b(a
2
+ bc)(b
2
+ ca)
(a + c)(b + c)
www.VNMATH.com
127
Do , chng minh bt ng thc cho, ta ch cn chng minh

cyc
b(a
2
+ bc)(b
2
+ ca)
(a + c)(b + c)
ab
2
+ bc
2
+ ca
2
Hay

cyc
_
b(a
2
+ bc)(b
2
+ ca)
(a + c)(b + c)
ab
2
_
0

cyc
bc(a b)
2
(a + b)
(a + c)(b + c)
0 (ng)
Vy bt ng thc cn chng minh ng. ng thc xy ra khi v ch khi a = b = c.

133 Tm hng s a nh nht sao cho bt ng thc sau


_
x + y + z
3
_
a
_
xy + yz + zx
3
_
3a
2

(x + y)(y + z)(z + x)
8
ng vi mi s thc dng x, y, z.
Li gii. Cho x = y = 1, z 0, ta suy ra c a
3 ln 34 ln 2
2 ln 2ln 3
= a
0
1.81884... Ta s chng minh y
l gi tr cn tm, tc l chng minh
_
x + y + z
3
_
a
0
_
xy + yz + zx
3
_
3a
0
2

(x + y)(y + z)(z + x)
8
V y l mt bt ng thc ng bc vi x, y, z nn ta c th chun ha cho x + y + z = 1. t
q = ab + bc + ca, r = abc th
1
3
q 9r 0. Bt ng thc cn chng minh tr thnh
r +
8q
3a
0
2
3
3+a
0
2
q 0
Xt 2 trng hp
Trng hp 1. 1 4q 0, khi
r +
8q
3a
0
2
3
3+a
0
2
q
8q
3a
0
2
3
3+a
0
2
q = q
3a
0
2
_
8
3
3+a
0
2
q
a
0
1
2
_
q
3a
0
2
_
8
3
3+a
0
2

_
1
4
_
a
0
1
2
_
= 0
Trng hp 2.
1
3
q
1
4
, khi , p dng bt ng thc Schur, ta c r
4q1
9
0. Do
r +
8q
3a
0
2
3
3+a
0
2
q
4q 1
9
+
8q
3a
0
2
3
3+a
0
2
q =
8q
3a
0
2
3
3+a
0
2

5q + 1
9
= f(q)
Ta c
f

(q) =
4(3 a
0
)
q
a
0
1
2
.3
a
0
+3
2

5
9
D dng kim tra c f

(q) l hm ng bin, li c f

_
1
3
_
< 0 v f

_
1
4
_
> 0, do tn ti duy
nht q
0

_
1
4
,
1
3
_
sao cho f

(q
0
) = 0. T y, ta d dng kim tra c
f(q) min
_
f
_
1
4
_
, f
_
1
3
__
www.VNMATH.com
128 CHNG 2. SOLUTION
Nhng f
_
1
4
_
= f
_
1
3
_
= 0. Do
f(q) 0
Bt ng thc c chng minh hon ton. Vy
a
min
=
3 ln 3 4 ln 2
2 ln 2 ln 3
.

134 Cho cc s khng m a, b, c tha a


2
+ b
2
+ c
2
= 1, chng minh bt ng thc
1
a

1 + bc
+
b

1 + ca
+
c

1 + ab

3
2
Li gii. Trc ht, ta s chng minh

cyc
a

1 + bc
1
Tht vy, s dng bt ng thc AMGM v bt ng thc Cauchy Schwarz, ta c

cyc
a

1 + bc
=

cyc
a
2
a

1 + bc

cyc
2a
2
1 + a
2
+ bc

2(a + b + c)
2
2 + ab + bc + ca

2(a + b + c)
2
2 + 4(ab + bc + ca)
= 1
Tip theo, ta s chng minh

cyc
a

1 + bc

3
2
S dng bt ng thc Cauchy Schwarz, ta c
_

cyc
a

1 + bc
_
2

cyc
a
__

cyc
a
1 + bc
_
By gi, ta s chng minh

cyc
a
1 + bc

cyc
a
(a + b)(a + c)
Tht vy, ta c

cyc
a
(a + b)(a + c)

cyc
a
1 + bc
=

cyc
a(b
2
+ c
2
ab ac)
(1 + bc)(a + b)(a + c)
=

cyc
_
ca(c a)
(1 + bc)(a + b)(a + c)

ab(a b)
(1 + bc)(a + b)(a + c)
_
=

cyc
_
ab(a b)
(1 + ca)(b + c)(b + a)

ab(a b)
(1 + bc)(a + b)(a + c)
_
=

cyc
ab(a b)
2
(a
2
+ b
2
)
(1 + ac)(1 + bc)(a + b)(b + c)(c + a)
0
Nh th, ta c
_

cyc
a

1 + bc
_
2

cyc
a
__

cyc
a
(a + b)(a + c)
_
=
2(a + b + c)(ab + bc + ca)
(a + b)(b + c)(c + a)

9
4
www.VNMATH.com
129
Suy ra

cyc
a

1 + bc

3
2
Bi ton c gii quyt hon ton. ng thc v tri xy ra khi v ch khi(a, b, c) = (1, 0, 0),
ng thc v phi xy ra khi v ch khi (a, b, c) =
_
1

3
,
1

3
,
1

3
_
.

135 Cho a, b, c l cc s khng m, chng minh bt ng thc


_
a(b + c)
b
2
+ c
2
+
_
b(c + a)
c
2
+ a
2
+
_
c(a + b)
a
2
+ b
2

_
2 + 2

_
1 + 4

abc(a + b)(b + c)(c + a)


(a
2
+ b
2
)(b
2
+ c
2
)(c
2
+ a
2
)
Li gii. Ta c

cyc
a(b + c)
b
2
+ c
2
2 =

cyc
ab(a b)
2
(a
2
+ b
2
+ 2c
2
) + 8a
2
b
2
c
2
(a
2
+ b
2
)(b
2
+ c
2
)(c
2
+ a
2
)
0

cyc
a(b + c)
b
2
+ c
2

b(c + a)
c
2
+ a
2
1 =
2abc((a + b + c)(a
2
+ b
2
+ c
2
) abc)
(a
2
+ b
2
)(b
2
+ c
2
)(c
2
+ a
2
)
0
Suy ra,

cyc
a(b + c)
b
2
+ c
2
2

cyc
a(b + c)
b
2
+ c
2

b(c + a)
c
2
+ a
2
1
t x =
_
a(b+c)
b
2
+c
2
, y =
_
b(c+a)
c
2
+a
2
, z =
_
c(a+b)
a
2
+b
2
, th ta c x, y, z 0 v theo trn, ta c
x
2
+ y
2
+ z
2
2
x
2
y
2
+ y
2
z
2
+ z
2
x
2
1
Do
(x + y + z)
2
= x
2
+ y
2
+ z
2
+ 2(xy + yz + zx)
2 + 2(xy + yz + zx)
= 2 + 2
_
x
2
y
2
+ y
2
z
2
+ z
2
x
2
+ 2xyz(x + y + z)
2 + 2
_
1 + 2xyz(x + y + z)
Suy ra
(x + y + z)
2
2 + 2
_
1 + 2xyz(x + y + z)
Ch rng x, y, z 0 nn t y, ta c
(x + y + z)
2
4
Hay
x + y + z 2
V nh th, ta c
(x + y + z)
2
2 + 2
_
1 + 4xyz
Hay
x + y + z
_
2 + 2
_
1 + 4xyz
y chnh l bt ng thc cho, vy ta c pcm. ng thc xy ra khi v ch khi (a, b, c)
(1, 1, 0).

www.VNMATH.com
130 CHNG 2. SOLUTION
136 Cho a, b, c l cc s thc dng, chng minh rng
a
2
ab + b
2
a + b
+
b
2
bc + c
2
b + c
+
c
2
ca + a
2
c + a

3
2

a
3
+ b
3
+ c
3
a
2
+ b
2
+ c
2
Li gii. Ta c bt ng thc cn chng minh tng ng vi

cyc
4(a
2
ab + b
2
)
a + b

6(a
3
+ b
3
+ c
3
)
a
2
+ b
2
+ c
2
Hay

cyc
_
4(a
2
ab + b
2
)
a + b
(a + b)
_

6(a
3
+ b
3
+ c
3
)
a
2
+ b
2
+ c
2
2(a + b + c)

cyc
3(a b)
2
a + b

2
_

cyc
(a b)
2
(a + b)
_
a
2
+ b
2
+ c
2

cyc
S
a
(b c)
2
0
trong
S
a
=
3a
2
+ b
2
+ c
2
4bc
b + c
, S
b
=
a
2
+ 3b
2
+ c
2
4ca
c + a
, S
c
=
a
2
+ b
2
+ 3c
2
4ab
a + b
Khng mt tnh tng qut, ta c th gi s a b c, khi ta d dng kim tra c S
a
, S
b
0.
Ta c
S
b
+ S
c
=
a
2
+ 3b
2
+ c
2
4ca
c + a
+
a
2
+ b
2
+ 3c
2
4ab
a + b

a
2
+ 3b
2
+ c
2
4ca
a + b
+
a
2
+ b
2
+ 3c
2
4ab
a + b
=
2(a b c)
2
a + b
0
Do a b c nn (c a)
2
(a b)
2
0. Do

cyc
S
a
(b c)
2
S
b
(c a)
2
+ S
c
(a b)
2
(S
b
+ S
c
)(a b)
2
0
Vy bt ng thc cn chng minh ng. ng thc xy ra khi v ch khi (a, b, c) (1, 1, 1) hoc
(a, b, c) (2, 1, 1).

137 Chng minh rng vi mi s dng a, b, c > 0 tha abc = 1, ta c bt ng thc


1
(1 + a)
2
+
1
(1 + b)
2
+
1
(1 + c)
2
+
1
a + b + c + 1
1
Li gii. t x =
1a
1+a
, y =
1b
1+b
, z =
1c
1+c
, khi ta c x, y, z [1, 1] v
(1 x)(1 y)(1 z) = (1 + x)(1 + y)(1 + z)
Suy ra
x + y + z + xyz = 0
t q = ab + bc + ca v r = abc ta c |r| 1. Ta c
x
2
y
2
z
2
= (x + y + z)
2
= x
2
+ y
2
+ z
2
+ 2q
www.VNMATH.com
131
Do
2q = x
2
(y
2
z
2
1) y
2
z
2
0
Mt khc
2q = x
2
(y
2
z
2
1) y
2
z
2
(y
2
z
2
1) y
2
z
2
= y
2
(z
2
1) z
2
1 (z
2
1) z
2
1 = 2
Nh vy, ta c 0 q 1. Tr li bi ton ca ta, bt ng thc tng ng vi
(x + 1)
2
+ (y + 1)
2
+ (z + 1)
2
+
4
1 +
1x
1+x
+
1y
1+y
+
1z
1+z
4
Hay
r
2
2r 2q +
1 + q
1 r
1
f(r) = r
3
+ 3r
2
r + q(2r 1) 0
Xt 3 trng hp
Trng hp 1. Nu r
1
2
th
f(r) = r
3
+ 3r
2
r + q(2r 1) r
3
+ 3r
2
r (2r 1) = (1 r)
3
0
Trng hp 2. Nu r 0 th
f(r) = r
3
+ 3r
2
r + q(2r 1) r
3
+ 3r
2
r = r(r
2
3r + 1) 0
Trng hp 3. Nu
1
2
r 0, khi s dng bt ng thc AMGM, ta c
q
2
= x
2
y
2
+ y
2
z
2
+ z
2
x
2
+ 2xyz(x + y + z)
= x
2
y
2
+ y
2
z
2
+ z
2
x
2
2x
2
y
2
z
2
3
3
_
x
4
y
4
z
4
2x
2
y
2
z
2
x
2
y
2
z
2
= r
2
Suy ra |q| |r| hay q r. Do
f(r) = r
3
+ 3r
2
r + q(2r 1) r
3
+ 3r
2
r r(2r 1)
= r
2
(1 r) 0
Nh vy, trong mi trng hp, ta lun c f(r) 0. Bt ng thc c chng minh xong. ng
thc xy ra khi v ch khi a = b = c = 1 hoc a = b +, c 0
+
v cc hon v tng ng.

138 Cho cc s dng x, y, x tha x + y + z = 1. Chng minh rng


_
x
2
+ xyz +
_
y
2
+ xyz +
_
z
2
+ xyz
_
x
2
+ y
2
+ z
2
+ xy + yz + zx + 2
_
3xyz
Li gii. Ta c bt ng thc cn chng minh tng ng vi
_

cyc
_
x
2
+ xyz
_
2

cyc
x
2
+

cyc
xy + 2
_
3xyz
Hay
2

cyc
_
xy(x + yz)(y + zx)

cyc
xy 3xyz + 2
_
3xyz
www.VNMATH.com
132 CHNG 2. SOLUTION
S dng bt ng thc Cauchy Schwarz, ta c
2
_
xy(x + yz)(y + zx) = 2
_
xy(x(x + y + z) + yz)(y(x + y + z) + zx)
= 2(x + y)
_
xy(x + z)(y + z)
= 2(x + y)
_
x
2
y
2
+ xyz (x + y)
_
xy +
_
3xyz
_
= xy xyz + (x + y)
_
3xyz
Tng t, ta c
2
_
yz(y + zx)(z + xy) yz xyz + (y + z)
_
3xyz
2
_
zx(z + xy)(x + yz) zx xyz + (z + x)
_
3xyz
Do
2

cyc
_
xy(x + yz)(y + zx)

cyc
_
xy xyz + (x + y)
_
3xyz
_
=

cyc
xy 3xyz + 2
_
3xyz
Vy bt ng thc cn chng minh ng. ng thc xy ra khi v ch khi x = y = z =
1
3
.

139 Chng minh rng nu x, y, z l cc s khng m tha x


2
+ y
2
+ z
2
= 1 th
9
3

18

1
3
_
1
_
x+y
2
_
2
+
1
3
_
1
_
y+z
2
_
2
+
1
3
_
1
_
z+x
2
_
2
1 +
4
3

6
Li gii. Trc ht, ta s chng minh
1
3
_
1
_
x+y
2
_
2
+
1
3
_
1
_
y+z
2
_
2
+
1
3
_
1
_
z+x
2
_
2
1 +
4
3

6
Tht vy, khng mt tnh tng qut, gi s x y z 0, khi
1
3
_
1
_
y+z
2
_
2
1,
1
3
_
1
_
x+y
2
_
2
+
1
3
_
1
_
x+z
2
_
2

4
3
_
4
_
2
_
x+y
2
_
2

_
x+z
2
_
2
_
Nh th, ta ch cn chng minh rng
2
_
2
_
x + y
2
_
2

_
x + z
2
_
2
_
3
Hay
2 (x + y)
2
+ (x + z)
2
2xy + 2xz y
2
+ z
2
(ng)
Ta cn phi chng minh
9
3

18

1
3
_
1
_
x+y
2
_
2
+
1
3
_
1
_
y+z
2
_
2
+
1
3
_
1
_
z+x
2
_
2
www.VNMATH.com
133
S dng bt ng thc Holder, ta ch cn chng minh

cyc
1
1
_
x+y
2
_
2

9
2
Hay

cyc
_
x+y
2
_
2
1
_
x+y
2
_
2

3
2
S dng bt ng thc Cauchy Schwarz, ta c
_
x+y
2
_
2
1
_
x+y
2
_
2

2
_
x+y
2
_
2
2 (x
2
+ y
2
)
=
(x + y)
2
2((x
2
+ z
2
) + (y
2
+ z
2
))

1
2
_
x
2
x
2
+ z
2
+
y
2
y
2
+ z
2
_
Suy ra

cyc
_
x+y
2
_
2
1
_
x+y
2
_
2

1
2

cyc
_
x
2
x
2
+ z
2
+
y
2
y
2
+ z
2
_
=
3
2
Bt ng thc c chng minh xong. ng thc v tri xy ra khi v ch khi x = y = z =
1

3
,
ng thc v phi xy ra khi v ch khi (x, y, z) = (1, 0, 0).

140 Chng minh rng vi mi s khng m a, b, c tha a + b + c = 1,


a

4a + 5b
2
+
b

4b + 5c
2
+
c

4c + 5a
2

17
Li gii. S dng bt ng thc Cauchy Schwarz, ta c
_

cyc
a

4a + 5b
2
_
2

cyc
a
__

cyc
a
4a + 5b
2
_
=

cyc
a
4a + 5b
2
Ta cn chng minh

cyc
a
4a + 5b
2

9
17
Hay

cyc
b
2
4a(a + b + c) + 5b
2

3
17
Li s dng bt ng thc Cauchy Schwarz, ta c

cyc
b
2
4a(a + b + c) + 5b
2

(a
2
+ b
2
+ c
2
)
2

cyc
b
2
(4a(a + b + c) + 5b
2
)
Ta cn chng minh
17(a
2
+ b
2
+ c
2
)
2
3

cyc
b
2
(4a(a + b + c) + 5b
2
)
Hay
17(a
2
+ b
2
+ c
2
)
2
15

cyc
a
4
+ 12

cyc
a
2
b
2
+ 12

cyc
ab
3
+ 12

cyc
a
2
bc
www.VNMATH.com
134 CHNG 2. SOLUTION
Hay

cyc
a
4
+ 11

cyc
a
2
b
2
6

cyc
ab
3
+ 6

cyc
a
2
bc
1
2

cyc
(a
2
b
2
+ 2ab + 2bc 4ca)
2
0 (ng)
Bt ng thc c chng minh xong. ng thc xy ra khi v ch khi a = b = c =
1
3
.

141 Tm hng s k = k(n) ln nht sao cho bt ng thc sau ng vi mi s thc a


1
, a
2
, . . . , a
n
a
2
1
+ a
2
2
+ + a
2
n
k(n)(a
1
a
2
+ a
2
a
3
+ + a
n1
a
n
)
Li gii. Cho a
1
= 1, a
2
=
sin
2
n+1
sin

n+1
, . . . , a
n
=
sin
n
n+1
sin

n+1
, ta c k(n)
1
cos

n+1
. Ta s chng minh
1
cos

n+1
l gi tr cn tm, tc l
a
2
1
+ a
2
2
+ + a
2
n

1
cos

n+1
(a
1
a
2
+ a
2
a
3
+ + a
n1
a
n
)
Nhng iu ny l hin nhin v
2 cos

n + 1
n

k=1
a
2
k
2
n1

k=1
a
k
a
k+1
=
n1

k=1
sin
k
n+1
sin
(k+1)
n+1
_
sin
(k+1)
n+1
sin
k
n+1
a
k
a
k+1
_
2
0
Bt ng thc c chng minh xong. Vy
k(n) =
1
cos

n+1
.

142 Vi mi s dng a, b, c, ta c
3
_
a
2
+ bc
b + c
+
3
_
b
2
+ ca
c + a
+
3
_
c
2
+ ab
a + b

3
_
9(a + b + c)
Li gii. Trc ht, ta chng minh

cyc
3
_
2(a
2
+ bc)
b + c

cyc
3
_
(a + b)(a + c)
b + c
Hay

cyc
M
a
(a b)(a c) 0
trong
M
a
=
1
3

b + c
_
3
_
4(a
2
+ bc)
2
+
3
_
2(a
2
+ bc)(a + b)(a + c) +
3
_
(a + b)
2
(a + c)
2
_
M
b
=
1
3

c + a
_
3
_
4(b
2
+ ca)
2
+
3
_
2(b
2
+ ca)(b + c)(b + a) +
3
_
(b + c)
2
(b + a)
2
_
M
c
=
1
3

a + b
_
3
_
4(c
2
+ ab)
2
+
3
_
2(c
2
+ ab)(c + a)(c + b) +
3
_
(c + a)
2
(c + b)
2
_
www.VNMATH.com
135
Khng mt tnh tng qut, gi s a b c > 0, khi ta c
a(a
2
+ bc) b(b
2
+ ca), a(b + c) b(c + a), a c
a(b c)
b
Nh th, ta c aM
a
bM
b
, do

cyc
M
a
(a b)(a c) M
a
(a b)(a c) + M
b
(b c)(b a)

aM
a
(a b)(b c)
b
+ M
b
(b c)(b a)
=
(a b)(b c)(aM
a
bM
b
)
b
0
T y, chng minh bt ng thc cho, ta ch cn chng minh bt ng thc sau vi x, y, z
l di 3 cnh ca mt tam gic
xy
z
+
yz
x
+
zx
y

3
_
9(x
3
+ y
3
+ z
3
)
Hay

cyc
S
z
(x y)
2
0
vi
S
x
=
x
3
(y
2
+ yz + z
2
)
2
2y
3
z
3
+
3x
3
yz
x y z
S
y
=
y
3
(z
2
+ zx + x
2
)
2
2z
3
x
3
+
3y
3
zx
x y z
S
z
=
z
3
(x
2
+ xy + y
2
)
2
2x
3
y
3
+
3z
3
xy
x y z
Khng mt tnh tng qut, gi s x y z, khi ta d thy S
x
S
y
S
z
. Mt khc, t
t =
y+z
2

x
2
, ta c
S
y
+ S
z

3y
3
(z
2
+ zx + x
2
)
2z
2
x
2
+
3z
3
(x
2
+ xy + y
2
)
2x
2
y
2
+
3(y
4
+ z
4
)
xyz
2(x + 2t)
=
3(y
3
+ z
3
)
2x
2
+
9(y
4
+ z
4
)
2xyz
+
3(y
5
+ z
5
)
2y
2
z
2
2(x + 2t)

3t
3
x
2
+
9t
2
x
+ 3t 2(x + 2t) =
3t
3
+ 9t
2
x tx
2
2x
3
x
2

3
_
x
2
_
3
+ 9x
_
x
2
_
2
x
2
_
x
2
_
2x
3
x
2
=
x
8
> 0
Do S
x
S
y
0. T y ta c pcm. ng thc xy ra khi v ch khi a = b = c.

143 Cho cc s khng m a, b, c, chng minh bt ng thc


_
a +
b
2
c
_
2
+
_
b +
c
2
a
_
2
+
_
c +
a
2
b
_
2

12(a
3
+ b
3
+ c
3
)
a + b + c
www.VNMATH.com
136 CHNG 2. SOLUTION
Li gii. Bt ng thc tng ng

cyc
a
2
+ 2

cyc
ab
2
c
+

cyc
a
4
b
2

12(a
3
+ b
3
+ c
3
)
a + b + c
S dng kt qu bi ton 35, ta c
2

cyc
ab
2
c

15(a
2
b
2
+ b
2
c
2
+ c
2
a
2
)
ab + bc + ca
3

cyc
ab,

cyc
a
4
b
2

15(a
4
+ b
4
+ c
4
)
2(a
2
+ b
2
+ c
2
)

3
2

cyc
a
2
Mt khc, s dng bt ng thc Cauchy Schwarz, ta c

cyc
a
4
b
2

(a
3
+ b
3
+ c
3
)
2
a
2
b
2
+ b
2
c
2
+ c
2
a
2
Suy ra

cyc
a
4
b
2

15(a
4
+ b
4
+ c
4
)
4(a
2
+ b
2
+ c
2
)

3
4

cyc
a
2
+
(a
3
+ b
3
+ c
3
)
2
2(a
2
b
2
+ b
2
c
2
+ c
2
a
2
)
Nh th, ta ch cn chng minh
15(a
4
+ b
4
+ c
4
)
a
2
+ b
2
+ c
2
+
2(a
3
+ b
3
+ c
3
)
2
a
2
b
2
+ b
2
c
2
+ c
2
a
2
+
60(a
2
b
2
+ b
2
c
2
+ c
2
a
2
)
ab + bc + ca
+

cyc
a
2
12

cyc
ab
48(a
3
+ b
3
+ c
3
)
a + b + c
Khng mt tnh tng qut, gi s a +b +c = 1, t ab +bc +ca =
1q
2
3
, r = abc (1 q 0) th ta
c
(1q)
2
(1+2q)
27
r max
_
0,
(1+q)
2
(12q)
27
_
, bt ng thc tr thnh
(37 + 117q
2
34q
4
)r
2

2
27
(1 q
2
)(24q
6
304q
4
+ 65q
2
+ 44)r
+
(q
2
1)(175q
8
418q
6
+ 159q
4
+ 20q
2
17)
243
0
Ch rng (q
2
1)(175q
8
418q
6
+ 159q
4
+ 20q
2
17) 0 nn
+, Nu 24q
6
304q
4
+ 65q
2
+ 44 0 th bt ng thc l hin nhin.
+, Nu 1 2q, ta c
V T (37 + 117q
2
34q
4
)
_
(1 + q)
2
(1 2q)
27
_
2

2
27
(1 q
2
)(24q
6
304q
4
+ 65q
2
+ 44)
(1 q)
2
(1 + 2q)
27
+
(q
2
1)(175q
8
418q
6
+ 159q
4
+ 20q
2
17)
243
=
q
2
(q + 1)(6 + 22q + 22q
2
+ 38q
3
+ 228q
4
164q
5
+ 563q
6
+ 341q
7
96q
8
)
729
0
+, Nu 1 2q v 24q
6
304q
4
+ 65q
2
+ 44 0, suy ra
3
4
q
1
2
, ta c
V T
2
27
(1 q
2
)(24q
6
304q
4
+ 65q
2
+ 44)
(1 q)
2
(1 + 2q)
27
+
(q
2
1)(175q
8
418q
6
+ 159q
4
+ 20q
2
17)
243
=
(1 q
2
)(96q
9
381q
8
1216q
7
618q
6
+ 260q
5
+ 521q
4
+ 176q
3
+ 74q
2
37)
729
www.VNMATH.com
137
Ta c th d dng chng minh c 96q
9
381q
8
1216q
7
618q
6
+260q
5
+521q
4
+176q
3
+74q
2
37
0 vi mi q
_
1
2
,
3
4

. Vy bt ng thc c chng minh xong. ng thc xy ra khi v ch khi


a = b = c.

144 Cho cc s khng m a, b, c tha ab + bc + ca = 1, chng minh bt ng thc


1

a + bc
+
1

b + ca
+
1

c + ab
2

2
Li gii. Khng mt tnh tng qut, gi s a b c 0. Khi d thy a
1

3
. Xt 2 trng hp
Trng hp 1. a 2. t t =
_
(a + b)(a + c) a. Khi t 0 v
t
2
+ 2at = 1 (2.6)
Ch rng (a + t
2
) (a + bc) = t
2
bc = a(

a + b

a + c)
2
0 nn
1

a + bc

a + t
2
(2.7)
Ta s chng minh
1

b + ca
+
1

c + ab

2
_
t(a + 1)
(2.8)
Do
1

b + ca
+
1

c + ab

2
4
_
(b + ca)(c + ab)
theo bt ng thc AMGM nn ta ch cn chng minh t
2
(a +1)
2
(b +ca)(c +ab). Tht vy, ta
c
t
2
(a + 1)
2
(b + ca)(c + ab) = a(a + 1)
2
(

a + b

a + c)
2
a(b c)
2
=
a(b c)
2
[(a + 1)
2
(

a + b +

a + c)
2
]
(

a + b +

a + c)
2
0
V
_

a +
_
a +
1
a
_
2

a + b +

a + c
_
2
=
bc
a
0,
v
(a + 1)
2

a +
_
a +
1
a
_
2
= a
2
+ 1
1
a
2
_
a
2
+ 1
=
_
a
2
+ 1
_
_
a
2
+ 1 2
_

1
a

_
2
2
+ 1
_
_
2
2
+ 1 2
_

1
2
=
9 4

5
2
> 0
T (2.7) v (2.8), ta cn phi chng minh
2
_
t(a + 1)
+
1

a + t
2
2

2
S dng (2.6), ta c bt ng thc tng ng
2

1 + 2t t
2
+
_
t
2t
3
t
2
+ 1
2
www.VNMATH.com
138 CHNG 2. SOLUTION
Hay
_
2

1 + 2t t
2
+
_
t
2t
3
t
2
+ 1
_
2
4
4
_
(1 + 2t t
2
)(2t
3
t
2
+ 1)

t(8t
4
+ 20t
3
7t
2
6t + 7)
S dng gi thit a 2 v (2.6), ta c t
1
4
. Do
(1 + 2t t
2
)(2t
3
t
2
+ 1) = t
4
(5 2t) + 2t(1 t) + 1 1
8t
4
+ 20t
3
7t
2
6t = 8t
4
t
2
(7 20t) 6t 0
Nh th ta ch cn chng minh
4 7

t
nhng y l iu hin nhin do t
1
4
.
Trng hp 2.
1

3
a 2. S dng bt ng thc AMGM,
1

a + bc
+
1

b + ca
+
1

c + ab

3
6
_
(a + bc)(b + ca)(c + ab)
Ta cn chng minh
(a + bc)(b + ca)(c + ab)
729
512
.
t S = a + b + c v P = abc. Do a 2 v ab + bc + ca = 1 ta suy ra c

3 S
5
2
. t
u =
1
3
(S

S
2
3) v v =
1
3
(S + 2

S
2
3). Ta d dng kim tra c
2u + v = S, u
2
+ 2uv = 1,
5

13
6
u
1

3
v
5 + 2

13
6
, 0 P u
2
v
v
(a + bc)(b + ca)(c + ab) = P
2
+ (S
2
2S 1)P + 1
Do f(P) = P
2
+ (S
2
2S 1)P + 1 l hm li nn
f(P) max{f(0), f(u
2
v)} = max{1, f(u
2
v)}
Ta ch cn chng minh
729
512
f(u
2
v) = v
2
(u + 1)
2
(v + u
2
)
Hay
729
64

(1 + 2u u
2
)
2
(2u
3
u
2
+ 1)
u
u
3
(2u
4
9u
3
+ 7u
2
1) + (7u
4
5u
3
+ u
2
7u + 1)
25
64
u 0
Ta s chng minh rng vi mi u
_
1
5
,
1

3
_
,
g(u) = 2u
4
9u
3
+ 7u
2
1 0
h(u) = 7u
4
5u
3
+ u
2
7u + 1 0
Tht vy,
Nu u
1
3
, ta c
g(u) 2u
4
+
26
5
u
2
1
161
405
< 0
h(u) = (u
2
+ 1)(1 5u) + u(7u
3
2) < 0
www.VNMATH.com
139
Nu u
1
3
, ta c
g

(u) = u(8u
2
27u + 14) > 0
h

(u) = 28u
3
15u
2
+ 2u 7 < 13u
2
+ 2u 7 2u
8
3
< 0
Suy ra g(u) l hm ng bin v h(u) l hm nghch bin. Do
g(u) g
_
1

3
_
=
14 9

3
9
< 0, h(u) h
_
1
3
_
=
107
81
< 0
Bt ng thc c chng minh xong.

145 Cho cc s dng a, b, c tha a + b + c =


1
a
+
1
b
+
1
c
, chng minh
_
a + b
b + 1
+
_
b + c
c + 1
+
_
c + a
a + 1
3
Li gii. S dng bt ng thc AMGM, ta ch cn chng minh
(a + b)(b + c)(c + a) (a + 1)(b + 1)(c + 1)
T a + b + c =
1
a
+
1
b
+
1
c
, ta c ab + bc + ca = abc(a + b + c). Do bt ng thc tng ng
(a + b + c)(ab + bc + ca) abc 1 + (a + b + c) + (ab + bc + ca) + abc
Hay
abc(a + b + c)
2
abc 1 + (a + b + c) + abc(a + b + c) + abc
abc((a + b + c)
2
1) (1 + abc)(a + b + c + 1)
(a + b + c)
2
1
_
1 +
1
abc
_
(a + b + c + 1)
a + b + c 1 1 +
1
abc
a + b + c 2 +
1
abc
1
a
+
1
b
+
1
c

1
abc
2
(ab + bc + ca)
2
a
2
b
2
c
2
(a + b + c)

1
abc
2
_
ab + bc + ca
abc(a + b + c)
_
3/2
(ab + bc + ca)
2
abc(a + b + c)
1 2
_
(ab + bc + ca)
3
abc(a + b + c)
3
_
1/2
Nhng
(ab + bc + ca)
3
abc(a + b + c)
3

(ab + bc + ca)
2
3abc(a + b + c)
= x
2
(x 1)
Suy ra ta ch cn chng minh
3x
2
1 2x
Hay
(x 1)(3x + 1) 0 (ng)
Bt ng thc c chng minh. ng thc xy ra khi v ch khi a = b = c = 1.

www.VNMATH.com
140 CHNG 2. SOLUTION
146 Cho a
1
, a
2
, . . . , a
5
l cc s dng tha
a
1
a
2
a
5
= a
1
(1 + a
2
) + a
2
(1 + a
3
) + + a
5
(1 + a
1
) + 2
Tm gi tr nh nht ca biu thc
P =
1
a
1
+
1
a
2
+ +
1
a
5
.
Li gii. Trc ht, ta s chng minh rng vi mi x, y, z, t, u 0,
(x + y + z + t + u)
3
25(xyz + yzt + ztu + tux + uxy)
Tht vy, khng mt tnh tng qut, gi s x = min{x, y, z, t, u}. t y = x + b, z = x + c, t =
x + d, u = x + e th b, c, d, e 0, ta c
V T V P = 5Ax + (b + c + d + e)
3
25cd(b + e)
vi
A = 3(b + c + d + e)
2
5be 10bc 5bd 10cd 5ce 10de
=
1
12
(6b + d + e 4c)
2
+
5
84
(7d 4c 5e)
2
+
5
28
(2c e)
2
+
5
4
e
2
0
S dng bt ng thc AMGM, ta c
25cd(b + e) 27cd(b + e) (c + d + (b + e))
3
= (b + c + d + e)
3
Bt ng thc c chng minh. S dng bt ng thc ny vi x =
1
a
1
, y =
1
a
2
, z =
1
a
3
, t =
1
a
4
, u =
1
a
5
, ta c
P
3

25(a
1
a
2
+ a
2
a
3
+ + a
5
a
1
)
a
1
a
2
a
5
Mt khc, s dng bt ng thc Maclaurin v bt ng thc AMGM, ta c
125(a
1
+ a
2
+ + a
5
)
a
1
a
2
a
5
P
4
,
3125
a
1
a
2
a
5
P
5
Suy ra
1 =
a
1
+ a
2
+ + a
5
a
1
a
2
a
5
+
a
1
a
2
+ a
2
a
3
+ + a
5
a
1
a
1
a
2
a
5
+
2
a
1
a
2
a
5

P
4
125
+
P
3
25
+
2P
5
3125
Hay
(2P 5)(P
4
+ 15P
3
+ 100P
2
+ 250P + 625)
3125
0
Do ta c P
5
2
. Mt khc, cho a
1
= a
2
= = a
5
= 2, ta c P =
5
2
, vy
min P =
5
2
.

147 Vi mi s dng a, b, c, ta c
a(a + c)
b(b + c)
+
b(b + a)
c(c + a)
+
c(c + b)
a(a + b)

3(a
2
+ b
2
+ c
2
)
ab + bc + ca
www.VNMATH.com
141
Li gii. S dng bt ng thc Cauchy Schwarz, ta c
_

cyc
ab(a + c)(b + c)
__

cyc
a(a + c)
b(b + c)
_
(a
2
+ b
2
+ c
2
+ ab + bc + ca)
2
.
S dng bt ng thc AMGM,
(a
2
+ b
2
+ c
2
+ ab + bc + ca)
2
4(a
2
+ b
2
+ c
2
)(ab + bc + ca),
Nh th ta ch cn chng minh
4(a
2
+ b
2
+ c
2
)(ab + bc + ca)
2
3(a
2
+ b
2
+ c
2
)

cyc
ab(a + c)(b + c)
Hay
(ab + bc + ca)
2
3abc(a + b + c)
Bt ng thc ny hin nhin ng theo bt ng thc AMGM, vy ta c pcm. ng thc xy
ra khi v ch khi a = b = c.

148 Chng minh rng vi mi a, b, c dng,


a(b + c)

a
2
+ bc
+
b(c + a)

b
2
+ ca
+
c(a + b)

c
2
+ ab

_
6(a
2
+ b
2
+ c
2
)
Li gii. S dng bt ng thc Cauchy Schwarz,
_

cyc
a(b + c)

a
2
+ bc
_
2

cyc
a
2
(b + c)
2
__

cyc
1
a
2
+ bc
_
Mt khc, cng theo bt ng thc Cauchy Schwarz, ta c (a
2
+ bc)(b + c) b(a + c)
2
v (a
2
+
bc)(c + b) c(a + b)
2
, suy ra
1
(a + b)
2
+
1
(a + c)
2

c
(a
2
+ bc)(c + b)
+
b
(a
2
+ bc)(b + c)
=
1
a
2
+ bc
Nh th, ta c
1
a
2
+ bc
+
1
b
2
+ ca
+
1
c
2
+ ab
2
_
1
(a + b)
2
+
1
(b + c)
2
+
1
(c + a)
2
_
Do
_

cyc
a(b + c)

a
2
+ bc
_
2
2
_

cyc
a
2
(b + c)
2
__

cyc
1
(b + c)
2
_
Khng mt tnh tng qut, gi s a b c. Khi , ta c
a
2
(b + c)
2
b
2
(c + a)
2
c
2
(a + b) v
1
(b + c)
2

1
(c + a)
2

1
(a + b)
2
Do , theo bt ng thc Chebyshev,
_

cyc
a
2
(b + c)
2
__

cyc
1
(b + c)
2
_
3

cyc
a
2
(b + c)
2
(b + c)
2
= 3(a
2
+ b
2
+ c
2
)
Vy
_

cyc
a(b + c)

a
2
+ bc
_
2
6(a
2
+ b
2
+ c
2
)
Bt ng thc c chng minh. ng thc xy ra khi v ch khi a = b = c.

www.VNMATH.com
142 CHNG 2. SOLUTION
149 Cho a, b, c l cc s dng, chng minh rng
3 +
a
b
+
b
c
+
c
a
2

(a + b + c)
_
1
a
+
1
b
+
1
c
_
Li gii. t x
3
= a, y
3
= b v z
3
= c. Theo bt ng thc Schur,
3 +
a
b
+
b
c
+
c
a
= 3
x
y

y
z

z
x
+
x
3
y
3
+
y
3
z
3
+
z
3
x
3

cyc
x
2
y
2
_
y
z
+
z
x
_
=

cyc
x
2
yz
+

cyc
xz
y
2
Mt khc, s dng bt ng thc AMGM th

cyc
x
2
yz
+

cyc
xz
y
2
=
x
3
+ y
3
+ z
3
xyz
+
(xz)
3
+ (yx)
3
+ (zy)
3
(xyz)
2
2

(x
3
+ y
3
+ z
3
)(x
3
z
3
+ y
3
x
3
+ z
3
y
3
)
x
3
y
3
z
3
= 2

(x
3
+ y
3
+ z
3
)
_
1
x
3
+
1
y
3
+
1
z
3
_
= 2

(a + b + c)
_
1
a
+
1
b
+
1
c
_
Bt ng thc c chng minh xong. ng thc xy ra khi v ch khi a = b = c.
150 Cho a, b, c l cc s khng m tha mn ab + bc + ca = 1, chng minh
a
2
b
+
b
2
c
+
c
2
a
2(a
2
+ b
2
+ c
2
)

3 2
Li gii. Bt ng thc tng ng

cyc
_
a
2
b
+ b 2a
_
+
_
_

cyc
a

cyc
ab
_
_
2
_

cyc
a
2

cyc
ab
_
Hay

cyc
S
c
(a b)
2
0
trong
S
a
=
1
c
+ t 1, S
b
=
1
a
+ t 1, S
c
=
1
b
+ t 1
vi t =
1
2(a+b+c+

3)
.
Ta c
S
a
+ S
b
+ S
c
=
1
a
+
1
b
+
1
c
3 +
3
2
_
a + b + c +

3
_
=
1
abc
3 +
3
2
_
a + b + c +

3
_

3
(ab + bc + ca)
3
2
3 +
3
2
_
a + b + c +

3
_
= 3

3 3 +
3
2
_
a + b + c +

3
_ > 0
www.VNMATH.com
143
S
a
S
b
+ S
b
S
c
+ S
c
S
a
=

cyc
_
t +
1
b
1
__
t +
1
c
1
_
= 3t
2
+ 2
_

cyc
1
a
3
_
t +

cyc
1
ab
2

cyc
1
a
+ 3
>

cyc
1
ab
2

cyc
1
a
+ 3 =
a + b + c + 3abc 2
abc
0
Tht vy, nu a +b +c 2 th iu ny l hin nhin, nu a +b +c 2, t p = a +b +c th theo
bt ng thc Schur, ta c abc
p(4p
2
)
9
0, do
a + b + c + 3abc 2 p +
4p p
3
3
2 =
(2 p)(p 1)(p + 3)
3
0
Bt ng thc c chng minh xong. ng thc xy ra khi v ch khi a = b = c =

3
3
.

151 Tm hng s k ln nht sao cho bt ng thc sau ng


a + b + c + kabc k + 3
vi mi s khng m a, b, c tha mn ab + bc + ca + 6abc = 9.
Li gii. Cho a = b = 3, c = 0, ta c k 3. Ta s chng minh y l gi tr ta cn tm, tc l
a + b + c + 3abc 6
t p = a + b + c, q = ab + bc + ca, r = abc. Gi thit ca bi ton c th vit li l q + 6r = 9. S
dng bt ng thc AM - GM, ta c p
2
3q 9. Bt ng thc tr thnh
p + 3r 6
Hay
2p q 3
Nu p 6, th iu ny hin nhin ng. Xt 6 p 3, khi c 2 trng hp xy ra
Trng hp 1. Nu p
2
4q th
2p q 2p
p
2
4
=
(p 2)(6 p)
4
+ 3 3
Trng hp 2. Nu p
2
4q th theo bt ng thc Schur, ta c r
p(4qp
2
)
9
0. Do
27 = 3q + 18r 3q + 2p(4q p
2
)
V v th
2p q 2p
2p
3
+ 27
8p + 3
Ta cn chng minh
2p
2p
3
+ 27
8p + 3
3
Hay
(p + 1)(p 3)(p 6) 0
Bt ng thc ny hin nhin ng. Vy
k
min
= 3.

www.VNMATH.com
144 CHNG 2. SOLUTION
152 Cho cc s khng m a, b, c tha a
2
+ b
2
+ c
2
= 1. Chng minh rng
a
3
b
2
bc + c
2
+
b
3
c
2
ca + a
2
+
c
3
a
2
ab + b
2

2
Li gii. Bt ng thc cn chng minh tng ng

cyc
a
3
(b + c)
b
3
+ c
3

2
Hay

cyc
_
a
3
(b + c)
b
3
+ c
3
+ b + c
_
2(a + b + c) +

2
Hay
(a
3
+ b
3
+ c
3
)

cyc
1
a
2
ab + b
2
2(a + b + c) +

2
S dng bt ng thc Cauchy Schwarz, ta c

cyc
1
a
2
ab + b
2

9
2(a
2
+ b
2
+ c
2
) (ab + bc + ca)
Do , ta cn chng minh
9(a
3
+ b
3
+ c
3
)
2(a
2
+ b
2
+ c
2
) (ab + bc + ca)
2(a + b + c) +

2
t p = a + b + c, q = ab + bc + ca, r = abc th ta c p, q, r 0 v p =

1 + 2q, 1 q. Khi , bt
ng thc trn c th vit li l
9(p(1 q) + 3r)
_
2p +

2
_
(2 q)
Hay
5p 7pq +

2q + 27r 2

2
Xt 2 trng hp
Trng hp 1. 1 2, bt ng thc tng ng vi
f(q) = 5
_
2q + 1 7q
_
2q + 1 +

2q + 27r 2

2
Ta c
f

(q) =
_
2(2q + 1) 21q 2

2q + 1

_
2(1 + 1) 21q 2

2q + 1
=
21q

2q + 1
< 0
Suy ra, f(q) l hm nghch bin. Suy ra,
f(q) f
_
1
2
_
= 2

2 + 27r 2

2
Trng hp 2. 2q 1, s dng bt ng thc Schur, ta c r
p(4qp
2
)
9
=
p(2q1)
9
0. Do , ta
ch cn chng minh
5p 7pq +

2q + 3p(2q 1) 2

2
Hay
g(q) = 2
_
2q + 1 q
_
2q + 1 +

2q 2

2
www.VNMATH.com
145
Ta c
g

(q) =
_
2(2q + 1) 3q + 1

2q + 1

_
2(1 + 1) 3q + 1

2q + 1
=
3(1 q)

2q + 1
0
Do , g(q) l hm ng bin. Suy ra,
g(q) g
_
1
2
_
= 2

2
Do , bt ng thc cn chng minh ng. ng thc xy ra khi v ch khi (a, b, c) =
_
1

2
,
1

2
, 0
_
.

153 Cho cc s khng m x, y, z tha 6 x + y + z 3, chng minh rng

1 + x +
_
1 + y +

1 + z
_
xy + yz + zx + 15
Li gii. t a
2
= 1 + x, b
2
= 1 + y, c
2
= 1 + z, d = a
2
+ b
2
+ c
2
th ta c a, b, c 1 v 9 d 6, bt
ng thc tr thnh
a + b + c
_
18 2d + a
2
b
2
+ b
2
c
2
+ c
2
a
2
Hay
3d + 2(ab + bc + ca) 18 + a
2
b
2
+ b
2
c
2
+ c
2
a
2
S dng gi thit 9 d 6 v bt ng thc AM - GM, ta c
3d(d 6)
1
3
d
2
(d 6) (d 6)(a
2
b
2
+ b
2
c
2
+ c
2
a
2
)
Suy ra
3d +
6(a
2
b
2
+ b
2
c
2
+ c
2
a
2
)
d
18 + a
2
b
2
+ b
2
c
2
+ c
2
a
2
Ta cn chng minh
ab + bc + ca
3(a
2
b
2
+ b
2
c
2
+ c
2
a
2
)
d
Hay
(ab + bc + ca)(a
2
+ b
2
+ c
2
) 3(a
2
b
2
+ b
2
c
2
+ c
2
a
2
)
Hay

cyc
(b + c)(4a b c)(a b)(a c) 0
Khng mt tnh tng qut, gi s a b c, ta c

cyc
(b +c)(4ab c)(ab)(ac) = (ab)
2
(5ab +3bc +3ca7c
2
) +(a+b)(4c ab)(c a)(c b)
Ch rng
9c
2
9 a
2
+ b
2
+ c
2
Suy ra
8c
2
a
2
+ b
2

(a + b)
2
2
Do
4c a b 0
T y, ta c pcm. ng thc xy ra khi v ch khi x = y = z = 2 hoc x = y = z = 1.

www.VNMATH.com
146 CHNG 2. SOLUTION
154 Cho cc s dng x, y, z tha xyz = 1, chng minh bt ng thc
y + z
x
3
+ yz
+
z + x
y
3
+ zx
+
x + y
z
3
+ xy

1
x
2
+
1
y
2
+
1
z
2
Li gii. S dng bt ng thc GM - HM, ta c
1 =
3

xyz
3
1
x
+
1
y
+
1
z
t a =
1
x
, b =
1
y
, c =
1
z
th ta c a, b, c > 0 v 1
3
a+b+c
, do

cyc
y + z
x
3
+ yz

_

cyc
a
__

cyc
a
3
(b + c)
3a
3
+ bc(a + b + c)
_
Ta cn chng minh

cyc
a
2

cyc
a
__

cyc
a
3
(b + c)
3a
3
+ bc(a + b + c)
_
Hay
3

cyc
a
2

cyc
a

cyc
3a
3
(b + c)
3a
3
+ bc(a + b + c)
Hay

cyc
(a b)
2

cyc
a
+

cyc
a(3a
3
3a
2
(b + c) + 3abc + bc(b + c 2a))
3a
3
+ bc(a + b + c)
0

cyc
(a b)
2

cyc
a
+ 3

cyc
a
2
(a b)(a c)
3a
3
+ bc(a + b + c)
+ abc

cyc
b + c 2a
3a
3
+ bc(a + b + c)
0
Khng mt tnh tng qut, gi s a b c, d thy
a
3
3a
3
+bc(a+b+c)

b
3
3b
3
+ac(a+b+c)
> 0 nn theo
nh l 2, ta c

cyc
a
2
(a b)(a c)
3a
3
+ bc(a + b + c)
0
Ta cn phi chng minh

cyc
b + c 2a
3a
3
+ bc(a + b + c)
0
Hay

cyc
S
c
(a b)
2
0
trong
S
a
= (3b
2
+ 3c
2
a
2
+ 3bc ca ab)(3a
3
+ bc(a + b + c))
S
b
= (3c
2
+ 3a
2
b
2
+ 3ca ab bc)(3b
3
+ ca(a + b + c))
S
c
= (3a
2
+ 3b
2
c
2
+ 3ab bc ca)(3c
3
+ ab(a + b + c))
Do a b c > 0 nn d thy S
b
, S
c
0. Ta c
a
2
S
b
+ b
2
S
a
= c(a + b + c)((a b)
2
(a + b)(2a
2
+ ab + 2b
2
) + c(a b)(a
3
b
3
)
+ a
5
+ b
5
+ 3(a
3
+ b
3
)c
2
+ 2(a
4
+ b
4
)c) + 3a
2
b
2
(2(a b)
2
(a + b)
+ 3(a + b)c
2
+ 2(a
2
+ b
2
)c + (a b)
2
c) 0
Bt ng thc c chng minh xong. ng thc xy ra khi v ch khi x = y = z = 1.

www.VNMATH.com
147
155 Cho cc s dng a, b, c, chng minh bt ng thc
3
9

9a(a + b)
2(a + b + c)
2
+
3

6bc
(a + b)(a + b + c)
4
Li gii. S dng bt ng thc Holder, ta c
_
3
9

9a(a + b)
2(a + b + c)
2
+
3

6bc
(a + b)(a + b + c)
_9
=
_
9

9a(a + b)
2(a + b + c)
2
+
9

9a(a + b)
2(a + b + c)
2
+
9

9a(a + b)
2(a + b + c)
2
+
9

6
3
b
3
c
3
(a + b)
3
(a + b + c)
3
_9

_
a
a + b
+
3(a + b)
2(a + b + c)
+
3(a + b)
2(a + b + c)
+
b
a + b
+
3c
a + b + c
_

_
3(a + b)
2(a + b + c)
+
3(a + b)
2(a + b + c)
+
a
a + b
+
b
a + b
+
3c
a + b + c
_

_
3(a + b)
2(a + b + c)
+
a
a + b
+
3(a + b)
2(a + b + c)
+
b
a + b
+
3c
a + b + c
_
= 4
9
T y ta c pcm. ng thc xy ra khi v ch khi a = b = c.
Nhn xt. Tng qut ha, ta c bi ton IMO Shortlist 2004
Nu a
1
, a
2
, . . . , a
n
l cc s thc dng. Gi g
n
l trung bnh nhn ca chng v A
1
, A
2
, . . . , A
n
l dy
trung bnh cng
A
k
=
a
1
+ a
2
+ + a
k
k
k = 1, n
Gi G
n
l trung bnh nhn ca A
1
, A
2
, . . . , A
n
. Chng minh bt ng thc
n
n
_
G
n
A
n
+
g
n
G
n
n + 1
Li gii nh sau
Ch rng vi mi k = 1, 2, . . . , n, ta c
a
k
A
k
=
kA
k
(k 1)A
k1
A
k
= k (k 1)
A
k1
A
k
y ta t A
0
= 0. t
x
1
= 1, x
k
=
A
k1
A
k
, k = 2, n
Ta c
n
_
G
n
A
n
=
n
2
_
A
1
A
2
. . . A
n
A
n
n
=
n
2
_
x
2
x
2
3
x
n1
n
,
g
n
G
n
=
n

_
n

k=1
a
k
A
k
=
n

_
n

k=1
(k (k 1)x
k
)
Do
n
n
_
G
n
A
n
+
g
n
G
n
= n
n
2
_
x
2
x
2
3
x
n1
n
+
n

_
n

k=1
(k (k 1)x
k
)
Ta s chng minh
n
n
2
_
x
2
x
2
3
x
n1
n
+
n

_
n

k=1
(k (k 1)x
k
) n + 1
bng 2 cch
www.VNMATH.com
148 CHNG 2. SOLUTION
Cch 1. S dng bt ng thc AMGM, ta c
n
n
2
_
x
2
x
2
3
x
n1
n
= n
n
2
_
x
n(n+1)/2
1
x
2
x
2
3
x
n1
n

1
n
_
n(n + 1)
2
x
1
+
n

k=2
(k 1)x
k
_
=
n + 1
2
+
1
n
n

k=1
(k 1)x
k
n

_
n

k=1
(k (k 1)x
k
)
1
n
n

k=1
(k (k 1)x
k
) =
n + 1
2

1
n
n

k=1
(k 1)x
k
Cng tng ng v vi v 2 bt ng thc trn, ta c pcm.
Cch 2. S dng bt ng thc Holder
n+1

j=1
n

1j

2j

nj

n

_
_
n+1

j=1

1j
__
n+1

j=1

2j
_

_
n+1

j=1

nj
_
vi

1j
= 1 j = 1, n + 1

kj
=
n
_
x
k1
k
, k = 2, n, j = 1, n

k,n+1
= k (k 1)x
k
, k = 2, n
Khi , ta c
n+1

j=1
n

1j

2j

nj
= n
n
2
_
x
2
x
2
3
x
n1
n
+
n

_
n

k=1
(k (k 1)x
k
)
Mt khc, ta cng c
n+1

j=1

1j
= n + 1,
n+1

j=1

kj
= n
n
_
x
k1
k
+ k (k 1)x
k
k = 2, n
Li c vi mi k = 2, n th
n
n
_
x
k1
k
+ k (k 1)x
k
= n
n
_
x
nk+1
1
x
k1
k
+ k (k 1)x
k
(n k + 1)x
1
+ (k 1)x
k
+ k (k 1)x
k
= n + 1
Vy ta c pcm.

156 Cho cc s khng m a, b, c, chng minh bt ng thc


1
(a + 2b)
2
+
1
(b + 2c)
2
+
1
(c + 2a)
2

1
ab + bc + ca
Li gii. Ta c 2 b sau
B 1. Vi mi a, b, c khng m, ta c
a
3
+ b
3
+ c
3
3abc 4(a b)(b c)(c a)
Tht vy, khng mt tnhtng qut, gi s c = min{a, b, c}, t a = c + x, b = c + y, ta c
V T V P = 3(x
2
xy + y
2
)c + x
3
+ y(2x y)
2
0
B 2. Vi mi a, b, c khng m th
ab
2
+ bc
2
+ ca
2

4
27
(a + b + c)
3
abc
www.VNMATH.com
149
Tht vy, khng mt tnh tng qut, gi s c = min{a, b, c}, t a = c + x, b = c + y, ta c
V T V P =
9(x
2
xy + y
2
)c + (2x y)
2
(x + 4y)
27
0
Tr li bi ton ca ta. Bt ng thc tng ng vi
4
_

cyc
a
4
__

cyc
ab
_
+ 5

cyc
a
3
b
3
+ 4

cyc
a
4
bc + 5abc

cyc
ab(a + b) 21a
2
b
2
c
2
12(a
2
b
2
)(b
2
c
2
)(c
2
a
2
) + 24abc

cyc
ab
2
Theo trn, ta c
4(a b)(b c)(c a)

cyc
a
3
3abc,

cyc
ab
2

4(a + b + c)
3
27
abc
Nn ta ch cn chng minh rng
4
_

cyc
a
4
__

cyc
ab
_
+ 5

cyc
a
3
b
3
+ 4

cyc
a
4
bc + 5abc

cyc
ab(a + b) 21a
2
b
2
c
2
3
_

cyc
a
3
3abc
_
(a + b)(b + c)(c + a) + 24abc
_
4(a + b + c)
3
27
abc
_
Hay
9

cyc
ab(a
4
+b
4
) +45

cyc
a
3
b
3
27

cyc
a
2
b
2
(a
2
+b
2
) 14abc

cyc
a
3
+3abc

cyc
ab(a+b) 3a
2
b
2
c
2
0
9
_

cyc
ab(a
4
+ b
4
)

cyc
a
2
b
2
(a
2
+ b
2
)
_
18
_

cyc
a
2
b
2
(a
2
+ b
2
) 2

cyc
a
3
b
3
_
+ 9
_

cyc
a
3
b
3
3a
2
b
2
c
2
_
14abc
_

cyc
a
3
3abc
_
+ 3abc
_

cyc
ab(a + b) 6abc
_
0

cyc
(a b)
2
(18ab(a
2
ab + b
2
) + 9c
2
(ab + bc + ca) 14abc(a + b + c) + 6abc
2
) 0
Ta s chng minh
18ab(a
2
ab + b
2
) + 9c
2
(ab + bc + ca) 14abc(a + b + c) + 6abc
2
0
Tht vy, t t =

ab, ta c
18ab(a
2
ab + b
2
) + 9c
2
(ab + bc + ca) 14abc(a + b + c) + 6abc
2
=
_

b
_
2
_
9c
3
14abc + 18ab
_

a +

b
_
2
_
+ t(18t
3
28t
2
c + tc
2
+ 18c
3
)

b
_
2
(9c
3
14t
2
c + 72t
3
) 0
Bt ng thc c chng minh xong. ng thc xy ra khi v ch khi a = b = c.

www.VNMATH.com
150 CHNG 2. SOLUTION
157 Cho cc s khng m a, b, c, chng minh bt ng thc
a
2
a
2
+ ab + b
2
+
b
2
b
2
+ bc + c
2
+
c
2
c
2
+ ca + a
2
+
ab + bc + ca
a
2
+ b
2
+ c
2
2
Li gii. Ta c
2

cyc
a
2
a
2
+ ab + b
2

ab + bc + ca
a
2
+ b
2
+ c
2
=
(a + b + c)(ab + bc + ca)(ab
5
+ bc
5
+ ca
5
ab
2
c
3
bc
2
a
3
ca
2
b
3
)
(a
2
+ b
2
+ c
2
)(a
2
+ ab + b
2
)(b
2
+ bc + c
2
)(c
2
+ ca + a
2
)
0
Nn bt ng thc cn chng minh ng. ng thc xy ra khi v ch khi a = b = c hoc
b
a
0,
c
b
0 v cc hon v tng ng.
Nhn xt. C mt ng thc kh p v c bit l
2

cyc
a
2
a
2
+ ab + b
2
=
(a + b + c)(ab + bc + ca)(ab
2
+ bc
2
+ ca
2
)
(a
2
+ ab + b
2
)(b
2
+ bc + c
2
)(c
2
+ ca + a
2
)

158 Cho cc s khng m x, y, z tha x + y + z = 3, chng minh bt ng thc


x
2
y + y
2
z +
3
2
xyz 4
Li gii. Nu x 2y, ta s chng minh
x
2
y + y
2
z +
3
2
xyz (x + z)
2
y
Tht vy, ta c
(x + z)
2
y x
2
y y
2
z
3
2
xyz =
yz(x 2y + 2z)
2
0
Mt khc, s dng bt ng thc AM-GM th
y(x + z)
2
4
_
y + (x + z)
3
_
3
= 4
Suy ra
x
2
y + y
2
z +
3
2
xyz (x + z)
2
y 4
Ta cn phi xt trng hp 2y x, khi bt ng thc tng ng vi
f(z) = 4(x + y + z)
3
27x
2
y 27y
2
z
81
2
xyz 0
Ta c
f

(z) = 3
_
4(x + y + z)
2

9
2
y(2y + 3x)
_
f

(z) = 0 z =
3
2

2
_
y(2y + 3x) x y
Do 2y x nn
3
2

2
_
y(2y + 3x) x y 0, t y d thy
f(z) f
_
3
2

2
_
y(2y + 3x) x y
_
=
27
2
y
_
x
2
+ 5xy + 2y
2

y(3x + 2y)
3/2

2
_
=
27xy(x 2y)
2
(2x + y)
4
_
x
2
+ 5xy + 2y
2
+

y(3x+2y)
3/2

2
_ 0
www.VNMATH.com
151
Bt ng thc c chng minh xong. ng thc xy ra khi v ch khi x = 2, y = 1, z = 0 hoc
x = 0, y = 2, z = 1.

159 Cho cc s khng m a, b, c, chng minh bt ng thc


1
a
2
+ bc
+
1
b
2
+ ca
+
1
c
2
+ ab

3(a + b + c)
2
2(a
2
+ b
2
+ c
2
)(ab + bc + ca)
Li gii. Bt ng thc tng ng vi

cyc
a
2
+ b
2
+ c
2
a
2
+ bc

3(a + b + c)
2
2(ab + bc + ca)
Hay
3 +

cyc
b
2
+ c
2
bc
a
2
+ bc

3(a + b + c)
2
2(ab + bc + ca)
S dng bt ng thc Cauchy Schwarz, ta c

cyc
b
2
+ c
2
bc
a
2
+ bc

(2(a
2
+ b
2
+ c
2
) ab bc ca)
2

cyc
(a
2
+ bc)(b
2
+ c
2
bc)
=
(2(a
2
+ b
2
+ c
2
) ab bc ca)
2
(ab + bc + ca)(a
2
+ b
2
+ c
2
+ ab + bc + ca) 4abc(a + b + c)
Nh th, ta ch cn chng minh
3 +
(2(a
2
+ b
2
+ c
2
) ab bc ca)
2
(ab + bc + ca)(a
2
+ b
2
+ c
2
+ ab + bc + ca) 4abc(a + b + c)

3(a + b + c)
2
2(ab + bc + ca)
Gi s a + b + c = 1, t q = ab + bc + ca, r = abc th ta c
1
3
q 0. Theo bt ng thc Schur
th r max
_
0,
4q1
9
_
. Bt ng thc tr thnh
3 +
(2 5q)
2
q q
2
4r

3
2q
Nu 4q 1, ta c
3 +
(2 5q)
2
q q
2
4r

3
2q
3 +
(2 5q)
2
q q
2

3
2q
=
(5 11q)(1 4q)
2q(1 q)
0
Nu 4q 1, ta c
3 +
(2 5q)
2
q q
2
4r

3
2q
3 +
(2 5q)
2
q q
2

4(4q1)
9

3
2q
=
3(11 4q)(1 3q)(4q 1)
2q(4 7q 9q
2
)
0
Bt ng thc c chng minh xong. ng thc xy ra khi v ch khi a = b = c hoc a = b, c = 0
v cc hon v tng ng.

160 Cho cc s khng m a, b, c, chng minh bt ng thc


4
3
(ab
2
+ bc
2
+ ca
2
) + a
2
+ b
2
+ c
2
+ 2 3(ab + bc + ca)
www.VNMATH.com
152 CHNG 2. SOLUTION
Li gii. S dng bt ng thc Cauchy Schwarz, ta c

cyc
a
b
=

cyc
1
b
1
a

_
1
a
+
1
b
+
1
c
_
2
1
ab
+
1
bc
+
1
ca
=
(ab + bc + ca)
2
abc(a + b + c)
Suy ra
ab
2
+ bc
2
+ ca
2

(ab + bc + ca)
2
a + b + c
Mt khc, s dng bt ng thc AMGM, ta c
4(ab + bc + ca)
2
3(a + b + c)
+ 2 6
3

(ab + bc + ca)
4
9(a + b + c)
2
Nh th, ta ch cn chng minh
a
2
+ b
2
+ c
2
+ 6
3

(ab + bc + ca)
4
9(a + b + c)
2
3(ab + bc + ca)
Bt ng thc ny hin nhin ng do (a + b + c)
2
3(ab + bc + ca). Vy ta c pcm. ng thc
xy ra khi v ch khi a = b = c = 1.

161 Cho cc s khng m a, b, c, chng minh bt ng thc


1

4a
2
+ bc
+
1

4b
2
+ ca
+
1

4c
2
+ ab

4
a + b + c
Li gii. S dng bt ng thc Holder, ta c
_

cyc
1

4a
2
+ bc
_
2
_

cyc
(b + c)
3
(4a
2
+ bc)
_
8(a + b + c)
3
Nh th, ta ch cn chng minh
(a + b + c)
5
2

cyc
(b + c)
3
(4a
2
+ bc)
Hay

cyc
a
3
(a b)(a c) + 4

cyc
ab(a
2
b
2
)(a b) + abc
_
19

cyc
a
2
18

cyc
ab
_
0
Bt ng thc ny hin nhin ng. Vy ta c pcm. ng thc xy ra khi v ch khi (a, b, c)
(1, 1, 0).

162 Cho cc s thc a, b, c, chng minh bt ng thc


1 + a
2
b
2
(a b)
2
+
1 + b
2
c
2
(b c)
2
+
1 + c
2
a
2
(c a)
2

3
2
Li gii. D dng chng minh c

cyc
1 + ab
a b

1 + bc
b c
= 1
www.VNMATH.com
153
v

cyc
1 ab
a b

1 bc
b c
= 1
Do , s dng bt ng thc x
2
+y
2
+z
2
xy +yz +zx v x
2
+y
2
+z
2
2(xy +yz +zx) vi
mi x, y, z R, ta c

cyc
_
1 + ab
a b
_
2

cyc
1 + ab
a b

1 + bc
b c
= 1

cyc
_
1 ab
a b
_
2
2

cyc
1 ab
a b

1 bc
b c
= 2
T , suy ra
2

cyc
1 + a
2
b
2
(a b)
2
=

cyc
_
1 + ab
a b
_
2
+

cyc
_
1 ab
a b
_
2
3
Bt ng thc c chng minh xong. ng thc xy ra chng hn khi (a, b, c) =
_

3, 0,

3
_
.

163 Cho cc s khng m a, b, c, chng minh rng


a
2
b
+
b
2
c
+
c
2
a
3
_
a
4
+ b
4
+ c
4
a
2
+ b
2
+ c
2
Li gii. Bt ng thc tng ng

cyc
a
4
b
2
+ 2

cyc
a
2
b
c

9(a
4
+ b
4
+ c
4
)
a
2
+ b
2
+ c
2
Hay

cyc
_
a
4
b
2
+ b
2
2a
2
_
+2

cyc
_
a
2
b
c
+ bc 2ab
_
3
_
3(a
4
+ b
4
+ c
4
)
a
2
+ b
2
+ c
2

cyc
a
2
_
+2
_

cyc
a
2

cyc
ab
_

cyc
S
a
(b c)
2
0
trong
S
a
=
b
2
c
2
+
2b
c
+
2a
b

3(b + c)
2
a
2
+ b
2
+ c
2
S
b
=
c
2
a
2
+
2c
a
+
2b
c

3(c + a)
2
a
2
+ b
2
+ c
2
S
c
=
a
2
b
2
+
2a
b
+
2c
a

3(a + b)
2
a
2
+ b
2
+ c
2
Khng mt tnh tng qut, ta ch cn xt trng hp a b c l . Khi , d thy rng S
a
0.
Xt 2 trng hp
Trng hp 1. Nu b c a b 0, suy ra 2(b c) a c v 2b a + c. Ta c
S
a
+ S
c
=
b
2
c
2
+
2b
c
+
2a
b
+
c
2
a
2
+
2c
a
+
2b
c

3(a + c)
2
+ 3(b + c)
2
a
2
+ b
2
+ c
2
9
3(a + c)
2
+ 3(b + c)
2
a
2
+ b
2
+ c
2
=
3((2a b)
2
+ (2c b)
2
)
2(a
2
+ b
2
+ c
2
)
0
www.VNMATH.com
154 CHNG 2. SOLUTION
S
a
+ 4S
b
=
4c
2
a
2
+
8c
a
+
10b
c
+
2a
b
+
b
2
c
2

3(b + c)
2
+ 12(a + c)
2
a
2
+ b
2
+ c
2

4c
a
1 +
8c
a
+
5a
c
+ 5 +
2a
b
+
b
2
c
2

3(b + c)
2
+ 12(a + c)
2
a
2
+ b
2
+ c
2
22
3(b + c)
2
+ 12(a + c)
2
a
2
+ b
2
+ c
2
=
10a
2
+ 19b
2
+ 7c
2
24ca 6bc
a
2
+ b
2
+ c
2
0
S
a
+ 4S
b
+ S
c
=
4c
2
a
2
+
10c
a
+
10b
c
+
4a
b
+
b
2
c
2
+
a
2
b
2

3(b + c)
2
+ 12(a + c)
2
+ 3(a + b)
2
a
2
+ b
2
+ c
2
26
3(b + c)
2
+ 12(a + c)
2
+ 3(a + b)
2
a
2
+ b
2
+ c
2
=
11a
2
+ 20b
2
+ 11c
2
24ab
a
2
+ b
2
+ c
2
0
Nh th
+, Nu S
b
0 th

cyc
S
a
(b c)
2
(S
a
+ S
c
)(a b)
2
0
+, Nu S
b
0, S
c
0 th

cyc
S
a
(b c)
2
(S
a
+ 4S
b
)(b c)
2
0
+, Nu S
b
, S
c
0 th

cyc
S
a
(b c)
2
(S
a
+ 4S
b
+ S
c
)(b c)
2
0
Trng hp 2. Nu a b b c 0, ta s chng minh S
c
0, tht vy xt hm s f(c) = S
c
=
a
2
b
2
+
2a
b
+
2c
a

3(a+b)
2
a
2
+b
2
+c
2
, r rng f(c) l hm ng bin nn
f(c) f(max{0, 2b a})
Nu a 2b th
f(c) f(0) =
a
2
b
2
+
2a
b

3(a + b)
2
a
2
+ b
2
8
3(a + b)
2
a
2
+ b
2
0
Nu 2b a th
f(c) f(2b a) =
a
2
b
2
+
2a
b
+
4b
a
2
3(a + b)
2
2a
2
4ab + 5b
2
0
Vy ta c S
a
, S
c
0. Nh vy nu S
b
0 th bt ng thc l hin nhin, ngc li nu S
b
0, ta
c
S
a
+ 2S
b
=
2c
2
a
2
+
4c
a
+
6b
c
+
b
2
c
2
+
2a
b

6(a + c)
2
+ 3(b + c)
2
a
2
+ b
2
+ c
2

8c
a
2 +
8b
c
1 +
2a
b

6(a + c)
2
+ 3(b + c)
2
a
2
+ b
2
+ c
2
12
6(a + c)
2
+ 3(b + c)
2
a
2
+ b
2
+ c
2
0
S
c
+ 2S
b
=
2c
2
a
2
+
6c
a
+
4b
c
+
a
2
b
2
+
2a
b

6(a + c)
2
+ 3(a + b)
2
a
2
+ b
2
+ c
2

_
2

2 + 6
_
c
a
1 +
4b
c
+
4a
b
1
6(a + c)
2
+ 3(a + b)
2
a
2
+ b
2
+ c
2
13.6
6(a + c)
2
+ 3(a + b)
2
a
2
+ b
2
+ c
2
0
www.VNMATH.com
155
Do

cyc
S
a
(b c)
2
(S
a
+ 2S
b
)(b c)
2
+ (S
c
+ 2S
b
)(a b)
2
0
Bt ng thc c chng minh xong. ng thc xy ra khi v ch khi a = b = c.

164 Cho cc s dng a, b, c, chng minh rng


_
a
b
+
b
c
+
c
a
2 +
8abc
(a + b)(b + c)(c + a)
2
Li gii. Cch 1. t x =
a
b
, y =
b
c
, z =
c
a
th ta c xyz = 1 v bt ng thc tr thnh
_
x + y + z 2 +
8
(x + 1)(y + 1)(z + 1)
2
Ch rng trong 3 s x, y, z lun tn ti t nht 2 s khng ln hn 1 hoc khng b hn 1, chng
hn (x 1)(y 1) 0, suy ra (x + 1)(y + 1) 2(xy + 1). t t =

xy, khi ta c
V T

2t + z 2 +
4
(z + 1)(t + 1)
=

2t
3
2t
2
+ 1
t
+
4t
2
(t
2
+ 1)
2
Ta cn chng minh

2t
3
2t
2
+ 1
t
+
4t
2
(t
2
+ 1)
2
2
Hay

2t
3
2t
2
+ 1
t
1 1
4t
2
(t
2
+ 1)
2
(t 1)
2
_
2t + 1
t

2t
3
2t
2
+ 1 + t
2

(t + 1)
2
(t
2
+ 1)
2
_
0
Theo bt ng thc AMGM, ta c
2t + 1
t

2t
3
2t
2
+ 1 + t
2

(t + 1)
2
(t
2
+ 1)
2

2t + 1
t
2
+(2t
3
2t
2
+1)
2
+ t
2

(t + 1)
2
(t
2
+ 1)
2
=
2t
5
3t
4
+ 4t
3
+ 2t
2
+ 2t + 1
(2t
3
+ t
2
+ 1)(t
2
+ 1)
2
0
Bt ng thc c chng minh xong. ng thc xy ra khi v ch khi a = b = c.
Cch 2. Khng mt tnh tng qut, gi s c = min{a, b, c}. Bt ng thc tng ng
a
b
+
b
c
+
c
a
2 4
32abc
(a + b)(b + c)(c + a)
+
64a
2
b
2
c
2
(a + b)
2
(b + c)
2
(c + a)
2
Ch rng
64a
2
b
2
c
2
(a+b)
2
(b+c)
2
(c+a)
2

8abc
(a+b)(b+c)(c+a)
nn ta ch cn chng minh
a
b
+
b
c
+
c
a
3 3
24abc
(a + b)(b + c)(c + a)
Hay
(a b)
2
ab
+
(a c)(b c)
ac

3(2c(a b)
2
+ (a + b)(a c)(b c))
(a + b)(b + c)(c + a)
c(a b)
2
((a + b)(b + c)(c + a) 6abc) + b(a c)(b c)(a + b)((a + c)(b + c) 3abc) 0
www.VNMATH.com
156 CHNG 2. SOLUTION
Ta chng minh kt qu mnh hn l
c(a b)
2
((a + b)(b + c)(c + a) 8abc) + b(a c)(b c)(a + b)((a + c)(b + c) 3abc) 0
Hay
2c
2
(a b)
4
+ (a c)(b c)(a + b)(c(a b)
2
+ b(a + c)(b + c) 3abc) 0
Ta c
c(a b)
2
+ b(a + c)(b + c) 3abc = ab
2
+ a
2
c + 2b
2
c + bc
2
4abc
ab
2
+ a
2
c + 3bc
2
4abc
_
3
3

3 4
_
abc 0
Bt ng thc c chng minh.

165 Cho cc s thc a, b, c, chng minh bt ng thc


_
a(b + c)
(a + b)(a + c)
_
2
+
_
b(c + a)
(b + c)(b + a)
_
2
+
_
c(a + b)
(c + a)(c + b)
_
2

1
2
Li gii. Ta c

cyc
_
a(b + c)
(a + b)(a + c)
_
2

1
2
=
(a b)
2
(b c)
2
(c a)
2
+ 32a
2
b
2
c
2
2(a + b)
2
(b + c)
2
(c + a)
2
Nn bt ng thc hin nhin ng. ng thc xy ra khi v ch khi (a, b, c) (1, 1, 0).

166 Cho cc s khng m x, y, z tha x + y + z = 1. Chng minh bt ng thc


_
x + y
2
+
_
y + z
2
+
_
z + x
2

11
5
Li gii. Khng mt tnh tng qut, gi s x = max{x, y, z}. t x + z = 2t, x z = 2m th ta c
t m 0. Khi , ta c
_
x + y
2
+
_
y + z
2
+
_
z + x
2
=
_
t + m + y
2
+
_
y + (mt)
2
+
_
(t + m)
2
+ t m = f(m)
Ta c
f

(m) =
8t 1
4((t + m)
2
+ t m)
3/2

1
4(t + m + y
2
)
3/2
+
y
(y + (mt)
2
)
3/2
Ta s chng minh f

(m) 0 bng cch chng min


8t 1
((t + m)
2
+ t m)
3/2

1
(t + m + y
2
)
3/2
Hay
(8t 1)
2
(t + m + y
2
)
3
((t + m)
2
+ t m)
3
(4(x + z) 1)
2
(x + y
2
)
3
(x
2
+ z)
3
+, Nu x y z 0 th ta c 2(x + z) x + y + z = 1, suy ra 4(x + z) 1 1. Do , ta ch
cn chng minh
x + y
2
x
2
+ z
Hay
y(x z) z
2
y
2
(ng)
www.VNMATH.com
157
+, Nu x z y 0, khi , d thy
x + y
2

4
9
(x
2
+ z) 0
Do
(4(x + z) 1)
2
(x + y
2
)
3
(x
2
+ z)
3
= (3x y + 3z)
2
(x + y
2
)
3
(x
2
+ z)
3

4
9
(3x y + 3z)
2
(x + y
2
)
2
(x
2
+ z) (x
2
+ z)
3
Nh vy, ta ch cn chng minh
4
9
(3x y + 3z)
2
(x + y
2
)
2
(x
2
+ z)
2
Hay
4(3x y + 3z)
2
(x
2
+ x(y + z) + y
2
)
2
9(x + y + z)
2
(x
2
+ xz + yz + z
2
)
2
2(3x y + 3z)(x
2
+ x(y + z) + y
2
) 3(x + y + z)(x
2
+ xz + yz + z
2
)
g(x) = 3x
3
+ (y + 6z)x
2
+ 2(2y
2
yz)x 2y
3
+ 3y
2
z 6yz
2
3z
3
0
D thy g(x) l hm ng bin nn
g(x) g(z) = 6z
3
7yz
2
+ 7y
2
z 2y
3
0 (do z y 0)
Vy trong mi trng hp, ta lun c
f

(m) 0
Do , f(m) l hm li. Suy ra
f(m) max {f(0), f(t)}
Nh vy, ta ch cn chng minh
max {f(0), f(t)}
11
5
iu ny c ngha l ta ch cn chng minh bt ng thc cho trong trng hp trong 3 s
x, y, z c 1 s bng 0, hoc trong trng hp trong 3 s x, y, z c 2 s bng nhau.
Trng hp 1. xyz = 0, khng mt tnh tng qut, gi s z = 0. Khi , ta cn chng minh
_
x + y
2
+

y + x
11
5
x, y 0 : x + y = 1
Hay
_
y
2
y + 1 y

y +
6
5
t
4
t
2
+ 1
_
t
2
t +
6
5
_
2
(t =

y [0, 1])
2t
3

22
5
t
2
+
12
5
t
11
25
0 (ng do 1 t 0)
Trng hp 2. (x y)(y z)(z x) = 0, khng mt tnh tng qut, gi s x = z, suy ra
1
2
x 0, y = 1 2x. Khi , ta cn chng minh
_
x + y
2
+
_
y + x
2
+
_
x + x
2

11
5
Hay
_
4x
2
3x + 1 +
_
x
2
+ x x +
6
5
www.VNMATH.com
158 CHNG 2. SOLUTION
_
_
4x
2
3x + 1 +
_
x
2
+ x
_
2

_
x +
6
5
_
2
2
_
(4x
2
3x + 1)(x
2
+ x) 4x
2
+
22
5
x +
11
25
4(4x
2
3x + 1)(x
2
+ x)
_
4x
2
+
22
5
x +
11
25
_
2
196
5
x
3

596
25
x
2
+
16
125
x
121
625
0 (ng do
1
2
x 0)
Bt ng thc c chng minh xong. ng thc khng xy ra.

167 Cho cc s khng m a, b, c, d tha a +b +c +d = 4, tm hng s k >


64
27
nh nht bt ng thc
sau ng
1
k abc
+
1
k bcd
+
1
k cda
+
1
k dab

4
k 1
Li gii. Cho d = 0, a = b = c =
4
3
, ta suy ra c k
48
11
. Ta s chng minh y l gi tr cn tm, tc
l

cyc
1
48 11abc

4
37
t x = ab, y = cd, z = a + b, t = c + d th ta c
z
2
4
x 0,
t
2
4
y 0, bt ng thc c vit
li nh sau
f(x) =
1
48 11xc
+
1
48 11xd
+
96 11yz
2304 528yz + 121xy
2

4
37
Ta c
f

(x) =
242c
2
(48 11xc)
3
+
242d
2
(48 11xd)
3
+
2(96 11yz)
(2304 528yz + 121xy
2
)
3
0
Suy ra f(x) l hm li, do
f(x) max
_
f(0), f
_
z
2
4
__
Li c
f(0) =
1
16
+
1
48 11yz
=
1
16
+
1
48 11cdz

1
16
+
1
48 11
_
c+d+z
3
_
3
=
4
37
f
_
z
2
4
_
=
96 11ut
121yu
2
528ut + 2304
+
4
96 11yz
= g(y)
trong u =
z
2
4
.
D thy g(y) cng l hm li nn
g(y) max
_
g(0), g
_
t
2
4
__
Ta li c
g(0) =
1
16
+
1
48 11ut
=
1
16
+
1
48 11
z
2

z
2
t

1
16
+
1
48 11
_
z+t
3
_
3
=
4
27
g
_
t
2
4
_
=
44(4 zt)(11z
2
t
2
+ 44zt 768)
37(384 11z
2
t)(384 11zt
2
)
+
4
37

4
37
www.VNMATH.com
159
(do zt
(z+t)
2
4
= 4, 11z
2
t
2
+ 44zt 768 114
2
+ 444 768 = 416 < 0)
Bt ng thc c chng minh xong. Vy
k
min
=
48
11
.

168 Cho cc s khng m a, b, c, chng minh bt ng thc


3(a + b + c) 2
_
_
a
2
+ bc +
_
b
2
+ ca +
_
c
2
+ ab
_
Li gii. Nu abc = 0, gi s c = 0 th d thy bt ng thc l hin nhin. Xt trng hp abc > 0,
khng mt tnh tng qut, gi s a b c > 0 v abc = 1, khi , tn ti cc s thc x y z
sao cho a = e
x
, b = e
y
, c = e
z
, suy ra x + y + z = 0, bt ng thc cho c vit li nh sau

cyc
f(x) 0
trong f(x) = 3e
t
2

e
2t
+ e
t
. Ta c
f

(t) =
6e
3t/2
(e
3t
+ 1)
3/2
4e
6t
14e
3t
1
2e
2t
(e
2t
+ e
t
)
3/2
t u = e
3t/2
> 0 th
f

(t) = 0 36u
1
(u
1
+ 1)
3
= (4u
2
+ 14u
1
+ 1)
2
Hay
g(u) = u
4
9u
3
+ 96u
2
+ 4u 20 = 0
Ta c g

(u) = 4u
3
27u
2
+ 192u + 4 =
1
4
u(4u 27)
2
+
39
4
u + 4 > 0, suy ra g(u) l hm ng bin,
li c g(0) = 20 < 0, g(1) = 73 > 0 nn phng trnh g(u) = 0 c duy nht 1 nghim u
0
> 0, t
y ta suy ra f

(t) = 0 c duy nht 1 nghim t


0
, ngoi ra ta c th kim tra c f(t) lm trn
(, t
0
] v li trn [t
0
, +, ).
Tr li bi ton ca ta, xt 2 trng hp
Trng hp 1. y t
0
, khi , s dng bt ng thc Jensen, ta c
f(x) + f(y) 2f
_
x + y
2
_
Ta cn chng minh
2f
_
x + y
2
_
+ f(z) 0
t m = e
(x+y)/2
=

ab e
z
= c, th sau khi ng bc, ta c bt ng thc tng ng
3(2m + c) 2
_
2
_
m
2
+ mc +
_
m
2
+ c
2
_
Bnh phng 2 v ri thu gn, ta c th vit li bt ng thc nh sau
16m
2
+ 20mc + 5c
2
16
_
m(m + c)(m
2
+ c
2
)
Hay
8
_
_
m
2
+ mc
_
m
2
+ c
2
_
2
+ 3c(4mc) 0
Bt ng thc ny hin nhin ng.
www.VNMATH.com
160 CHNG 2. SOLUTION
Trng hp 2. y t
0
, khi , ta c t
0
y y + z t
0
, suy ra
f(y) + f(z) f(t
0
) + f(y + z t
0
)
Mt khc, s dng bt ng thc Jensen, ta li c
f(x) + f(t
0
) 2f
_
x + t
0
2
_
Ta cn chng minh
2f
_
x + t
0
2
_
+ f(y + z t
0
) 0
Bt ng thc ny ng theo trng hp trn. Vy bt ng thc c chng minh xong. ng
thc xy ra khi v ch khi (a, b, c) (1, 1, 0).
Nhn xt. Trong bi ny, c mt k thut ng ch chnh l vic i bin u = e
3t/2
, cc bn hy ngh
xem ti sao ta li khng t u = e
3t/2
cho "tin" m li t nh th? tm cu tr li, ta hy th t
u = e
3t/2
, khi phng trnh f

(t) = 0 tng ng vi
g(u) = 20u
4
4u
3
96u
2
+ 9u 1 = 0
Ta c g

(u) = 80u
3
12u
2
192u+9, n y cc bn c th thy tht kh m xc nh c s bin thin
ca g(u), iu ny s gy kh khn t nhiu cho ta trong vic gii bi ton, ngc li nu ta t u = e
3t/2
th sau khi thu gn, ta li c ngay mt hm ng bin! Xin c nu mt v d n gin cc bn c th
thy r hn iu ny: Xc nh s nghim dng nu c ca phng trnh h(x) = 22x
13
12x
12
1990 = 0,
nu cc bn vn gi nguyn v xt hm s theo bin x th ta c h

(x) = 2x
11
(143x 72), t y, c th
thy h(x) gim trn
_
0,
72
143

v tng trn
_
72
143
, +
_
, ngha l h(x) c 2 khong bin thin. By gi nu ta
t x =
1
t
th phng trnh tng tng
22
t
13

12
t
12
1990 = 0, hay m(t) = 1990t
13
+ 12t 22 = 0, r rng
m(t) l hm ng bin trn (0, +), li c g(0) = 22 < 0, g(1) = 1980 > 0 nn phng trnh m(t) = 0 c
nghim dng t duy nht, ch rng vi mi gi tr t > 0 ch cho ta mt gi tr x > 0, do phng trnh
h(x) = 0 c nghim dng x duy nht. Cc bn thy khng, y l mt k thut kh hay ng khng no?
:)

169 Cho dy dng {x


n
} tha
k

i=1
x
i

k vi mi k = 1, 2, . . . , n, chng minh bt ng thc


x
2
1
+ x
2
2
+ + x
2
n

1
4
_
1 +
1
2
+
1
3
+ +
1
n
_
Li gii. Xt hm s f(x) = x
2
vi x > 0, ta c
f

(x) = 2x > 0, f

(x) = 2 > 0
Suy ra f(x) l hm li, do s dng b Lagrange, ta c
f(x
i
) f
_

i 1
_
+ f

i 1
__
x
i

i 1
__
i = 1, 2, . . . , n
Do
n

i=1
f(x
i
)
n

i=1
f
_

i 1
_
+
n

i=1
f

i 1
__
x
i

i 1
__
www.VNMATH.com
161
S dng k thut nhm Abel, ta c
n

i=1
f

i 1
__
x
i

i 1
__
=
n1

i=1
_
f

i 1
_
f

i + 1

i
__
_
_
i

j=1
x
j

i

j=1
_
_
j
_
j 1
_
_
_
+ f

_
n

n 1
_
_
n

i=1
x
i

i=1
_

i 1
_
_
=
n1

i=1
_
f

i 1
_
f

i + 1

i
__
_
_
i

j=1
x
j

i
_
_
+ f

_
n

n 1
_
_
n

i=1
x
i

n
_
Ch rng vi mi i 1 th

i + 1 +

i +

i 1, hay
1

i +

i 1

1

i + 1 +

i
Hay

i 1

i + 1

i > 0
Suy ra
n

i=1
f

i 1
__
x
i

i 1
__
0
Ta cn phi chng minh
n

i=1
f
_

i 1
_

i=1
1
4i
chng minh, ta ch cn ch rng vi mi i 1 th

i 1
1
2

i
Tht vy, ta c
2

i
_

i 1
_

i +

i 1
__

i 1
_
= 1
Bt ng thc c chng minh xong. ng thc khng xy ra.

170 Cho cc s khng m a, b, c tha 6 a + b + c 3, chng minh bt ng thc

a + 1 +

b + 1 +

c + 1

15 + ab + bc + ca
Li gii. Cch 1. t x
2
= 1 + a, y
2
= 1 + b, z
2
= 1 + c, d = x
2
+ y
2
+ z
2
th ta c x, y, z 1 v
9 d 6, bt ng thc tr thnh
x + y + z
_
18 2d + x
2
y
2
+ y
2
z
2
+ z
2
x
2
Hay
3d + 2(xy + yz + zx) 18 + x
2
y
2
+ y
2
z
2
+ z
2
x
2
S dng gi thit 9 d 6 v bt ng thc AMGM, ta c
3d(d 6)
1
3
d
2
(d 6) (d 6)(x
2
y
2
+ y
2
z
2
+ z
2
x
2
)
www.VNMATH.com
162 CHNG 2. SOLUTION
Suy ra
3d +
6(x
2
y
2
+ y
2
z
2
+ z
2
x
2
)
d
18 + x
2
y
2
+ y
2
z
2
+ z
2
x
2
Ta cn chng minh
xy + yz + zx
3(x
2
y
2
+ y
2
z
2
+ z
2
x
2
)
d
Hay
(xy + yz + zx)(x
2
+ y
2
+ z
2
) 3(x
2
y
2
+ y
2
z
2
+ z
2
x
2
)

cyc
(y + z)(4x y z)(x y)(x z) 0
(5xy + 3zx + 3yz 7z
2
)(x y)
2
+ (4z x y)(x + y)(x z)(y z) 0
Mt khc li c
9z
2
9 x
2
+ y
2
+ z
2
Suy ra
8z
2
x
2
+ y
2

(x + y)
2
2
Do
4z x y 0
T y, vi gi s z = min{x, y, z}, ta c pcm. ng thc xy ra khi v ch khi a = b = c = 1
hoc a = b = c = 2.
Cch 2. Khng mt tnh tng qut, gi s c = min{a, b, c}, suy ra c 2. t t =
_
(a + 1)(b + 1)
th ta c 1 +
a+b
2
t

a + b + 1, bt ng thc cho c vit li nh sau


f(t) =

2t + a + b + 2 +

c + 1
_
t
2
+ 14 + (a + b)(c 1) 0
Ta c
f

(t) =
1
(2t + a + b + 2)
3/2

14 + (a + b)(c 1)
(t
2
+ 14 + (a + b)(c 1))
3/2
< 0
Suy ra f(t) l hm lm, do
f(t) min
_
f
_

a + b + 1
_
, f
_
1 +
a + b
2
__
Ta c f
_
a + b + 1
_
0, tht vy t y =
_
(c + 1)(a + b + 1) th 1+
a+b+c
2
y

a + b + c + 1,
bt ng thc tng ng
g(y) = 1 +
_
2y + a + b + c + 2
_
y
2
+ 14 a b c 0
Ta cng c g

(y) =
1
(2y+a+b+c+2)
3/2

14abc
(y
2
+14abc)
3/2
< 0 nn g(y) cng l hm lm, do
g(y) min
_
g
_

a + b + c + 1
_
, g
_
1 +
a + b + c
2
__
Li c
g
_

a + b + c + 1
_
=

a + b + c + 1 + 2

15 4

15 > 0
g
_
1 +
a + b + c
2
_
= 1 + 2

m + 1
_
m
2
+ 15 = h(m)
vi m =
a+b+c
2

_
3
2
, 3

. Ta c h

(m) =
1
2(m+1)
3/2

15
(m
2
+15)
3/2
< 0 nn h(m) l hm lm, suy ra
h(m) min
_
h
_
3
2
_
, h(3)
_
= min
_

10 + 1

69
2
, 5

24
_
> 0
www.VNMATH.com
163
Ta cn phi chng minh f
_
1 +
a+b
2
_
0, t x =
a+b
2
, suy ra
6c
2
x
3c
2
th bt ng thc
tng ng
u(x) = 2

x + 1 +

c + 1
_
x
2
+ 2xc + 15 0
Ta c u

(x) =
1
2(x+1)
3/2

15c
2
(x
2
+2xc+15)
3/2
< 0, suy ra u(x) l hm lm, do
u(x) min
_
u
_
6 c
2
_
, u
_
3 c
2
__
Li c
u
_
6 c
2
_
=
3(c 2)
2
(20 + 20c 3c
2
)
2
_
2
_
2(8 c) + 2

c + 1 +

3c
2
+ 12c + 96
__
8
_
2(8 c)(c + 1) 3c
2
+ 16c + 28
_
u
_
3 c
2
_
=
3(c 1)
2
(5 c)(3c + 1)
2
_
2
_
2(5 c) + 2

c + 1 +

3c
2
+ 6c + 69
__
8
_
2(5 c)(c + 1) 3c
2
+ 10c + 25
_
Bt ng thc c chng minh.

www.VNMATH.com
164 CHNG 2. SOLUTION
2.2. Tc gi cc bi ton
Phm Th Hng () [18]
V Quc B Cn () [1] [6] [21] [26] [27] [28] [35] [36] [44] [45] [46] [47] [48] [50] [52] [53] [54]
[55] [56] [57] [59] [61] [63] [64] [65] [66] [68] [73] [76] [77] [78] [80] [86] [89]
[90] [92] [94] [95] [97] [101] [102] [103] [106] [107] [108] [109] [110] [113]
[114] [117] [122] [123] [126] [135] [139] [140] [141] [142] [144] [150] [151]
[152] [158] [166] [167] [168]
Nguyn Vn Thch [17] [23] [24] [38] [44] [61] [84] [109] [162] [163] [164]
Nguyn Phi Hng [51]
Phan Hng Sn [9] [10] [100] [104] [165]
Phm Kim Hng [3] [7] [8] [20] [30] [31] [42] [70] [118] [120] [137] [143] [156] [160] [161]
Phm Vn Thun [39] [40] [43] [49] [81] [82] [85] [119] [121] [127] [134] [154] [157]
Phm Hu c [4] [29] [34] [37] [41] [69] [87] [93] [125] [128] [132] [145] [147] [159]
Vasile Cirtoaje [14] [33] [83] [84] [111] [158]
Nguyn Anh Cng [16] [22] [124]
Phm Sinh Tn [58] [115]
Phan Thnh Nam [13] [91]
L Vn Chnh [11]
Darij Grinberg [62]
Gabriel Dospinescu [74] [146]
Kiran Kedlaya [75]
Nguyn Anh Tun [96] [129]
V nh Qu [98] [136]
Thomas J. Mildorf [99]
Phan Thnh Vit [116]
Iurie Borieco [133]
Ivan Borsenco [133]
inh Tun ng [2]
Titu Andreescu [169]
www.VNMATH.com

You might also like